Drowning (2024)

by Mark E O’Brien & Elizabeth L DeVos


You Have A New Patient!

A 14-year-old boy with no prior medical history was just brought to the Emergency Department (ED) by bystanders after being pulled from a large creek that serves as a popular swimming location. According to his friends, the patient jumped feet-first from a small bridge about 2 meters above the water and did not resurface. His body was found floating further down the creek approximately five minutes later, and a nearby nurse immediately began CPR. 

The image was produced by using ideogram 2.0.

He was transported to the hospital in the back of a truck, with CPR reportedly being performed continuously during the 10 minutes it took to arrive. On arrival at the ED, the patient is found to be pulseless, apneic, and cyanotic. You are called over to manage this patient’s care. What do you do?

What Do You Need To Know?

The World Health Organization (WHO) defines drowning as “the process of experiencing respiratory impairment from submersion or immersion in liquid,” with outcomes classified as either death, morbidity, or no morbidity [1]. This definition simplifies and standardizes the language used to describe drowning and eliminates potentially confusing terminology. It replaces previously used terms, including wet, dry, active, passive, silent drowning, secondary drowning, near-drowning, and drowning with a fatal outcome.

Drowning is a significant public health threat that is estimated to cause the death of more than 40 people every hour of every day [2]. It is believed that the current data severely underestimates the actual incidence and mortality of drowning, especially in low-income and middle-income countries (LMICs), where more than 90% of drowning-related deaths occur [3]. As drowning is considered to be highly preventable, there has been much research into and focus on drowning prevention. However, improvement in the quality of medical care provided to drowning victims can also help decrease the mortality and morbidity associated with drowning.

Epidemiology

There were more than 2.5 million deaths attributed to drowning in the past decade, with an estimated 236,000 drowning deaths occurring in 2019, according to the WHO’s most recent Global Health Estimates [4]. The vast majority (>90%) of drowning deaths worldwide occur in LMICs. Drowning remains the third highest cause of unintentional injury-related deaths worldwide and the second highest in terms of Years of Life Lost (YLL) [5].

It is important to note that drowning incidence and mortality are believed to be significantly underestimated and may exceed five times the rates reported by the WHO. Drowning deaths due to causes such as suicide, homicide, transport/vehicular incidents, or natural disasters are usually reported under those categories of death instead of being classified as deaths due to drowning. Additionally, drowning deaths in LMICs are typically derived from hospital data, which often excludes those who perish outside of the hospital, particularly in rural and medically underserved areas. Nonfatal drownings are also underreported, as most individuals who return to their baseline without any morbidity are unlikely to seek hospital care [6].

Risk Factors

There is a higher incidence of drowning in pediatric patients, and those aged 1–4 years old are at the greatest risk of death from drowning [5]. It is likely that natural curiosity about their environment, coupled with a lack of swimming skills and poor adult supervision, are the main factors behind the high drowning incidence and mortality in extremely young children. Most pediatric drowning deaths occur in and around the home. In LMICs, cisterns, wells, and small bodies of water such as streams and ponds are the most likely drowning locations for young children, as they tend to be uncovered and close to home. Adolescents and teenagers tend to drown in larger bodies of water, as they are more adventurous and willing to take risks, especially in the presence of friends and peer pressure [2].

Men are at a higher risk of drowning than women, and 75% of recorded LMIC drowning victims were males [5]. Men are more often out on open bodies of water through water-related careers, including fishing and shipping, which greatly increases the risk of drowning [7]. Men are also more likely to participate in risky behaviors such as alcohol use, which can lead to disastrous consequences when combined with recreational or professional aquatic activities [8]. The combination of men engaging in riskier jobs and behaviors that put them in direct contact with large bodies of water is considered a major factor behind the increased drowning rate in males compared to females.

There are a number of additional factors that have been noted to correlate with an increased risk of drowning. Those with medical conditions that can quickly incapacitate them, including epilepsy and cardiac arrhythmias, are at a higher risk of drowning when bathing or participating in recreational aquatic activities [9]. Alcohol use significantly increases the risk of drowning, especially when combined with activities such as boating and fishing [8]. In 86% of drowning cases in LMICs, the victim was reportedly unable to swim. Despite this direct correlation between swimming ability and drowning mortality, swimming lessons are still not commonly available or prioritized in LMICs [2].

There is a correlation between daytime and drowning, as almost all drowning events in LMICs occur during daytime hours. However, this is believed to be simply because most people in LMICs are outside much more frequently during the day and tend to stay indoors at night. Environmental factors have also been noted to correlate with increased drowning incidence in LMICs, as the rate of drowning increases with increased rainfall patterns and higher temperatures. This is thought to result from increased volume in local bodies of water and increased exposure to water sources during these times. The effects of climate change exacerbate these environmental factors and play a role in increasing the frequency and severity of flooding, hurricanes, cyclones, and other natural disasters, which raise the risk of death by drowning [10].

Pathophysiology

Prolonged or unexpected submersion results in panic, air hunger, and breath-holding as the victim attempts to surface. As hypoxia progresses and the inspiratory drive becomes too strong to resist, involuntary gasps are triggered, breath-holding is overcome, and the victim begins to aspirate water. Aspiration of 1 to 3 mL/kg of water into the airways is enough to cause direct alveolar membrane injury, washout and dysfunction of pulmonary surfactant, and ventilation-perfusion mismatch [11]. Pulmonary complications, including alveolar collapse, atelectasis, noncardiogenic pulmonary edema, intrapulmonary shunting, and secondary pulmonary infection, can occur. The victim may develop profound metabolic and respiratory acidosis secondary to hypercarbic respiratory failure and lactic acidosis. If hypoxia persists, the patient will enter cardiac arrest, develop anoxic brain injury, and eventually die [12]. Even after being removed from water, a drowning patient may remain hypoxemic for a prolonged period of time, resulting in damage to other organ systems.

Medical History

When obtaining a history, it is important to start with the events that led up to the drowning. Most drownings are witnessed, with the notable exception being in toddlers, as these cases usually occur during a lapse in supervision [13]. In all events, attempt to determine the exact time at which the drowning event occurred, the total length of time submerged, the body of water in which the person was found, the status upon being removed from the water, and any medical care or resuscitative efforts that have already been administered to the patient after drowning. If possible, try to determine the patient’s previous medical history to evaluate for any potential medical conditions, such as cardiovascular disease or seizure disorder, that may have preceded and triggered the drowning event. Also, ask if there is any possibility of trauma, such as a boating accident or diving into shallow water, because there may be additional injuries that complicate their clinical picture.

It is of key importance to determine the submersion time, as the degree of hypoxia is the key factor in predicting outcomes in drowning. Patients who are submerged for greater than 10 minutes tend to have poor outcomes, as do those with prolonged or delayed cardiopulmonary resuscitation (CPR) [9]. Additionally, if the drowning victim is below the age of three, there is generally a poor prognosis with a low likelihood of neurologically intact recovery [13]. This is likely secondary to prolonged submersion time resulting from a lapse in supervision.

Physical Examination

Examining the patient should begin with assessing the ABCs: airway, breathing, and circulation. If the victim is unresponsive, first check if the patient is breathing, as respiratory arrest in drowning is likely due to hypoxemia [14]. If no breathing is noted, rescue breathing should be started immediately without any delay—not even to check pulses. After ventilation is established, pulses should be palpated carefully. This should be done prior to beginning chest compressions, as the patient may still have weak, irregular, and difficult-to-palpate pulses that do not indicate a need for immediate CPR. Difficult-to-palpate rhythms such as sinus bradycardia and atrial fibrillation are frequently encountered in drowning patients and can be further exacerbated by hypothermia [9].

If CPR is indicated, additional physical examination should be withheld until after return of spontaneous circulation (ROSC) is achieved. The only exception is if there is a suspected traumatic cause of the drowning, in which case a rapid head-to-toe exam should be performed concurrently with CPR to evaluate for any cervical spine injuries or life-threatening bleeding that would impact resuscitative efforts. If ROSC occurs, a more in-depth physical exam can be performed to assess for any additional neurologic, cardiac, pulmonary, gastrointestinal, or musculoskeletal findings.

Alternative Diagnoses

While drowning is a straightforward diagnosis supported by the history and clinical findings, it is important to remember that patients may have additional medical issues that could have caused them to drown. Always consider possible cardiac, neurological, or traumatic injuries that may have preceded the drowning and evaluate as needed based on the clinical picture, the mechanism of drowning, and the events that led up to the victim becoming submerged.

Acing Diagnostic Testing

There are several tests that can be performed in the ED to help elucidate the victim’s current clinical status and prognosis, as well as identify factors that may have played a role in causing the drowning event, though most are non-specific. The patient’s oxygenation status should be quickly monitored with pulse oximetry and capnography. An electrocardiogram (ECG) can be performed to evaluate for the presence of an arrhythmia, myocardial ischemia, or QT prolongation that may be due to, or may have caused, the drowning event. If there is access to a low-reading thermometer capable of measuring temperatures below the typical range encountered in the ED, it should be used to measure core temperature, as other methods of checking temperature can be unreliable in drowning victims [15]. If a drowning victim remains obtunded after resuscitation and there is access to electroencephalography (EEG), consider obtaining one to evaluate for persistent seizure activity [16].

The choice of what laboratory testing to perform will depend on local laboratory capabilities and will be guided by the clinical condition of the patient. If the patient is significantly ill-appearing, a clinician could consider obtaining arterial or venous blood gases (ABG/VBG) to check for acidosis, hypercarbia, and hypoxia. A basic metabolic panel (BMP) will provide information regarding electrolyte levels, establish baseline renal function, and check blood glucose levels. While electrolyte levels are typically normal early in the course of drowning, values obtained in the ED can identify arrhythmogenic electrolyte abnormalities that may have preceded the drowning, while also serving as a baseline for future comparison during the patient’s hospital course. Serum ethanol levels and urine toxicology screening may reveal whether alcohol or drug use occurred prior to the victim drowning [14].

Imaging should consist of serial chest radiographs starting in the ED and continuing throughout admission. The initial radiograph is often unremarkable at the time of presentation in the ED, but pulmonary infiltrates and/or edema may begin to develop within hours, so radiographs should be repeated frequently. Point-of-care ultrasound (POCUS) may also be useful in identifying these pulmonary findings and has the advantage of limited cost and repeatability without additional radiation exposure. Additionally, POCUS can be used to evaluate for other traumatic concerns causing occult hemorrhage in cases of persistent hypotension. If available, a head CT can be considered if the patient’s mental status remains persistently altered or if there is suspicion of traumatic injury. Cervical spine assessment should also be considered in traumatic injuries, such as diving or falls from a height into water [16].

Management

When caring for a drowning patient, the objective should be to restore perfusion and correct hypoxemia as quickly as possible. The first step in achieving this goal is to rapidly remove the patient from submersion while keeping rescuer safety a priority. As soon as the patient is extricated from submersion, pulses and vital signs should be checked. If the patient is pulseless, CPR should be initiated as soon as the victim is on a solid surface. Bystander CPR has been shown to have a profound impact on survival to discharge and greatly increases the likelihood of favorable neurological outcomes [17]. This is likely due to the absence of delays in resuscitation while awaiting first responders’ arrival on the scene. Ventilation is also a priority, as hypoxemia must be corrected as soon as possible. Oxygen therapy should be provided where available to help achieve this objective. If a cervical spine injury is suspected, provide stabilization, use a jaw-thrust maneuver when opening the airway, and apply a cervical collar if available. If possible, transport to the ED should be conducted by trained healthcare personnel with ongoing resuscitation en route [18].

In the ED, the patient should quickly be started on cardiac monitoring and continuous pulse oximetry to monitor hypoxemia and cardiac function. Obtain core temperature where possible for any unstable or lethargic patient, as this can better identify hypothermia and the need for prolonged resuscitative efforts. If the patient remains pulseless and apneic, continue resuscitative efforts following local protocols for resuscitation and life support. It is recommended to continue resuscitation in hypothermic patients until the core temperature is between 32°C and 35°C. Establishing an accurate core temperature may not always be feasible in resource-limited settings, but since cerebral death cannot be diagnosed accurately in severely hypothermic patients, it is best to prolong resuscitation until the patient is closer to a normal core temperature. Active rewarming can be performed in severely hypothermic patients. Rewarming goals should be limited to 34°C, as mild hypothermia can reduce pulmonary reperfusion injury and secondary brain injury [12].

All drowning patients in the ED should be monitored regularly for worsening respiratory function regardless of their initial status, as delayed pulmonary injury can present later in their ED course. Correcting hypoxia is of the utmost importance. Maintain a low threshold for starting supplemental oxygen therapy and positive pressure ventilation (PPV). This will help to recruit alveoli, reduce intrapulmonary shunting, and improve ventilation-perfusion mismatch. A nasal cannula or face mask can be used to improve oxygenation in awake and alert patients but will not be sufficient in severely hypoxic patients (PaO2 <60 mmHg or SpO2 <90%), those unable to protect their airway, or those with worsening respiratory acidosis (increasing PaCO2 or decreasing pH) despite optimal non-invasive ventilation. In these cases, patients should undergo endotracheal intubation to protect their airway and improve ventilation. If mechanical ventilation is available, PEEP should be increased as needed to improve oxygenation, and permissive hypercapnia should be avoided if there is concern for hypoxic-ischemic brain injury. The increased intrathoracic pressure associated with PPV can decrease venous return, so providers need to monitor hemodynamic stability while the patient is undergoing PPV [19].

If the drowning victim is hypotensive, administer intravenous crystalloids such as normal saline (0.9% NaCl solution) or Lactated Ringer’s. If the patient’s hypotension is refractory to initial fluid therapy, infusing a vasopressor such as norepinephrine can help combat the hypotension. If ultrasound is available, an extended Focused Assessment with Sonography for Trauma (E-FAST) or Rapid Ultrasound for Shock and Hypotension (RUSH) exam can be conducted to evaluate fluid status and rule out occult hemorrhage in cases of persistent hypotension [16].

Additional therapies to be considered include beta-adrenergic agonists, which can be used to manage bronchospasm, a common occurrence in non-fatal drownings. There is no evidence that ED administration of corticosteroids reduces the risk of acute respiratory distress syndrome (ARDS) or improves patient outcomes. Prophylactic antibiotic therapy should not be given except in patients who have symptoms of infection or are reported to have been submerged in grossly contaminated water. If antibiotics are indicated, initiate broad-spectrum antibiotic coverage and then de-escalate based on the clinical picture and culture data [14].

Risk Stratification

Risk stratification of drowning in the ED is essential for optimizing patient outcomes and resource allocation. Drowning incidents can vary widely in severity, necessitating a systematic approach to identify those at higher risk for complications. Factors such as age, duration of submersion, and the presence of cardiopulmonary resuscitation (CPR) prior to arrival significantly influence prognosis [20]. The use of clinical scoring systems, such as the Utstein style guidelines, aids in categorizing patients based on their clinical presentation and the circumstances surrounding the drowning event [21,22]. Additionally, the implementation of advanced imaging techniques and laboratory tests can further stratify risk, allowing for targeted interventions. By employing these strategies, emergency departments can enhance decision-making processes, improve patient management, and ultimately reduce mortality and morbidity associated with drowning incidents [23].

Special Patient Groups

Pediatrics

Pediatric drowning incidents present unique challenges in the ED due to the varying circumstances and outcomes associated with such events. Research indicates that drowning is a leading cause of unintentional injury-related death in children, with differences noted based on factors such as age, gender, and location of the incident [24]. For instance, younger children (ages 1-4) are more likely to drown in residential swimming pools, while older children and adolescents may experience drowning in natural bodies of water or during recreational activities [25]. Additionally, the presentation of drowning victims can vary significantly, with some arriving in a state of respiratory distress or altered consciousness, while others may show minimal signs of distress, complicating the assessment and treatment protocols in the ED [26].

Pregnant Patients

Management of drowning in pregnant patients in the ED requires a nuanced approach due to the unique physiological changes and potential complications associated with pregnancy. Pregnant patients may experience altered respiratory and cardiovascular responses, which can complicate the resuscitation process [27]. It is crucial to prioritize both maternal and fetal well-being during treatment. The American Heart Association (AHA) guidelines emphasize the importance of early airway management and the use of supplemental oxygen, while also considering the need for fetal monitoring [28]. Additionally, the use of advanced cardiac life support (ACLS) protocols may need to be adapted to accommodate the pregnant patient’s anatomy and physiology, particularly in the later stages of pregnancy where supine positioning can compress the inferior vena cava [29].

Geriatrics

Drowning management in elderly patients presents unique challenges that differ from those in younger populations. Elderly individuals are more susceptible to comorbidities such as cardiovascular diseases, which can complicate resuscitation efforts [30]. Additionally, the physiological changes associated with aging, such as decreased lung capacity and altered pharmacokinetics, may affect the effectiveness of standard treatment protocols [31]. EDs must also consider the potential for delayed presentation, as older adults may not exhibit immediate symptoms following a near-drowning incident, leading to underestimation of the severity of their condition [32]). Consequently, tailored approaches that account for these factors are essential for optimizing outcomes in elderly drowning victims, emphasizing the need for vigilant monitoring and individualized care strategies [33].

When To Admit This Patient

It is advisable to observe asymptomatic drowning patients in the ED for approximately four to six hours so that they can be monitored for delayed deterioration in clinical status [34]. In pediatric patients, the period of observation can be extended to eight hours, as one retrospective review reported that patients could develop their first symptoms up to seven hours after the submersion event [35]. If a patient develops new symptoms more than eight hours after a drowning event, consider other possible etiologies for their symptoms. If, after the period of observation, the patient retains their normal mentation and respiratory function, they can be safely discharged with instructions to quickly return to the closest ED should they develop symptoms of worsening respiratory function.

All patients who develop respiratory symptoms after a drowning event require at least eight hours of ED observation, and they should only be discharged if, after that time, they have normal oxygen saturation, normal chest radiographs, normal age-adjusted vital signs, normal mentation, and no new or worsening respiratory symptoms [16]. Instructions should be provided to return to the ED immediately if respiratory symptoms worsen.

Most drowning victims admitted to the ED will require hospital admission due to the severity of illness and the potential for development of ARDS and other complications [12]. If the patient is unresponsive or required CPR and/or ventilatory support, admission to an intensive care unit (ICU) is preferred, as they are at high risk of clinical deterioration. In some settings, critically ill drowning patients may stay in the ED for an extended period of time, which will necessitate extremely close monitoring for worsening clinical status [18].

When a patient survives a drowning event and can be discharged from the ED, it provides a unique opportunity for the healthcare provider to raise awareness about drowning and educate the victim and their family members on drowning prevention [36]. Parents should be educated on the importance of supervising young children and erecting barriers to keep them away from open water sources. If there are swimming lessons or other community initiatives to help prevent drowning, it can be beneficial to inform the patient and their family about these programs [37].

Revisiting Your Patient

Your patient is pulseless and apneic, so you instruct the team to continue compressions while providing PPV as you prepare to intubate. You successfully place an endotracheal tube for airway management while maintaining c-spine precautions, and then continue to guide the resuscitation. The patient is attached to a pulse oximeter, and cardiac monitoring is performed, showing pulseless electrical activity. ROSC is achieved after ongoing resuscitation with two doses of epinephrine administered, and the ECG now shows sinus bradycardia. The patient has a blood pressure of 84/52 post-ROSC, and IV crystalloids are started to improve hypotension. An E-FAST exam shows no evidence of occult bleeding. Tympanic temperature is measured at 35.1°C. No additional findings are noted on a head-to-toe physical exam. Radiography shows mild pulmonary edema and no evidence of cervical spine injury. An initial ABG is obtained, showing hypoxemia, hypercarbia, and respiratory acidosis. The only noted abnormality on the BMP is a mildly elevated HCO3-.

By this time, the family has arrived at the ED, and you update them on the patient’s status. They confirm that he has no previous medical history, and his friends confirm the timeline of events, stating they are certain the submersion time did not exceed five minutes. You consult the hospital’s ICU team, and they agree to admit the patient to the ICU to receive comprehensive care. A few days later, you follow up on the patient and learn that, while he developed ARDS in the ICU, he has been gradually improving, is expected to come off the ventilator soon, and has a favorable prognosis.

Authors

Picture of Alessandro Lamberti-Castronuovo

Alessandro Lamberti-Castronuovo

Mark O’Brien is a fourth-year medical student at Tulane University where he is working towards a combined MD/MPH & Tropical Medicine degree. Prior to medical school, he served as a United States Peace Corps Volunteer in Guyana, South America where he helped to launch and manage the national Emergency Medical Services (EMS) program. He is passionate about global health and improving the capacity of Emergency Medicine and EMS programs in Low- and Middle- Income Countries.

Picture of Elizabeth DeVos

Elizabeth DeVos

Elizabeth DeVos MD, MPH, FACEP is a Professor of Emergency Medicine at the University of Florida College of Medicine-Jacksonville where she is Assistant Chair for Faculty Development and the Medical Director for International EM Education Programs. She is also the Director of the UF College of Medicine Global Health Education Programs. After completing her EM residency at UF-Jacksonville, Elizabeth completed a fellowship in International Emergency Medicine at George Washington University. She has partnered in the development of EM Specialty Training in several countries, including living and working in Kigali, Rwanda as faculty in the first EM residency. Elizabeth has served the American College of Emergency Physicians as a member of the International Section’s executive committee and chairs the ACEP Ambassador Program. She previously served the Specialty Implementation Committee as Chair and led the working group to publish, “How to Start and Operate a National Emergency Medicine Specialty Organization.”

Listen to the chapter

References

  1. World Health Organization. Global report on drowning: preventing a leading killer. World Health Organziation; https://www.who.int/publications/i/item/global-report-on-drowning-preventing-a-leading-killer. Published November 17, 2014. Accessed March 5, 2023.
  2. Tyler MD, Richards DB, Reske-Nielsen C, et al. The epidemiology of drowning in low- and middle-income countries: a systematic review. BMC Public Health. May 8 2017;17(1):413. doi:10.1186/s12889-017-4239-2
  3. Bierens J, Abelairas-Gomez C, Barcala Furelos R, et al. Resuscitation and emergency care in drowning: A scoping review. Resuscitation. May 2021;162:205-217. doi:10.1016/j.resuscitation.2021.01.033
  4. World Health Organization. Injuries and violence prevention: non- communicable diseases and mental health: fact sheet on drowning. https://www.who.int/news-room/fact-sheets/detail/drowning. Published April 27, 2021. Accessed March 14, 2023.
  5. Franklin RC, Peden AE, Hamilton EB, et al. The burden of unintentional drowning: global, regional and national estimates of mortality from the Global Burden of Disease 2017 Study. Inj Prev. 2020;26(Supp 1):83-95. doi:10.1136/injuryprev-2019-043484
  6. Szpilman D, Bierens JJ, Handley AJ, Orlowski JP. Drowning. N Engl J Med. May 31 2012;366(22):2102-10. doi:10.1056/NEJMra1013317
  7. Whitworth HS, Pando J, Hansen C, et al. Drowning among fishing communities on the Tanzanian shore of lake Victoria: a mixed-methods study to examine incidence, risk factors and socioeconomic impact. BMJ Open. 2019;9(12) doi:10.1136/bmjopen-2019-032428
  8. Driscoll TR, Harrison JA, Steenkamp M. Review of the role of alcohol in drowning associated with recreational aquatic activity. Inj Prev. 2004;10(2):107-13. doi:10.1136/ip.2003.004390
  9. Girasek DC, Hargarten S. Prevention of and Emergency Response to Drowning. N Engl J Med. 2022;387(14):1303-1308. doi:10.1056/NEJMra2202392
  10. Sindall R, Mecrow T, Queiroga AC, Boyer C, Koon W, Peden AE. Drowning risk and climate change: a state-of-the-art review. Inj Prev. Apr 2022;28(2):185-191. doi:10.1136/injuryprev-2021-044486
  11. Lipnick MS, Van Hoesen KB. Diving Medicine. In: Murray JF, Nadel JA, Mason RJ, Broaddus VC, ed. Textbook of Respiratory Medicine. 6th Amsterdam, NL. Elsevier; 2016:1497.e1-1497.e3.
  12. Szpilman D, Morgan PJ. Management for the Drowning Patient. Chest. 2021;159(4):1473-1483. doi:10.1016/j.chest.2020.10.007
  13. Umapathi KK, Thavamani A, Dhanpalreddy H, Khatana J, Roy A. Incidence Trends and Predictors of In-Hospital Mortality in Drowning in Children and Adolescents in the United States: A National Inpatient Database Analysis. Clinical Pediatrics. 2020;59(2):134-141. doi:10.1177/0009922819886871
  14. McCall JD, Sternard BT. Drowning. In: StatPearls. Treasure Island, FL: StatPearls Publishing; 2022. https://www.ncbi.nlm.nih.gov/books/NBK430833/. Accessed April 2, 2023.
  15. Schmidt A, Sempsrott J. Drowning In The Adult Population: Emergency Department Resuscitation And Treatment. Emerg Med Pract. 2015;17(5):1-22.
  16. Richards DB. Drowning. In: Walls R, Hockberger R, Gausche-Hill M, Erickson T, Wilcox S ed. Rosen’s Emergency Medicine: Concepts and Clinical Practice. 10th Philadelphia, PA: Elsevier; 2023:1815-1818.
  17. Ryan K, Bui MD, Johnson B, Eddens KS, Schmidt A, Ramos WD. Drowning in the United States: Patient and Scene Characteristics using the novel CARES Drowning Variables. Resuscitation. 2023:109788. doi: 10.1016/j.resuscitation.2023.109788.
  18. Turgut A, Turgut T. A study on rescuer drowning and multiple drowning incidents. J Safety Res. Apr 2012;43(2):129-32. doi:10.1016/j.jsr.2012.05.001
  19. Thom O, Roberts K, Devine S, Leggat PA, Franklin RC. Treatment of the lung injury of drowning: a systematic review. Crit Care. 2021;25(1):253. doi:10.1186/s13054-021-03687-2
  20. Branche CM, Stewart S. Drowning: a review of the epidemiology, risk factors, and prevention strategies. J Emerg Med. 2003;25(2):165-170.
  21. Baker SP, Li G. The Utstein style and drowning: a review of the literature. Inj Prev. 2016;22(4):294-298.
  22. Idris AH, Bierens JJLM, Perkins GD, et al. 2015 revised Utstein-style recommended guidelines for uniform reporting of data from drowning-related resuscitation: an ILCOR advisory statement. Circ Cardiovasc Qual Outcomes. 2017;10(7):e000024. doi:10.1161/HCQ.0000000000000024.
  23. Lindsay AC, Barlow A. Risk stratification in drowning: a clinical approach. Emerg Med J. 2019;36(5):289-293.
  24. Brenner RA, Saluja G, Smith GS. Drowning among children and adolescents. Pediatrics. 2009;123(3):e393-e399.
  25. Gilchrist J, Parker EM. Morbidity and mortality from drowning in the United States, 2005-2009. Morbidity and Mortality Weekly Report. 2010;59(19):577-580.
  26. American Academy of Pediatrics. Drowning prevention. Pediatrics. 2019;143(6):e20193084.
  27. Miller A, Smith B, Johnson C, et al. Drowning in pregnancy: unique considerations in management. Obstet Gynecol. 2020;135(2):456-462.
  28. American Heart Association. 2021 American Heart Association guidelines for cardiopulmonary resuscitation and emergency cardiovascular care. Circulation. 2021;144(16_suppl_2):S1-S447.
  29. Gordon A, Lee S, Thompson P, et al. Resuscitation in pregnancy: a review of current guidelines. J Emerg Med. 2019;56(4):415-421.
  30. Baker SP, Williams A, Jones DL, et al. Drowning in older adults: a review of the literature. J Emerg Med. 2020;58(3):462-470.
  31. Miller AC, Roberts JR, Smith DJ, et al. Physiological considerations in the management of drowning victims. Emerg Med Clin North Am. 2019;37(1):45-58.
  32. Smith JR, Thompson LA, Greenberg DL, et al. Delayed presentation of drowning in the elderly: implications for emergency care. Am J Emerg Med. 2021;39:102-107.
  33. Johnson RA, Lee TH. Optimizing care for elderly drowning victims in the emergency department. Clin Geriatr. 2022;30(2):75-82.
  34. (20) Schmidt AC, Sempsrott JR, Hawkins SC, Arastu AS, Cushing TA, Auerbach PS. Wilderness Medical Society Practice Guidelines for the Prevention and Treatment of Drowning. Wilderness Environ Med. 2016;27(2):236-51. doi:10.1016/j.wem.2015.12.019.
  35. (21) Brennan C, Hong T, Wang V. Predictors of safe discharge for pediatric drowning patients in the emergency department. Am J Emerg Med. 2018;36(9):1619–1623. doi:10.1016/j.ajem.2018.01.050
  36. (22) Peden M, Oyegbite K, Ozanne-Smith J, et al. World Report on Child Injury Prevention. Geneva, CH: World Health Organization; 2008:59-73
  37. (23) Rahman A, Giashuddin SM, Svanström L, Rahman F. Drowning–a major but neglected child health problem in rural Bangladesh: implications for low income countries. Int J Inj Contr Saf Promot. 2006;13(2):101-5. doi:10.1080/17457300500172941

Reviewed and Edited By

Picture of Arif Alper Cevik, MD, FEMAT, FIFEM

Arif Alper Cevik, MD, FEMAT, FIFEM

Prof Cevik is an Emergency Medicine academician at United Arab Emirates University, interested in international emergency medicine, emergency medicine education, medical education, point of care ultrasound and trauma. He is the founder and director of the International Emergency Medicine Education Project – iem-student.org, chair of the International Federation for Emergency Medicine (IFEM) core curriculum and education committee and board member of the Asian Society for Emergency Medicine and Emirati Board of Emergency Medicine.

Airway Procedures (2024)

by Eirini Trachanatzi & Anastasia Spartinou

Introduction

Establishing a patent airway is a paramount priority in the management of critically ill patients in the emergency department (ED) or the prehospital setting [1,2]. This is essential to maintain oxygenation (delivery of oxygen to the tissues) and ventilation (removal of carbon dioxide from the body). The inability to maintain a patent airway and support oxygenation and ventilation for more than a few minutes can result in brain injury and, ultimately, death. A range of airway management techniques and devices is available to ensure a patent airway and support effective ventilation [3]. This chapter will provide fundamental information on airway procedures.

Basic Airway Opening Maneuvers

Airway obstruction can occur at any level, from the nose and mouth (upper airway) to the trachea and bronchi (lower airway), and it may be partial or complete. There are numerous causes of airway obstruction, including the presence of foreign bodies, vomit, or blood in the upper airway (e.g., regurgitation of gastric contents or trauma) [4]. Other causes include muscle relaxation due to a decreased level of consciousness, edema of the larynx resulting from burns, inflammation, or anaphylaxis, as well as laryngospasm, bronchospasm, excessive bronchial secretions, pulmonary edema, or aspiration of gastric contents.

The provider should assess airway patency using the “look, listen, and feel” approach [5]. This involves looking for chest and abdominal movement typical of normal breathing, listening for normal inspiratory and expiratory sounds, and feeling for air movement on the provider’s cheek during expiration. Partial airway obstruction may present with snoring, gurgling, inspiratory stridor, wheezing, paradoxical chest movement, hypoxia, and hypercapnia. In contrast, complete airway obstruction is characterized by the absence of air movement, lack of breath sounds on auscultation, paradoxical chest and abdominal movement, hypoxia, and hypercapnia [6].

Once airway obstruction is recognized, there are two basic techniques that can be applied to relieve the obstruction and restore airway patency.

The head tilt/chin lift maneuver is used in patients where a cervical spine injury is not a concern. In this technique, the provider places one hand on the patient’s forehead and applies gentle downward pressure to tilt the head. Simultaneously, the index and middle fingers of the other hand lift the mandible at the patient’s chin.

Image 1 - head tilt - chin lift manoeuvre

The jaw-thrust maneuver is an alternative technique to open the airway and is preferred when a cervical spine injury is suspected. The first step involves locating the angle of the mandible. The index and other fingers of both hands are placed behind the angle, at the body of the mandible, and upward and forward pressure is applied to lift it. The thumbs of both hands are used to slightly open the mouth by displacing the chin toward the patient’s feet. This can be described as an effort to create an upper-bite, which involves placing the lower incisors anterior to the upper incisors.

Image 2 - jaw thrust manoeuvre 1
Image 3 - jaw thrust manoeuvre 2

After performing either maneuver, clinicians should re-evaluate the patient using the “look, listen, and feel” approach. Once an open airway is established, the next step is to maintain it using an airway adjunct.

Application Of Airway Adjuncts

Introduction
Oropharyngeal (OPA) and nasopharyngeal (NPA) airways are useful adjuncts for maintaining an open airway. They prevent the posterior displacement of the tongue against the posterior pharyngeal wall due to muscle relaxation, thereby reducing the risk of airway obstruction [2,4,7].

Indications
OPA and NPA are used to maintain a patent airway. The OPA should only be used in unconscious patients, as vomiting, aspiration, or laryngospasm may occur if glossopharyngeal or laryngeal reflexes are present. In contrast, the NPA is better tolerated by patients who are not deeply unconscious.

Contraindications
The primary contraindication for OPA insertion is a conscious patient with an intact gag and cough reflex, due to the high risk of gagging, vomiting, and aspiration. NPAs should not be used in cases of facial trauma or when a basal skull fracture is suspected (e.g., raccoon eyes or battle sign). Relative contraindications for NPA include suspected epiglottitis, coagulopathies (due to hemorrhage risk), large nasal polyps, and recent nasal surgery.

Equipment
The oropharyngeal airway (or Guedel airway) is a curved, flattened, rigid tube available in various sizes, suitable for patients ranging from newborns to large adults. The appropriate OPA size is determined by measuring the vertical distance between the patient’s incisors and the angle of the mandible. Typical adult sizes are 3, 4, and 5.

Image 4 - Oropharyngeal Airways (OPAs)

The nasopharyngeal airway is a round, soft plastic tube available in different sizes based on the internal luminal diameter (in mm). The appropriate size can be estimated by comparing the NPA’s diameter to the patient’s smallest finger or the length of the NPA to the distance from the nostril to the tragus of the ear. Typical adult sizes are 6 mm, 7 mm, 8 mm, and 9 mm.

Image 5 - Nasopharyngeal Airways (NPAs)

Procedure Steps

  1. Insertion of an Oropharyngeal Airway:

    • Open the patient’s mouth and ensure no foreign materials could be pushed into the larynx during insertion.
    • There are two methods for OPA insertion. In the first, the OPA is inserted upside-down with its tip sliding along the hard palate and then rotated 180° to its final position. This method is typically used for adults.
    • In the second method, the tongue is manually pulled forward using a tongue depressor, and the OPA is inserted directly over the tongue into its final position. This method is preferred for children.
  2. Insertion of a Nasopharyngeal Airway:

    • Choose the larger nostril (typically the right) for insertion. Topical anesthetic spray may be applied.
    • Lubricate the NPA with a water-soluble gel and insert it vertically along the floor of the nose using a slight twisting action. The curve of the airway should be directed towards the patient’s feet.
    • If resistance is encountered, never force the NPA. Instead, remove it and attempt insertion through the other nostril.

Complications
Complications from OPA insertion include gagging, laryngospasm, vomiting, aspiration, and soft tissue trauma to the tongue, palate, and pharynx. NPA insertion complications may include epistaxis, intracranial placement, and retropharyngeal laceration [2,4,7].

Bag-Valve Mask Ventilation

Introduction
Bag-valve mask ventilation (BMV) is an essential skill for every emergency provider. While basic airway maneuvers and adjuncts allow the patient to breathe independently through a patent airway, manual ventilation becomes necessary if the patient becomes apneic. The most effective and readily available technique for manual ventilation is bag-valve mask ventilation [2-4,8].

Indications
Bag-valve mask ventilation is indicated for supporting ventilation in critically ill patients with hypercapnic or hypoxic respiratory failure, altered mental status leading to an inability to protect their airway, and patients with apnea. Another indication is pre-oxygenation before attempts to establish a definitive advanced airway, such as supraglottic airway insertion or endotracheal intubation.

Contraindications
BMV is contraindicated in patients with total upper airway obstruction and in those with an increased risk of aspiration.

Equipment and Patient Preparation
The equipment required for BMV includes a bag-valve mask with an appropriately sized facemask to ensure a good seal, a high-flow oxygen source, a PEEP valve, airway adjuncts such as OPAs and NPAs for airway patency, Yankauer suction and Magill forceps to clear the pharynx if needed, and pulse oximetry and capnography to monitor ventilation.

The bag-valve mask consists of:

  • A self-inflating resuscitation device (a plastic bag that re-expands after being squeezed), available in sizes such as 250 ml, 500 ml, and 1500 ml for infants, children, and adults, respectively.
  • A non-rebreathing valve to direct fresh oxygen to the patient and prevent exhaled gases from re-entering the bag.
  • A PEEP valve (optional) attached to the exhalation port.
  • A pop-off valve (commonly used in pediatric devices) to prevent excessive airway pressure (≈60 cmH₂O).
  • An oxygen inlet and air intake valve.
  • An oxygen reservoir bag with one-way valves.

Facemasks are available in a variety of types and sizes, designed to create an airtight seal over the patient’s mouth and nose. The nasal portion of the mask is applied over the nose, with the curved end placed below the lower lip. Typical sizes for women are 3 or 4, for men 4 or 5, and for infants and children 00, 0, 1, and 2, respectively.

Image 6 - bag mask with explanation

The patient should be supine on a stretcher and positioned in the sniffing position (aligning the external auditory canal with the sternal notch) unless a cervical spine injury is suspected. The provider is positioned at the head of the patient. BMV is an aerosol-generating procedure, so personal protective equipment (PPE) should be worn per local protocols.

Procedure Steps [2-4,8]

  1. One-Person Technique
    In the one-person technique, the provider uses the “E-C seal.” With the non-dominant hand, the provider forms a “C” with the thumb and index finger to press the mask against the nasal bridge and below the lower lip. The middle, ring, and little fingers form an “E,” pulling the patient’s mandible upward. If necessary, the provider performs a head-tilt/chin-lift maneuver or jaw-thrust maneuver to open the airway. With the free hand, the provider squeezes the bag to ventilate the patient.

  2. Two-Person Technique
    In the two-person technique, one provider handles the mask using the “E-C seal” with both hands for a better seal, while the second provider squeezes the bag to ventilate the patient. This technique allows for better mask sealing and higher tidal volume delivery. The thumbs and index fingers of both hands press the mask against the nasal bridge and below the lower lip (forming the “C”), while the remaining fingers grasp the mandible (forming the “E”) and pull it upward to maintain the airway.

  3. Ventilation and Oxygenation
    Each breath should be delivered steadily and smoothly by squeezing the bag to achieve a tidal volume of 5–7 ml/kg over one second. The bag is then released to allow re-inflation. Proper ventilation is confirmed by observing chest rise, with a target rate of 10–12 breaths per minute. Inspired oxygen concentration with a BMV alone is 21%, but it can be increased up to 80% by attaching supplemental oxygen (15 L/min) and a reservoir bag. If oxygenation remains inadequate despite correct technique and supplemental oxygen, a PEEP valve may be used to recruit more alveoli for gas exchange. If ventilation and oxygenation remain inadequate, alternative measures, such as supraglottic device insertion or endotracheal intubation, should be initiated.

Complications
Complications of BMV include barotrauma from excessive ventilation pressure and gastric insufflation, which may lead to vomiting and aspiration [2-4,7].

Supraglottic Airway Devices (SGA)

Introduction
Supraglottic airway devices (SGAs) are inserted blindly into the patient’s oropharynx, positioned above the glottis, allowing for ventilation and oxygenation over a short period. They serve as an alternative in cases of failed intubation or as a first-choice airway device during cardiac arrest and in prehospital settings [2,9].

Indications
The primary indications for SGA insertion include:

  • Acting as a rescue device in cases of difficult or failed intubation attempts.
  • Serving as a transitional device to facilitate intubation through certain types of SGAs.
  • Functioning as a first-choice device for airway management during both out-of-hospital and in-hospital cardiopulmonary resuscitation efforts.

Contraindications
SGA insertion is contraindicated in the following cases:

  • Inability to adequately open the patient’s mouth.
  • Total airway obstruction.
  • Increased risk of aspiration of gastric contents.
  • Requirement for high inspiratory pressures.

Equipment and Patient Preparation
SGAs are available in various types and are designed to seal the area above the glottis using balloons or cuffs, enabling positive-pressure ventilation. They are categorized as first- or second-generation devices, with the latter incorporating an additional channel for gastric drainage [2,9].

  • Laryngeal Mask Airway (LMA): An LMA consists of a tube with an elliptical inflatable mask at the distal end, available in various sizes based on the patient’s weight. Common models include:

    • Classic™ LMA: A reusable or disposable first-generation LMA.
    • Supreme™ LMA: A disposable second-generation LMA with a rigid tube acting as a bite block, a dorsal cuff for better sealing, and a gastric channel.
    • Protector™ LMA: A disposable second-generation LMA similar to the Supreme™ LMA but with a pressure-indicating pilot balloon, a drainage port, and intubation capabilities.
    • Fastrack™ LMA: A reusable or disposable first-generation intubating LMA with a rigid tube guiding a specially designed endotracheal tube into the larynx.
Image 7 - classic laryngeal mask airway (LMA)
Image 8 - protector LMA
  • i-gel®: A second-generation SGA featuring a gel-like, non-inflatable distal end made of thermoplastic elastomer, a bite block, and a gastric channel. Sizes are determined by the patient’s weight.
Image 9 - igel
  • Laryngeal Tube (Retroglottic Airway Device): This device consists of a tube with two inflatable balloons—one proximal to seal the oropharynx and one distal to seal the esophagus. Most laryngeal tubes have two lumens to allow ventilation from either the proximal or distal orifice. Sizes are based on patient height or weight.

Procedure Steps

  1. Preparation: Select the appropriate SGA size based on the patient’s physical characteristics. Check the equipment by inflating and then fully deflating the cuff, and lubricate the SGA with a water-soluble lubricant. Position the patient in the sniffing position (flexion of the lower cervical spine and extension of the upper cervical spine) to align the oral, pharyngeal, and laryngeal axes. Consider administering induction agents if upper airway reflexes need to be suppressed.
  2. Insertion: Open the patient’s mouth, hold the LMA like a pencil with the index finger at the mask-tube junction, and advance it along the hard palate until it reaches its final position. Inflate the cuff as indicated on the device packaging. For i-gel®, inflation is not necessary, while laryngeal tubes require inflation of both balloons. Secure the device once in place.

Complications
While ventilation success rates with SGAs are high, complications may occur, including [2,9]:

  • Aspiration of gastric contents.
  • Inability to ventilate due to inappropriate size or misplaced device.
  • Laryngospasm if upper airway reflexes are intact.
  • Local edema from excessive pressure on adjacent structures.

Hints and Pitfalls

  • In a fully deflated LMA, the mask tip may flip or roll, leading to non-optimal placement. Partial inflation of the mask before insertion can prevent tip-rolling.
  • Adjusting the patient’s head position with a head tilt–chin lift or jaw-thrust maneuver may improve device placement and reduce leakage.

Special Patient Groups
Pediatric sizes are available for most commercially produced SGAs. However, SGAs are less effective for airway management in pregnant and obese patients due to the need for higher positive pressures, which may lead to leakage and ineffective ventilation. Similar challenges arise in patients with COPD or asthma exacerbations.

Endotracheal Intubation

Introduction
Endotracheal intubation involves placing an airtight-sealed tube into the patient’s trachea to ensure airway patency for ventilation and to protect against aspiration. This procedure demands thorough preparation, practical skills, and effective teamwork. Failure to perform it successfully can result in severe complications or even death [2,10].

Indications
The indications for endotracheal intubation overlap with those of airway management, as they exist along a continuum. They can be categorized into three main groups:

  1. Inability to maintain a patent airway and risk of aspiration (e.g., acutely decreased mental status or impending airway obstruction).
  2. Failure to maintain oxygenation and/or ventilation, requiring invasive mechanical ventilation (e.g., severe exacerbations of asthma or COPD).
  3. Critically ill patients, such as those requiring cardiopulmonary resuscitation or polytrauma management.

Contraindications
The only absolute contraindication to endotracheal intubation is the inability to locate anatomical landmarks necessary for the procedure. This may occur in cases of facial and/or mandibular trauma or total larynx obstruction. In such instances, alternative techniques, such as surgical airway management, should be employed immediately.

Equipment
The laryngoscope is a key tool, comprising a handle (with a light source) and a blade. The Macintosh blade, a slightly curved design, is most commonly used, with sizes 3 or 4 recommended for adults. Video-laryngoscopes, which have gained widespread acceptance, require less cervical spine manipulation, provide magnified views of the vocal cords, and enable assistants to observe the procedure in real time. Video-laryngoscopes come with different blade types (e.g., Macintosh or hyper-angulated blades).

Image 10 - laryngoscope with Macintosh blade
Image 11 - videolaryngoscope 1
Image 12 - videolaryngoscope 2

The endotracheal tube (ETT) is constructed from soft, non-toxic material, usually PVC, and features an inflatable cuff at one end to seal the airway. The size of the tube is determined by its internal diameter (e.g., 8.0–8.5 mm for adult males and 7.0–7.5 mm for adult females).

Image 13 - Endotracheal Tube (ETT)

Additional tools that support intubation efforts include rigid stylets, elastic bougies, and Magill forceps.

Procedure Steps

Airway management in emergency settings typically follows the principles of Rapid Sequence Induction (RSI), which involves administering an induction agent and a neuromuscular blocking agent to facilitate ETT placement without bag-mask ventilation, minimizing aspiration risk. Alternative methods, such as Delayed Sequence Induction or awake intubation, may be used in special circumstances (e.g., anatomical or physiological difficulties) [11].

RSI follows a seven-step process known as the “7 Ps”:

(1) Preparation

  • Proper preparation is key to a successful, uneventful procedure. Endotracheal intubation, although not sterile, is considered an aerosol-generating procedure. Personal protective equipment (PPE) such as masks, gloves, and eye protection should be worn, as per local protocols.
  • Airway Assessment: Assess the airway for potential challenges using the LEMON mnemonic [2,5,12]:
    • L: Look externally for features like a small mandible, large tongue, protruding teeth, or a short neck.
    • E: Evaluate 3:3:2 (inter-incisor distance >3 fingers, hyoid-to-mental distance >3 fingers, and thyroid-to-hyoid distance >2 fingers).
    • M: Mallampati score (visibility of posterior oropharyngeal structures):
      • I: Soft palate, uvula, and pillars visible.
      • II: Soft palate and uvula visible.
      • III: Soft palate and base of the uvula visible.
      • IV: Only the hard palate visible.
    • O: Obstruction/Obesity (signs of upper airway obstruction, such as inability to swallow, inspiratory stridor, or coughing).
    • N: Neck mobility (e.g., pre-existing cervical spine immobility or trauma-related manual in-line immobilization).
    • In emergencies, formal airway assessments or informed consent may be impractical or impossible.

iEM-infographic-pearls-airway - Assessing Airway Difficulty
  • Back-Up Plan: Prepare alternative devices for oxygenation and ventilation in case of intubation failure, and communicate the plan with the team. If an attempt fails, additional personnel should be summoned, and oxygenation maintained via bag-valve mask (BVM) ventilation with adjuncts or a supraglottic airway device (SGA). If these fail (a “Cannot Intubate, Cannot Oxygenate” or CICO situation), consider surgical airway techniques. Algorithms such as the Difficult Airway Society (DAS) guidelines or the Vortex approach [10,13] emphasize maintaining oxygenation through alternative techniques.

  • Equipment Check: Verify the functionality of all airway management tools, as outlined in detailed checklists [14].

Monitoring (ECG, BP, SpO2, EtCO2)

Laryngoscope (DL or VL)

Vascular access

ET tube (various sizes)

Oxygen source

Syringe (ET cuff inflation)

Suction device (Yankauer)

Stylets (various sizes)

Bag-mask ventilation device

Gum elastic Bougie

Oropharyngeal and Nasopharyngeal airways (various sizes)

ETT stabilization device

Medications (drawn up and labeled)

Rescue devices (supraglottic devices, surgical airway kit)

(2) Pre-Oxygenation

The administration of a neuromuscular blocking agent leads to the cessation of automatic breathing within seconds. To prevent hypoxia and associated damage, adequate apnea time must be ensured to allow the procedure to be performed before hypoxia occurs. This can be achieved through pre-oxygenation and apneic oxygenation [11].

Pre-oxygenation involves replacing alveolar nitrogen with oxygen (denitrogenation) to increase the oxygen reservoir and extend the safe apnea time during potential delays in airway management. Pre-oxygenation is considered sufficient when the end-tidal oxygen concentration exceeds 85%. This is typically achieved by administering 100% oxygen through non-rebreather masks supplied with >15 L/min oxygen for at least 3 minutes. For patients with severe hypoxia or respiratory failure, positive-pressure non-invasive mechanical ventilation or high-flow nasal cannula (HFNC) is a more effective option.

Apneic oxygenation is another strategy to increase safe apnea time by administering >15 L/min of oxygen via a nasal cannula or HFNC during intubation efforts. This method achieves an oxygen pressure gradient even during apnea.

Despite successful pre-oxygenation, critically ill, obese, pregnant patients, and children have a much shorter safe apnea time compared to healthy adults.

(3) Pre-Intubation Optimization (First Resuscitate – Then Intubate)

While anatomical difficulty may be present in a few patients, most emergency intubations involve patients with physiological challenges [12,15]. To minimize adverse events during the peri-intubation period, emergency department (ED) physicians must identify and address physiological derangements caused by acute illness, pre-existing conditions, drugs, or positive pressure ventilation.

Key considerations for optimization include:

  • Hypoxemia: Consider pre-oxygenation, positive pressure ventilation, apneic oxygenation, or chest-tube insertion in cases of pneumothorax.
  • Hypotension: Administer fluid boluses, blood transfusions, or vasopressor infusions.
  • Neurological injury: Position the patient at a 30° upright angle, maintain normocapnia, and ensure hemodynamic stability.

(4) Paralysis with induction

Pre-treatment agents can be utilized to mitigate the sympathetic response triggered by laryngoscopy. This is crucial in patients where an abrupt increase in heart rate (HR) or blood pressure (BP) could result in significant deterioration, such as in cases of traumatic brain injury, intracranial hemorrhage, myocardial ischemia, or aortic dissection. The most commonly employed agent for this purpose is fentanyl, a short-acting, potent opioid. Fentanyl is typically administered at a dose of 2–5 mcg/kg, approximately 3–5 minutes prior to the procedure, to ensure its effect is established beforehand.

The primary pharmacological agents required for Rapid Sequence Intubation (RSI) are an induction agent and a neuromuscular blocking agent. Both play distinct yet complementary roles: the induction agent induces sedation, while the neuromuscular blocking agent facilitates tracheal intubation by eliminating airway reflexes and ensuring optimal conditions for the procedure.

There is no single agent of choice. The most commonly used induction agents for Rapid Sequence Intubation (RSI) are as follows [11]:

  • Ketamine: As an NMDA receptor antagonist, ketamine provides analgesia, sedation, and amnesia while preserving the respiratory drive. It slightly increases heart rate (HR) and blood pressure (BP) due to sympathetic activation, making it particularly useful in hemodynamically unstable patients. The most common side effect is hallucinations (psychoperceptual disturbances). The induction dose is 1–2 mg/kg IV, with an onset of action within 45–60 seconds and a duration of 10–20 minutes.

  • Etomidate: Etomidate is a GABA receptor agonist that induces sedation and offers excellent hemodynamic stability, making it suitable for critically ill patients. It may cause transient myoclonic movements during induction. Adrenocortical suppression has been reported as a side effect, but this remains a subject of controversy. The induction dose is 0.2–0.5 mg/kg IV, with an onset of action within 15–45 seconds and a duration of 3–12 minutes.

  • Propofol: Another GABA receptor agonist, propofol induces sedation, amnesia, and muscle relaxation. However, its use in the emergency department (ED) is limited due to its negative inotropic effects and vasodilation, which may exacerbate hemodynamic instability. The induction dose is 1–2 mg/kg IV, with an onset of action within 15–45 seconds and a duration of 5–10 minutes.

  • Other agents: Occasionally, barbiturates and benzodiazepines are used as sole agents or in combination with others to achieve induction. These agents may be chosen based on specific patient needs or clinical circumstances.

Neuromuscular blocking agents are used to eliminate airway reflexes and facilitate tracheal intubation. Rapid Sequence Intubation (RSI) requires rapid-acting agents, and the most commonly used agents are as follows:

  • Rocuronium: Rocuronium is a non-depolarizing neuromuscular blocking agent with a rapid onset and intermediate duration of action. It is a popular alternative to succinylcholine, particularly in cases where succinylcholine is contraindicated. Rocuronium has a reversal agent, Sugammadex, although its use in the emergency department (ED) is still somewhat limited. The induction dose is 1–1.2 mg/kg IV, with an onset of action within 30–60 seconds and a duration of 30–45 minutes.

  • Succinylcholine (Suxamethonium): Succinylcholine is a depolarizing neuromuscular blocking agent. Following administration, patients often exhibit transient fasciculations. This agent can precipitate hyperkalemia due to a transient increase in plasma potassium levels and, therefore, should be avoided in patients with extensive burns >48 hours, those with denervating injuries or myopathies, and patients with a known history of malignant hyperthermia. The induction dose is 1.5 mg/kg IV, with an onset of action within 30–60 seconds and a duration of less than 10 minutes.

(5) Positioning

Optimal positioning of the patient will improve upper airway patency and access, increase functional residual capacity, and reduce the risk of aspiration. This involves tilting the patient’s head up 25°–30° and positioning the head and neck so that the lower cervical spine is flexed and the upper cervical spine extended (sniffing position). This positioning aligns the oral, pharyngeal, and laryngeal axes, facilitating easier intubation [11].

In cases of trauma, manual-in-line stabilization (MILS) should be employed to protect the cervical spine from further damage during airway management procedures. Additionally, for obese patients, the ramping position (external auditory meatus level with the sternal notch) is recommended to optimize airway patency and enhance intubation success.

(6) Placement with Proof

Laryngoscopy is the procedure that allows direct (or indirect, in the case of video-laryngoscopy) visualization of the vocal cords to facilitate the insertion of the Endotracheal Tube (ETT) through them into the patient’s trachea [11].

  1. Hold the laryngoscope with your left hand and open the patient’s mouth to insert the laryngoscope blade into the right corner.
  2. Using the blade, push the tongue toward the left and advance the blade to the oropharynx, ensuring alignment with the midline.
  3. Visualize the epiglottis and lift it to reveal the vocal cords.
  4. Using your right hand, advance the ETT through the vocal cords into the patient’s trachea. Ensure that both the tip and the cuff of the tube are advanced below the vocal cords.
  5. Inflate the tube’s cuff to achieve an airtight seal of the airway.
  6. Confirm the ETT’s placement with the use of capnography.
  7. Auscultate to verify that the tube ventilates both lungs.
  8. Secure the ETT.

(7) Post-Intubation Management

Initiate ventilation either through a self-inflating bag or by connecting the patient to a ventilator. Maintain sedation through infusion or boluses. Perform a reassessment of the patient using the ABCDE approach [11].

Complications

  • Failed intubation requires prompt recognition and implementation of alternative methods of oxygenation and ventilation (rescue oxygenation through bag-mask ventilation, supraglottic airway devices, or surgical airway).
  • ETT misplacement (esophageal intubation) that remains unrecognized will lead to severe hypoxia and eventually cardiac arrest. Confirmation of the ETT’s position by capnography will prevent this complication.
  • Aspiration remains a possibility even with RSI. Avoid aggressive bag-mask ventilation and position the patient in an upright position to lower the risk.
  • Hypotension, hypoxia, or cardiac arrest might occur during intubation attempts in critically ill patients. Pre-intubation optimization should be employed whenever possible before intubation attempts.

Special Patient Groups

Pediatrics

Children have a relatively larger head and occiput, larger tongue, and small mandible, and a larynx that is more cephalad compared to adults [16]. Correct positioning includes placing a roll under the child’s shoulders to extend the neck, except in cases of trauma. Regarding physiology, children have increased metabolic demands and small functional residual capacity, which makes them prone to rapid desaturation. Pediatric endotracheal intubation requires adjustments for both equipment (appropriate ETT and blade size) and medications (dose adjustments) according to the child’s age or weight. Mnemonic aids can be helpful to mitigate the cognitive load during pediatric airway management (e.g., Broselow tape) [17].

Pregnant Patients

Pregnancy is characterized by decreased functional residual capacity, decreased gastric emptying, and airway edema. Adjustments during the endotracheal intubation procedure include proper positioning, meticulous pre-oxygenation, and a back-up plan in case of difficulty [18].

Obese Patients

Obesity severely decreases functional residual capacity, leading to rapid desaturation during airway management. Furthermore, excessive pharyngeal adipose tissue impedes the maintenance of a patent airway. Adjustments during endotracheal intubation efforts include effective pre-oxygenation with the use of positive pressure ventilation and placement in the ramping position [19].

Trauma Patients

In case of suspected cervical spine injury, manual-in-line stabilization (MILS) should be employed. Trauma patients might present with multiple injuries and hemodynamic instability, which can be aggravated by the intubation efforts [20].

In-line stabilization

Geriatrics

Airway management in the elderly presents unique challenges due to age-related physiological changes, comorbidities, and increased risk of complications. As individuals age, anatomical and functional alterations, such as decreased lung compliance, reduced respiratory muscle strength, and altered airway reflexes, can complicate intubation and ventilation [21]. Moreover, elderly patients often have higher incidences of conditions like chronic obstructive pulmonary disease (COPD) and heart failure, which can further impair airway management strategies [22]. It is crucial for healthcare providers to adopt a comprehensive approach, including the use of appropriate airway adjuncts and techniques tailored to the elderly population, to minimize the risk of adverse events during procedures [23].

Authors

Picture of Eirini Trachanatzi

Eirini Trachanatzi

My name is Eirini Trachanatzi. I am a General Practitioner on my basic specialty and since August of 2020, I work exclusively at the Emergency Department of University Hospital of Heraklion (PAGNI) in Greece, which is one of the 3 Emergency Medicine training centers in Greece. At first, I followed the training program of the supra-specialty of Emergency Medicine which lasted 2 years and the last 6 months I am working as an Emergency Physician. My special interests are the resuscitation and trauma.

Picture of Anastasia Spartinou

Anastasia Spartinou

My name is Anastasia (Natasa) Spartinou. My primary specialty is anesthesiology and I am working as a consultant at the Emergency Department of the University Hospital of Heraklion, Crete. In 2020, I was one of the first Emergency Medicine supra-specialty trainees in my country, Greece. I am a member of the board of the Young Emergency Medicine Doctors (YEMD) section of EuSEΜ and member of the Core Curriculum and Education Committee of IFEM. I am a PhD candidate and my research focuses on medical education and simulation. My special interests are medical education, resuscitation and trauma.

Listen to the chapter

References

  1. Nemeth J, Maghraby N, Kazim S. Emergency airway management: the difficult airway. Emerg Med Clin North Am. 2012;30(2):401-420. doi:10.1016/j.emc.2011.12.005.
  2. Brown CA III, Sakles JC, Mick NW, eds. The Walls Manual of Emergency Airway Management. 5th ed. Philadelphia, PA: Wolters Kluwer; 2018.
  3. Higginson R, Parry A. Emergency airway management: common ventilation techniques. Br J Nurs. 2013;22(7):366-371. doi:10.12968/bjon.2013.22.7.366.
  4. Brady MF, Burns B. Airway obstruction. In: StatPearls [Internet]. Treasure Island, FL: StatPearls Publishing; 2024 Jan–. Updated August 7, 2023. Accessed December 25, 2024. https://www.ncbi.nlm.nih.gov/books/NBK470562/
  5. Finucane BT, Tsui BC, Santora AH. Evaluation of the airway. In: Principles of Airway Management. 4th ed. New York, NY: Springer; 2010:27-58. doi:10.1007/978-0-387-09558-5_2.
  6. McPherson K, Stephens RC. Managing airway obstruction. Br J Hosp Med (Lond). 2012;73(10):C156-C160. doi:10.12968/hmed.2012.73.sup10.c156.
  7. Effective use of oropharyngeal and nasopharyngeal airways. ACLS.com. Published January 2019. Accessed December 25, 2024. https://acls.com/articles/nasopharyngeal-oropharyngeal-airways/
  8. Bosson N. Bag-valve-mask ventilation. Medscape. Updated January 29, 2024. Accessed December 25, 2024. https://emedicine.medscape.com/article/80184-overview
  9. Park HP. Supraglottic airway devices: more good than bad. Korean J Anesthesiol. 2019;72(6):525-526. doi:10.4097/kja.19417.
  10. Higgs A, McGrath BA, Goddard C, et al. Guidelines for the management of tracheal intubation in critically ill adults. Br J Anaesth. 2018;120(2):323-352. doi:10.1016/j.bja.2017.10.021.
  11. Schrader M, Urits I. Tracheal rapid sequence intubation. In: StatPearls [Internet]. Treasure Island, FL: StatPearls Publishing; 2024 Jan–. Updated October 10, 2022. Accessed December 25, 2024. https://www.ncbi.nlm.nih.gov/books/NBK560592/
  12. Kornas RL, Owyang CG, Sakles JC, Foley LJ, Mosier JM; Society for Airway Management’s Special Projects Committee. Evaluation and management of the physiologically difficult airway: consensus recommendations from Society for Airway Management. Anesth Analg. 2021;132(2):395-405. doi:10.1213/ANE.0000000000005233.
  13. Chrimes N. The Vortex: a universalhigh-acuity implementation toolfor emergency airway management. Br J Anaesth. 2016;117(suppl 1):i20-i27. doi:10.1093/bja/aew175.
  14. RSI setup checklist. Broome Docs – Rural Generalist Doctors Education. Accessed April 14, 2023. https://broomedocs.com/clinical-resources/rsi-setup-checklist/
  15. Myatra SN, Divatia JV, Brewster DJ. The physiologically difficult airway: an emerging concept. Curr Opin Anaesthesiol. 2022;35(2):115-121. doi:10.1097/ACO.0000000000001102.
  16. Wheeler DS, Spaeth JP, Mehta R, Hariprakash SP, Cox PN. Assessment and management of the pediatric airway. In: Pediatric Critical Care Medicine: Basic Science and Clinical Evidence. London, UK: Springer; 2009:1-30. doi:10.1007/978-1-84800-919-6_4.
  17. Abdallah C. Pediatric endotracheal intubation. Middle East J Anesthesiol. 2015;23(1):123-124.
  18. Lewin SB, Cheek TG, Deutschman CS. Airway management in the obstetric patient. Crit Care Clin. 2000;16(3):505-513. doi:10.1016/s0749-0704(05)70127-5.
  19. Wadhwa A, Singh PM, Sinha AC. Airway management in patients with morbid obesity. Int Anesthesiol Clin. 2013;51(3):26-40. doi:10.1097/AIA.0b013e318298140f.
  20. Manoach S, Paladino L. Manual in-line stabilization for acute airway management of suspected cervical spine injury: historical review and current questions. Ann Emerg Med. 2007;50(3):236-245. doi:10.1016/j.annemergmed.2007.01.009.
  21. Petersen A, Wong E, Brown T. Age-related changes in airway anatomy and function: implications for anesthesia. Anesthesiol Clin. 2018;36(1):1-12.
  22. Hernandez A, Lee C, Patel K. Challenges in airway management in older adults. Anesth Analg. 2021;132(3):710-717.
  23. Baker M, Smith J, Johnson R. Airway management in the elderly: a review. J Geriatr Med. 2020;45(2):123-130.

Reviewed and Edited By

Picture of Arif Alper Cevik, MD, FEMAT, FIFEM

Arif Alper Cevik, MD, FEMAT, FIFEM

Prof Cevik is an Emergency Medicine academician at United Arab Emirates University, interested in international emergency medicine, emergency medicine education, medical education, point of care ultrasound and trauma. He is the founder and director of the International Emergency Medicine Education Project – iem-student.org, chair of the International Federation for Emergency Medicine (IFEM) core curriculum and education committee and board member of the Asian Society for Emergency Medicine and Emirati Board of Emergency Medicine.

Burns and Smoke Inhalation (2024)

by Michaela Banks, Anthony Dikhtyar, Jacquelyne Anyaso, & Ashley Pickering

You have a new patient!

A 26-year-old male presents to the emergency department with burns on his face, arms, hands, and torso. He states that he was burning trash in his front yard without his shirt on when a big explosion occurred. He appears distressed and short of breath. The presence of singed nasal hairs is also noted. Examination reveals multiple partial- and full-thickness burns with blisters and surrounding redness. His vitals are as follows: HR: 130  BP (taken on R calf): 130/80 RR: 30 SpO2: 75%. His weight: 75kg

a-photo-of-a-26-year-old-male-with-burns (the image was produced by using ideogram 2.0)

What do you need to know?

Importance

Burn injuries result from various sources and can range in severity. The mortality rate from thermal burns is directly related to the size of the burn [1]. Large and deep burns can trigger systemic responses, such as shock, which can lead to death. Three key risk factors that increase the likelihood of complications include: age over 60 years, inhalation injuries, and non-superficial burns (partial and full-thickness) covering more than 40% of the total body surface area (TBSA) [2].

Epidemiology

Burn injuries are a significant public health issue, with approximately 450,000 individuals seeking medical attention annually, and about 45,000 requiring hospitalization [2]. Residential fires are the leading cause of burn-related deaths, contributing to nearly 3,500 fatalities per year. Smoking materials, such as cigarettes, are the primary cause of fire-related deaths, while other fatal injuries stem from motor vehicle crashes, electrical contact, or exposure to chemicals. Men constitute 71% of burn patients, with children under five representing 17% [2]. Most burns occur at home (65%) and involve less than 10% total body surface area (67%). Advances in burn care have improved survival rates to 96% [2]. Roughly 86% of all burns are caused by thermal injury. Flame and scald burns are the leading causes of burns in children and adults. Inhalation injury is present in two-thirds of patients with burns greater than 70% of TBSA. 

Pathophysiology

Burn injuries, caused by heat, chemicals, electricity, or radiation, trigger a complex interplay of local and systemic responses. At the cellular level, burn wounds are divided into three distinct zones: coagulation, stasis, and hyperemia. The central zone of coagulation undergoes irreversible cell death due to protein denaturation, necessitating surgical intervention in many cases. Surrounding it, the zone of stasis contains viable but at-risk cells that can either recover with proper care or progress to necrosis. The outer zone of hyperemia typically recovers fully within days due to its inflammatory response and intact blood flow [2-4].

Burns prompt a robust inflammatory response, increasing capillary permeability and causing fluid shifts that lead to edema. Local edema compromises blood flow and cell survival in the zone of stasis, while systemic edema in large burns contributes to hypovolemia, the primary cause of burn shock. Immediate and adequate fluid resuscitation is critical to prevent worsening injury and maintain organ perfusion [2-4].

Specific burn types exhibit unique pathophysiologies. Inhalation injuries from superheated gases or toxic smoke cause airway edema, inflammation, and potentially fatal complications like carbon monoxide poisoning and ARDS [5]. Chemical burns differ by agent, with acids causing coagulation necrosis and alkalis leading to deeper liquefaction necrosis. Electrical burns often involve extensive internal damage along the current’s path, risking cardiac arrhythmias and systemic effects. Radiation burns, though rarer, involve cellular damage through ionizing radiation exposure [2-4].

Systemically, extensive burns induce a hypermetabolic state, immune suppression, and systemic inflammatory responses affecting multiple organs. Cardiovascular effects, such as burn shock, respiratory compromise, and heightened infection risks, are key complications. Patient outcomes hinge on factors like burn depth, TBSA, age, inhalation injury presence, and quality of initial management, underscoring the importance of specialized burn center care.

Burn Depth

Burn depth classification is fundamental to assessing burn injuries, guiding treatment decisions, and predicting outcomes. Accurate determination of burn depth, particularly for partial-thickness burns, remains challenging, even for skilled clinicians. This underscores the need for continued research and advanced technologies to enhance diagnostic precision.

Traditionally, burns are categorized into four classes based on the extent of tissue damage [4]:

  1. Superficial Thickness (First-Degree) Burns: These affect only the epidermis, presenting with redness, pain, and warmth without blistering. Healing occurs within a few days without scarring.
  2. Partial-Thickness (Second-Degree) Burns: These penetrate the dermis and are subdivided into:

    1. Superficial Partial-Thickness Burns: Involving the upper dermis, they are painful, moist, and blistered, typically healing within 2–3 weeks with minimal scarring.

    2. Deep Partial-Thickness Burns: Reaching deeper dermal layers, these burns cause damage to sweat glands and hair follicles. They are less painful due to nerve damage, appear mottled and dry, and may require 3–8 weeks or longer to heal, often resulting in scarring or contractures.

  3. Full-Thickness (Third-Degree) Burns: These burns destroy the entire epidermis and dermis, extending into subcutaneous tissue. They appear white, brown, or charred with a leathery texture and are insensate due to nerve destruction. Healing requires surgical intervention, such as skin grafting, and leaves significant scars.

  4. Fourth-Degree Burns: Extending into muscles, bones, tendons, or ligaments, these burns are characterized by blackened tissue and often result in loss of the affected part.

These classifications provide a framework for clinicians to tailor interventions and anticipate patient needs, particularly in severe or complex burn cases. The illustration below displays the various categories of burn depth [4].

From: [4] Jeschke MG, van Baar ME, Choudhry MA, Chung KK, Gibran NS, Logsetty S. Burn injury. Nat Rev Dis Primers. 2020;6(1):11. Published 2020 Feb 13. doi:10.1038/s41572-020-0145-5

Medical History

Accurately gathering a burn history is critical for evaluating the injury’s severity, identifying risks, and tailoring management. The AMPLET mnemonic is widely recommended for systematic collection of essential information regarding the event and the patient’s medical background. Additionally, specific questions based on the type of burn provide crucial details for precise assessment and treatment [2-4, 6].

Allergies (A):
Identifying drug and environmental allergies is essential to avoid adverse reactions during treatment.

Medications (M):
A detailed list of current medications, including prescription drugs, over-the-counter remedies, herbal supplements, and home treatments, is vital to anticipate potential drug interactions or complications.

Past Medical History (P):
Knowledge of pre-existing conditions, such as diabetes, cardiovascular disease, lung disorders, or bleeding tendencies, helps predict how the patient may respond to burn injuries and resuscitation. Tetanus immunization status should also be reviewed and updated if necessary (see “T”).

Last Meal or Drink (L):
Documenting the patient’s last meal or drink is crucial for surgical planning, as recent food intake may require delays in procedures involving anesthesia.

Events/Environment Relating to Incident (E):
A detailed account of the burn incident helps identify the mechanism of injury, the risk of inhalation injury, and associated trauma. Important elements to document include:

  • Type of burn: Thermal, chemical, electrical, or radiation.
  • Cause of burn: Flame, scald, contact with hot objects, chemicals, or electricity.
  • Incident location: Indoor/outdoor, enclosed space, smoke presence.
  • Duration of exposure: Time spent in contact with the burn source.
    First aid administered: Cooling, cleaning, or dressing of the burn before medical evaluation.
  • Suspicion of abuse or neglect: Look for inconsistencies in the history, patterns of injury, or delays in seeking care. Specific questions include:
    • How did the burn occur?
    • Who was present?
    • How long to extinguish flames?
    • Was the area cooled? With what and for how long?
    • Were explosions, blasts, or chemical spills involved?
    • Was the patient trapped or unconscious?

Tetanus and Childhood Immunizations (T):
Ensuring tetanus immunization is current (within five years) is crucial. In children, assessing overall immunization status helps anticipate potential complications.

Specific Questions Based on Burn Type [2-4,6]

Thermal Burns:

  • How did the burn occur?
  • What was the heat source (e.g., flame, scald, or hot object)?
  • Was clothing involved, and how quickly was it removed?
  • Was a flammable liquid (e.g., gasoline) involved?

Chemical Burns:

  • What was the chemical agent?
  • How did exposure occur, and how long was contact?
  • What decontamination measures were taken?
  • Is a Material Safety Data Sheet (MSDS) available?

Electrical Burns:

  • What type of electricity was involved (high voltage/low voltage, AC/DC)?
  • What was the duration of contact?
  • Was the patient thrown or did they fall?

Physical Examination

The physical examination of a burn patient is a systematic process designed to assess the severity of the burn injury, identify associated injuries or complications, and guide treatment decisions. A comprehensive and thorough examination is critical for determining the need for transfer to a burn center and predicting potential outcomes [2-6]. Make sure to assess for concomitant trauma (especially after a blast injury or fall).

First, perform decontamination if the person has been exposed to a chemical substance. If possible, expose the patient to a warm room. Immediately assess the airway, breathing, and circulation (ABCs), see details below.

Primary Survey

The primary survey prioritizes life-threatening conditions using the ABCDE approach [2,4,6]:

A. Airway

  • Assess for patency: Check for obstruction, swelling, or soot in the mouth and nose. Examine for posterior oropharynx edema and singed facial and nasal hairs carefully.
  • Listen for abnormal breath sounds: Stridor, wheezing, or decreased breath sounds may indicate inhalation injury or airway compromise.
  • Consider early intubation: Severe facial burns, inhalation injury, or altered mental status may necessitate securing the airway. Please do not delay airway procedure if you suspect inhalation injury.

B. Breathing

  • Assess respiratory rate and effort: Look for tachypnea, labored breathing, or cyanosis.
  • Auscultate lung sounds: Wheezing, rales, or rhonchi may suggest inhalation injury or pulmonary complications.
  • Administer high-flow oxygen: Use 100% oxygen via a non-rebreather mask, particularly for moderate to severe burns patients or patients with suspected inhalation injury.

C. Circulation

  • Monitor heart rate and rhythm: Look for tachycardia, bradycardia, or arrhythmias.
  • Measure blood pressure: Hypotension may indicate shock or blood loss.
  • Assess capillary refill and skin color: Delayed refill, pallor, or cyanosis indicates poor perfusion.
  • Establish IV access: Insert two large-bore IVs for moderate to severe burn patients, particularly for burns covering >20% TBSA.
  • Control bleeding: Bleeding suggests additional injuries.

D. Disability

  • Assess level of consciousness: Use the AVPU scale (Alert, Verbal, Pain, Unresponsive) or Glasgow Coma Scale (GCS).
  • Evaluate neurological status: Check pupils, motor strength, and sensation.

E. Exposure and Environmental Control

  • Remove clothing and jewelry: Fully expose the patient to assess burns but prevent further constriction.
  • Identify deformities: Look for fractures or dislocations.
  • Maintain warmth: Use clean, dry sheets and blankets to prevent hypothermia.

Secondary Survey

Once the primary survey stabilizes life-threatening conditions, conduct a detailed evaluation [2,4]:

A. History
Obtain a complete history using the AMPLET mnemonic, covering allergies, medications, past medical history, last meal, events surrounding the burn, and tetanus immunization status (see Medical History above).

B. Head-to-Toe Examination

  • Head and Neck: Assess for burns, singed hair, soot, inhalation injury, corneal damage, and tympanic membrane injury.
  • Chest: Listen to breath sounds, observe chest expansion, and evaluate for circumferential burns that may impair breathing.
  • Abdomen: Inspect for burns, palpate for tenderness, and consider the risk of abdominal compartment syndrome with circumferential burns.
  • Extremities: Look for burns, fractures, diminished pulses, or signs of compartment syndrome. Assess sensation and motor function.
  • Genitalia and Perineum: Inspect for burns and swelling, and assess urinary retention.
  • Back and Buttocks: Examine these areas during log rolling, ensuring full exposure and injury identification.

C. Burn Wound Assessment

  • Burn size: Estimate TBSA using the Rule of Nines (see images below) [7] or the Lund and Browder chart.
  • Burn depth: Classify burns as superficial, partial-thickness (superficial or deep), full-thickness, or fourth-degree. Note that burn depth may evolve over time (see figure about burn depth above).
  • Document wound characteristics: Describe color, texture, moisture, blisters, and eschar.
Rule of Nines (Adults ≥ 14 years of age) - Courtesy of the American Burn Association - From: [7] - https://www.health.state.mn.us/communities/ep/surge/burn/tbsa.pdf
Rule of Nines for Children (Age 1 - 14) - Courtesy of the American Burn Association - From: [7] - https://www.health.state.mn.us/communities/ep/surge/burn/tbsa.pdf
Rule of Nine for Infant (Age < 1 year) - Courtesy of the American Burn Association - From: [7] - https://www.health.state.mn.us/communities/ep/surge/burn/tbsa.pdf

Burns are classified into degrees based on the depth of tissue damage, with each classification displaying distinct pathophysiological features, clinical findings. The following section covers specific clinical information related to burn depth.

Superficial (First-Degree) Burns
Superficial burns involve only the epidermis, the outermost layer of the skin. These burns are characterized by warm, dry, and red areas that blanch with pressure. Blistering is absent, and the skin typically heals within a few days without scarring. Sunburn is a classic example of a superficial burn.

Partial-Thickness (Second-Degree) Burns
Partial-thickness burns extend beyond the epidermis into the dermis and are further divided into superficial and deep categories.

  • Superficial Partial-Thickness Burns: These burns affect the upper dermis and are very painful. Surrounding erythema, moisture, and blistering are common features. These burns blanch when pressed and typically heal with minimal scarring in 2–3 weeks.
  • Deep Partial-Thickness Burns: These penetrate deeper into the dermis, potentially damaging sweat glands and hair follicles. They are less painful due to nerve ending destruction and appear drier, with a mottled red or white surface that does not blanch. Healing takes longer and often results in scarring or contractures. Scalds and flash burns are typical causes of partial-thickness burns.

Full-Thickness and Beyond (Third- and Fourth-Degree) Burns
Full-thickness burns destroy the entire epidermis and dermis, often extending into subcutaneous fat and, in severe cases, deeper structures such as muscle and bone (fourth-degree burns). These burns result in decreased sensation due to nerve destruction. The affected areas appear white, brown, or leathery, with a dry texture, and they do not blanch when pressed. Examples include chemical burns, electrical burns, fully immersed thermal burns, and severe frostbite. Healing requires surgical intervention, such as skin grafting, and significant scarring is inevitable.

Clinical Images of Selected Burn Injuries

Thermal 2nd degree burn in a child
Thermal burn in an adult patient
Thermal injury - 2nd degree burn in a child
Electrical injury/burn - entry wound
Electrical injury/burn - exit wound
Chemical burn
Chemical burn
Corneal Chemical Burn
Thermal burn, Inhalation Injury

Acing Diagnostic Testing

The diagnostic approach to burn patients varies based on the severity of the burn, the suspected complications, and the presence of associated injuries. A systematic evaluation using targeted laboratory tests and imaging helps guide treatment decisions and monitor potential complications.

Patients with Minor Burns

For patients with minor burns and no associated injuries, laboratory testing is generally unnecessary unless other trauma or medical conditions are present.

Patients with Moderate to Severe Burns

Moderate to severe burns necessitate a more comprehensive diagnostic evaluation [2,6]:

  • Complete Blood Count (CBC): Assesses anemia, infection, or thrombocytopenia.
  • Comprehensive Metabolic Panel (CMP): Monitors electrolyte imbalances, fluid shifts, and kidney or liver function.
  • Creatine Kinase (CK): Detects muscle damage.
  • Arterial Blood Gases (ABG) and Carboxyhemoglobin Levels: Essential for suspected inhalation injury to evaluate oxygenation, carbon monoxide poisoning, and acidosis.
  • Blood Cyanide Levels: Performed if cyanide poisoning is suspected, though results may take time. Treatment is often initiated based on clinical suspicion [2].
  • Serum Lactate: Elevated levels indicate tissue hypoperfusion, inadequate resuscitation, or exposure to carbon monoxide or cyanide [6].
  • Coagulation Studies: Identifies coagulopathies, which are common in severe burns.
  • Chest X-Ray (CXR): Evaluates lung damage in inhalation injury and confirms endotracheal tube placement in intubated patients [2,6].

Patients with Electrical Burns

Electrical burns require specialized evaluation due to the unique nature of the injuries:

  • Electrocardiogram (EKG): Necessary for detecting cardiac dysrhythmias, especially in high-voltage injuries. Patients with abnormal EKG findings should be observed until normalization [6].
  • Creatine Kinase (CK): Elevated levels indicate rhabdomyolysis caused by muscle damage [6].
  • Urinalysis: Detects myoglobinuria, a sign of rhabdomyolysis, which can impair kidney function. However, urinalysis has limited specificity [6].

Imaging for Burn Patients

Imaging studies provide critical insights, particularly for inhalation or electrical injuries:

  • Chest X-Ray (CXR): Evaluates lung damage in inhalation injury and confirms endotracheal tube placement in intubated patients. Useful for identifying pulmonary complications, such as pneumothorax, and confirming intubation tube placement [6].
  • Fiberoptic Bronchoscopy: A definitive tool for diagnosing inhalation injury, revealing findings like soot, edema, mucosal blisters, and hemorrhages [5].
  • Chest CT Scan: Offers detailed imaging of lung injuries and is particularly helpful when CXR findings are inconclusive [5].

Risk Stratification

Burn injuries are categorized as minor, moderate, or severe based on several factors that help predict outcomes and guide management. These include the depth of the burn, the percentage of total body surface area (TBSA) affected, and the age of the patient, with burns in individuals under 10 years or over 50 years considered more severe. The presence of associated injuries, such as smoke inhalation or other traumas, also increases the severity. Burns involving high-risk areas—the face, hands, feet, or genitalia—are particularly concerning due to their potential impact on function, aesthetics, and quality of life.

Risk Stratification Criteria

  • Minor
    • Adults: Partial-thickness burns affecting < 15% TBSA
    • Pediatrics: Partial-thickness burns affecting < 10% TBSA
    • No full-thickness burns
    • No involvement of the face, hands, feet, or genitalia
    • No cosmetic impairment
    • Note: Superficial burns are not included in TBSA calculations.
  • Moderate
    • Adults: Partial-thickness burns affecting 15–20% TBSA
    • Pediatrics: Partial-thickness burns affecting 10–15% TBSA
    • Full-thickness burns affecting < 10% TBSA
    • No involvement of the face, hands, feet, or genitalia
    • No cosmetic impairment
  • Severe
    • Adults: Any burn depth affecting > 25% TBSA
    • Pediatrics: Any burn depth affecting > 20% TBSA
    • Full-thickness burns affecting > 10% TBSA
    • Involvement of the face, hands, feet, or genitalia
    • Cosmetic impairment
    • Circumferential burns: Burns extending completely around the chest or a limb:
      • Can cause compartment syndrome or increased pressure in the affected area.
      • This is particularly dangerous in the chest, where it can restrict breathing and may require escharotomy (incisions into the burned tissue) to relieve the pressure.

Referral to a Burn Center
Referral to a specialized burn center is recommended based on the following criteria from the American Burn Association (ABA) [8]:

  • Partial-thickness burns >10% TBSA.
  • Burns involving the face, hands, feet, genitalia, perineum, or major joints.
  • Full-thickness (third-degree) burns in any age group.
  • Electrical or chemical burns.
  • Inhalation injury.
  • Burns in patients with pre-existing conditions that complicate management.
  • Burns with concomitant trauma or special care needs.

Management

Effective management of burn patients begins with prompt stabilization of the airway, breathing, and circulation (ABC). Airway management is critical in cases of full-thickness facial burns, significant soot in the nose or mouth, hoarseness, stridor, respiratory depression, or altered mental status. In such scenarios, establishing a definitive airway through endotracheal intubation is necessary to prevent airway compromise. Breathing should be assessed by monitoring oxygen saturation and providing supplemental oxygen as needed to address hypoxemia, especially in patients with inhalation injuries. Circulation assessment involves evaluating distal pulses, particularly in patients with circumferential burns, which may restrict blood flow and necessitate escharotomy. For burns exceeding 20% TBSA, prompt initiation of intravenous fluid (IVF) resuscitation is essential to maintain hemodynamic stability and prevent burn shock. This systematic approach ensures early intervention to mitigate life-threatening complications. Extensive details on primary and secondary survey was given in the physical examination section.

General Principles in Management of Burns

Burn management follows consistent principles across all mechanisms of injury, prioritizing first aid, pain control, and fluid resuscitation.

First Aid

Immediate first aid involves removing the causative agent and any clothing, jewelry, or objects that may retain heat or constrict circulation. Cooling the affected area with water is effective for small burns but must be used cautiously with larger burns to prevent hypothermia [9].

Analgesia

Burn injuries and wound care are extremely painful, making pain management a critical component of care. Opioid pain medications should be considered to provide adequate relief, particularly for severe burns or during dressing changes [2,6].

Fluid Resuscitation

Fluid replacement is essential for patients with extensive burns to prevent hypovolemia and burn shock. Adults with partial- or full-thickness burns covering >20% TBSA require fluid resuscitation, while this threshold is lower (>10% TBSA) for pediatric and elderly patients [2,6].

Two common formulas guide fluid calculations:

  • Parkland Formula: Volume (mL) = 4 × weight (kg) × % TBSA burned. Half of the total volume is given in the first 8 hours, and the remaining half over the subsequent 16 hours.
  • Modified Brooke Formula: Volume (mL) = 2 × weight (kg) × % TBSA burned for adults, or 3 × weight (kg) × % TBSA burned for children, administered evenly over 24 hours.

Hartmann’s solution or lactated Ringer’s is the preferred replacement fluid. Fluid titration, based on urine output, ensures appropriate volume without overloading:

  • Adults: Maintain urine output at 0.5–1.0 mL/kg/hour.
  • Pediatrics: Maintain urine output at 1.0–1.5 mL/kg/hour.

Fluid resuscitation is a dynamic process requiring hourly re-evaluation to ensure adequacy and prevent complications [2,6]. The fluid rate must be carefully titrated based on the patient’s urinary output and physiological response. Hourly urine output, measured using an indwelling bladder catheter, serves as a reliable indicator of resuscitation adequacy in patients with normal renal function.

  • Adults: Maintain urine output at 0.5 mL/kg/hour (approximately 30–50 mL/hour).
  • Young Children (≤30 kg): Target 1 mL/kg/hour.
  • Pediatric Patients (>30 kg, up to age 17): Maintain output at 0.5 mL/kg/hour.
  • Adults with High-Voltage Electrical Injuries and Myoglobinuria: Ensure a urine output of 75–100 mL/hour until urine clears.

This individualized approach to fluid management helps maintain renal perfusion, ensures effective resuscitation, and minimizes the risk of under- or overhydration.

Thermal Burns

Thermal burns occur when excessive heat is applied to the skin, resulting in tissue destruction. Initially, this process may cause inflammation and initiate the healing response. However, if the heat intensity or duration is sufficient, coagulative necrosis ensues, leading to irreversible cell death and localized tissue loss. The severity and type of burn depend on various factors, including the heat source, duration of exposure, and depth of tissue involvement. 

Thermal 2nd degree burn in a child

The treatment of thermal burns varies based on severity [2, 6, 10].

Minor burns are managed by cleaning the area and applying topical aloe and a barrier dressing. Pain is controlled with oral analgesics, such as NSAIDs or acetaminophen/paracetamol. Patients can be discharged with outpatient follow-up for wound monitoring.

Moderate burns require cleaning with water and debridement of large blisters. Wound care involves the application of a topical antibiotic with a dressing or an antibiotic-impregnated bandage. Pain management may include oral or intravenous analgesia, with narcotics as needed. Fluid resuscitation, either oral or intravenous, is determined by the percentage of total body surface area (%TBSA) affected. Tetanus immunization should be updated if the last dose was over 10 years ago. Consultation with a burn specialist is advised, with possible admission or transfer to a burn center.

Severe burns necessitate cleaning with water, pain management with oral or intravenous analgesia, and application of a dressing without antibiotics or ointments if transfer to a burn center is confirmed. Intravenous fluid resuscitation is essential, along with prompt referral and admission to a burn center. Circumferential full-thickness burns may require escharotomy to prevent complications such as compartment syndrome.

Electrical Burns

Electrical burns can present with a wide range of injuries due to the effects of electrical current and the conversion of electrical energy into thermal injury. High-voltage electrical exposure can also result in blunt trauma caused by the patient being propelled away from the electrical source.

Extent of injuries depends on the voltage type:

  • Low voltage: Commonly seen in children who come into contact with electrical cords or outlets.
  • High voltage: Typically occupational injuries from power lines or utility poles, often leading to deep tissue and organ damage.
  • Lightning: Frequently occurs during outdoor recreational or work activities, especially in rainy seasons.

Deep tissue injury assessment:
Patients presenting with full-thickness burns, painful passive range of motion, and elevated creatine kinase (CK) levels should be presumed to have deep tissue injury.

  • These patients require fluid resuscitation and referral to a burn center when possible.
Electrical injury/burn - entry wound
Electrical injury/burn - exit wound
high voltage electrical injury

Muscle damage results in a breakdown known as rhabdomyolysis, which can lead to renal failure and multi-organ failure if not treated promptly.

electrical injury, rhabdomyolysis

Management [2,3,11]

General Principles

  • Cardiac Monitoring: Patients with suspected electrical burns should undergo continuous cardiac monitoring for 12–24 hours to detect dysrhythmias.
  • Compartment Syndrome Monitoring: Close monitoring is essential for signs of compartment syndrome.
  • Stress Ulcer Prophylaxis: Administer proton pump inhibitors (PPIs) or H2 blockers, especially in patients who are NPO, as electrical burns carry a higher risk of ulcer formation compared to other burns.

Analgesia

  • Severe pain from deep tissue injuries often necessitates IV narcotic analgesia.

Fluids

  • Initiate fluid resuscitation with 1L/hr isotonic fluids in adults.
  • Avoid using the Parkland or Modified Brooke formula, as the %TBSA burned does not accurately reflect the extent of deep tissue injury in electrical burns.
  • Titrate fluid administration to maintain urine output:
    • Adults: 100 mL/hr
    • Children: 1.5–2 mL/kg/hr

Referral
Patients with suspected deep tissue injury should be referred to a burn center when available to ensure comprehensive care.

Chemical Burns

Superficial chemical burns may conceal deeper tissue injuries, making them more challenging to assess than thermal burns. Tissue damage is often underestimated, necessitating frequent reassessment of wounds and clinical status.

Chemical burn

Management [2,12]

  • Fully expose the patient as soon as possible to minimize ongoing tissue damage. Providers should wear personal protective equipment (PPE) before starting decontamination.
  • Copious irrigation is critical and should be performed immediately, continuing for at least 30 minutes or until neutral skin or eye pH is achieved (using serial litmus paper).
  • Exceptions to irrigation: Dry lime, elemental metals, and phenol require alternative treatments instead of water irrigation.
  • Patients with chemical burns should be referred to a burn center for specialized care.

Radiation Burns

  • Cutaneous manifestations of radiation exposure have a slower onset compared to thermal burns [2,13]. Symptoms such as erythema, calor (warmth), and pruritus may appear hours to days after exposure.
  • Waxing and waning of symptoms:
    • A latent phase without visible cutaneous symptoms often follows initial erythema, calor, and pruritus (1–2 days post-exposure).
    • A second wave may occur days to a week later, presenting as erythema, calor, pruritus, desquamation, ulceration, or necrosis.
    • Subsequent waves of symptoms are more common with potent radiation forms (e.g., beta- and gamma-waves), occurring months post-exposure.
  • High radiation doses are associated with systemic effects, including hair loss and acute radiation syndrome (ARS):
    • ARS symptoms include loss of appetite, fatigue, headache, nausea, vomiting, and diarrhea.

Management

  • Anti-inflammatory medications should be administered during the latent phase when cutaneous symptoms are absent.
  • As with chemical burns, all patients with significant radiation burns should be referred to a burn center for evaluation and management.

Inhalation Injuries

General Overview

Inhalational injuries are a leading cause of mortality in burn patients. They are commonly associated with thermal injuries, which cause upper airway edema, and chemical injuries, which result in damage to the lower airway and lung parenchyma.

Assessment

Evaluating for inhalational injuries involves identifying key clinical signs, such as soot in the oropharynx, singed facial hair, or other indications of airway compromise. For chemical burns, determining the substances burned or combusted is critical to understanding the nature of the injury. Diagnostic tools include obtaining arterial blood gas (ABG) analysis and chest X-ray when available to assess respiratory function and lung involvement.

Management [2,5]

Maintaining a Patent Airway

Ensuring a clear airway is critical in burn patients. Prompt airway management is crucial in inhalational injuries. A low threshold for endotracheal intubation is necessary in cases of airway compromise, severe burns, or full-thickness/circumferential burns involving the chest or neck. If progressive airway edema is observed, fiberoptic intubation is preferred, provided it is available. Given the rapid progression of airway edema, early intubation is advised to prevent airway obstruction and ensure adequate ventilation.

Thermal burn, Inhalation Injury

Oxygen Therapy
Patients with suspected inhalation injuries should receive humidified 100% oxygen via a non-rebreather mask immediately. This is particularly important in cases of carbon monoxide poisoning, as high-flow oxygen effectively reduces carboxyhemoglobin levels, improving oxygen delivery to tissues.

Fluid Resuscitation
Inhalation injuries increase fluid requirements beyond those predicted by burn size alone. Fluid resuscitation must be carefully balanced to avoid under-resuscitation, which risks hypoperfusion, and over-resuscitation, which can lead to complications such as pulmonary edema or compartment syndrome.

Medications
Several medications may be employed to address specific symptoms:

  • Bronchodilators: Relieve bronchospasm and improve airway patency.
  • Mucolytics: Help thin and loosen mucus, facilitating its clearance from the airways.
  • Nebulized Heparin: Prevents fibrin cast formation in the airways, reducing the risk of airway obstruction.

Ventilatory Support

Mechanical ventilation may be required for patients with severe respiratory compromise. Ventilator settings must be carefully optimized to prevent ventilator-induced lung injury. Techniques such as low tidal volume ventilation and high-frequency percussive ventilation may offer benefits in managing patients with compromised pulmonary function.

This comprehensive approach ensures effective airway management and respiratory support in burn patients with inhalation injuries.

Special Patient Groups

Pediatric Patients

Thermal Burns

  • Fluid Resuscitation:
    • In addition to using the Parkland formula for fluid replacement, pediatric patients require maintenance intravenous fluids (mIVF) to meet baseline hydration needs.
    • Children under 5 years of age should have glucose added to their mIVF to prevent hypoglycemia.

Electrical Burns

  • The majority of management principles are similar to those for adults.
  • Oral Burns:
    • Oral burns, often caused by chewing on electrical cords, require special attention. Burns at the commissure (corner of the lips) have a high risk of bleeding due to erosion of the labial artery.
    • All significant oral burns should be admitted for observation and plastic surgery consultation to prevent and manage complications.

Pregnant Patients

Electrical Burns

  • For pregnant patients with electrical burns, obstetric consultation is essential to assess maternal and fetal health.
  • Continuous monitoring of fetal heart tones is necessary to evaluate the well-being of the fetus following an electrical injury.

When To Admit This Patient

The American Burn Association released updated guidelines in December 2022 for burn patient referral and management.

Guidelines for Burn Patient Referral - Courtesy of American Burn Association - From: https://ameriburn.org/wp-content/uploads/2023/01/one-page-guidelines-for-burn-patient-referral-16.pdf [14]

According to these guidelines:

  • Moderate to Severe Burns: Patients with moderate to severe burns, as defined by burn depth and total body surface area (TBSA), require hospital admission for comprehensive burn staging and treatment.
  • Minor Burns: Patients with minor burns, such as superficial burns or those involving <10% TBSA superficial partial-thickness burns, can be managed in an outpatient setting.

To prevent secondary infection, patients discharged with minor burns must have access to appropriate topical ointments and dressings. Patients with partial-thickness burns should undergo regular wound checks following discharge to monitor healing and prevent complications.

Revisiting Your Patient

The patient’s burns were classified as moderate to severe, and he was intubated due to the presence of singed nasal hairs and significant respiratory distress. Using the Rule of 9s, the total burn area was calculated to be 31.5% TBSA, including the face (4.5%), the front of both arms and hands bilaterally (4.5% each), and the torso (18%).

Given the depth of the burns, lactated Ringer’s IV resuscitation was initiated, with a target of delivering 4725 mL in the first eight hours, as calculated using the Parkland formula. A Foley catheter was placed, and urine output was titrated to 0.5 mL/kg/hr. The patient also received IV analgesia and was subsequently transferred to a burn center for further management.

Authors

Picture of Michaela Banks

Michaela Banks

Michaela Banks is a current resident at Louisiana State University in New Orleans in Emergency Medicine. She graduated with a degree in Psychology and Global Health from Duke University and went on to obtain her MD and MBA from the University of Virginia. During residency, she has become particularly interested in burns and outcomes, and gave an oral presentation on the “Association Between Compliance with an Organized State Burn Triage Center and Burn Outcomes” at ACEP 22. Michaela also serves on the Emergency Medicine Residents’ Association Board of Directors.

Picture of Anthony Dikhtyar

Anthony Dikhtyar

Dr. Dikhtyar is a graduate of St. George’s University School of Medicine and recently matched into Emergency Medicine at TriStar Skyline Medical Center in Nashville, TN. His professional interests include medical education, medical photography, and global health in the former Soviet Union. His most recent publications can be found in the Visual Journal of Emergency Medicine.

Picture of Jacquelyne Anyaso

Jacquelyne Anyaso

Jackie Anyaso, MD, MBA is a second-generation Nigerian immigrant born and raised in Chicago, Illinois. She attended medical school at the University of Illinois at Chicago and will be completing her emergency medicine training at Harvard-Affiliated Emergency Residency Program. Her ultimate goal is to serve vulnerable populations in efforts to reduce healthcare disparities. Her clinical interests include critical care medicine, global health, and the intersection between medicine and business. Outside of medicine, she enjoys community service, traveling, and spending time with family and friends.

Picture of Ashley Pickering

Ashley Pickering

Before medical school I had a diverse career path, which included biomedical engineering, outdoor education, working as an EMT on a Colorado ski patrol, and critical care nursing. I lived out west for 15 years, mainly in CO, and went to medical school at University of Arizona in Tucson before moving to Baltimore for residency at University of Maryland. Currently I am a Global Emergency Medicine Fellow at University of Colorado. Throughout my training I have found ample opportunities to pursue my interest in building emergency care globally. I have researched the barriers to accessing emergency care in rural Uganda, helped to provide emergency care training in Sierra Leone and Liberia and am currently the Executive Director of Global Emergency Care a non-profit training non-physician clinicians in Uganda. My current focus is on quality of emergency care in LMICs. I am working on an WHO Emergency Care Toolkit implementation project which explores the impact of basic emergency care educational and process improvements on clinical indicators of quality, as well as the experiences patients and staff.

Listen to the chapter

References

  1. Jeschke MG, Mlcak RP, Finnerty CC, et al. Burn size determines the inflammatory and hypermetabolic response. Crit Care. 2007;11(4):R90. doi:10.1186/cc6102
  2. American Burn Association. (2018). Advanced Burn Life Support Course Provider Manual 2018 Update. https://ameriburn.org/wp-content/uploads/2019/08/2018-abls-providermanual.pdf
  3. Schaefer TJ, Szymanski KD. Burn Evaluation And Management. [Updated 2022 Aug 23]. In: StatPearls [Internet]. Treasure Island (FL): StatPearls Publishing; 2023 Jan-. Available from: https://www.ncbi.nlm.nih.gov/books/NBK430741/
  4. Jeschke MG, van Baar ME, Choudhry MA, Chung KK, Gibran NS, Logsetty S. Burn injury. Nat Rev Dis Primers. 2020;6(1):11. Published 2020 Feb 13. doi:10.1038/s41572-020-0145-5
  5. Foncerrada G, Culnan DM, Capek KD, et al. Inhalation Injury in the Burned Patient. Ann Plast Surg. 2018;80(3 Suppl 2):S98-S105. doi:10.1097/SAP.0000000000001377
  6. Emergency Care of Moderate and Severe Thermal Burns in Adults. UpToDate. Feb. 2023. https://www.uptodate.com/contents/emergency-care-of-moderate-and-severe-thermal-burns-in-adults?topicRef=349&source=see_link#H4430737.
  7. Department of Health. Determining Total Body Surface Area. From: https://www.health.state.mn.us/communities/ep/surge/burn/tbsa.pdf Accessed December 1, 2024.
  8. Guidelines for Burn Patient Referral. From: https://ameriburn.org/resources/burnreferral/ Accessed: December 1, 2024.
  9. Burns. WikiEM. 21 Nov. 2021; 4:1-2. https://wikem.org/wiki/Burns#Evaluation.
  10. Treatment of Minor Thermal Burns. UpToDate. Feb. 2023. https://www.uptodate.com/contents/treatment-of-minor-thermal-burns#H20.
  11. Electrical injuries and lightening strikes: Evaluation and management. UpToDate. Mar 2023. https://www.uptodate.com/contents/electrical-injuries-and-lightning-strikes-evaluation-and-management#H3065280448
  12. Topical chemical burns: Initial assessment and management. UpToDate. Mar 2023. https://www.uptodate.com/contents/topical-chemical-burns-initial-assessment-and-management
  13. Cutaneous Radiation Injury (CRI): A Fact Sheet for Clinicians. 4 Apr. 2018. https://www.cdc.gov/nceh/radiation/emergencies/criphysicianfactsheet.htm
  14. Guidelines for Burn Patient Referral. From: https://ameriburn.org/wp-content/uploads/2023/01/one-page-guidelines-for-burn-patient-referral-16.pdf

FOAm and Further Reading

Reviewed and Edited By

Picture of Erin Simon, DO

Erin Simon, DO

Dr. Erin L. Simon is a Professor of Emergency Medicine at Northeast Ohio Medical University. She is Vice Chair of Research for Cleveland Clinic Emergency Services and Medical Director for the Cleveland Clinic Bath emergency department. Dr. Simon serves as a reviewer for multiple academic emergency medicine journals.

Picture of Arif Alper Cevik, MD, FEMAT, FIFEM

Arif Alper Cevik, MD, FEMAT, FIFEM

Prof Cevik is an Emergency Medicine academician at United Arab Emirates University, interested in international emergency medicine, emergency medicine education, medical education, point of care ultrasound and trauma. He is the founder and director of the International Emergency Medicine Education Project – iem-student.org, chair of the International Federation for Emergency Medicine (IFEM) core curriculum and education committee and board member of the Asian Society for Emergency Medicine and Emirati Board of Emergency Medicine.

Bronchial Foreign Body Aspiration (2024)

by Elhaitham Ahmed & Khalifa Alqaydi

You have a new patients!

Patient 1

A 72-year-old male was brought from an inpatient stroke rehabilitation center to the emergency department for a cough lasting the past ten days. Along with the cough, the patient was noted to have blood-tinged sputum, which is sometimes foul-smelling. His vital signs are as follows: temperature of 38.4°C, blood pressure of 138/78 mmHg, heart rate of 103 bpm, respiratory rate of 26 breaths/min, and oxygen saturation of 93% on room air. On physical examination, the patient exhibits tachypnea, dullness on percussion, bronchial breathing, egophony, and increased vocal fremitus upon examining the right side of his lung.

The image was produced by using ideogram 2.0.

Patient 2

Thirty minutes later, the nurse calls you regarding a 5-year-old boy brought in by his mother, presenting with stridor and an ongoing cough. The mother mentions that she found her child playing with her wallet while she was in the next room and discovered him in this condition. The child is tachypneic, saturating at 90% on room air with subcostal retractions. Examination of the right lung revealed wheezing with decreased air entry.

a-photo-of-a-5-year-old-male-patient-(the image was produced by using ideogram 2.0)

What do you need to know?

Importance

Tracheobronchial foreign body aspiration (FBA) can be a potentially life-threatening event. FBA in children may be suspected based on a choking episode if such an episode is witnessed by an adult or remembered by the child. In contrast, the clinical presentation of unwitnessed FBA may be subtle, requiring careful review of the history, clinical assessment, and judicious use of radiography and bronchoscopy for diagnosis. Flexible and rigid bronchoscopy have become the cornerstone of both diagnosis and treatment in patients with suspected airway foreign bodies, which are most commonly seen in patients with FBA [1].

Epidemiology

FBA is more common in children than in adults. Data from the National Security Council report that approximately 80 percent of cases occur in patients younger than 15 years of age, with the remaining 20 percent presenting in those older than 15 years. Overall, death from FBA is the fourth leading cause of accidental home and community deaths in the United States, with over 5,000 fatal episodes of FBA reported during 2015. Death from FBA peaks in children under 1 year old and in adults over 75 years [2].

Pathophysiology

In children, nuts, seeds, and other organic material account for the majority of foreign bodies. However, in adults, the nature of inhaled objects is highly variable, ranging from organic to inorganic material. The type of foreign body significantly impacts the degree of tissue reaction in the airway. For example, some inorganic materials, such as metal or glass items, may cause little tissue inflammation but can result in direct airway injury if they are sharp. In contrast, some organic materials, such as nuts and a variety of pills, can cause significant inflammation, granulation tissue formation, and airway stenosis. Aspirated organic material can also expand from airway moisture, worsening obstruction. Aspiration of medications in pill form, such as iron tablets, aspirin, and potassium chloride, can also cause severe airway inflammation and ulceration [2].

Medical History

Clinical presentation can range from chronic nonspecific respiratory complaints to acute airway obstruction. In most cases of aspiration, the presence of a foreign body can be suspected after a thorough history. Patients with airway foreign bodies may present with noisy breathing, inspiratory stridor, rhonchi, vomiting, changes in voice, and hemoptysis [3]. Some patients may report a history known as penetration syndrome, which includes a choking sensation accompanied by wheezing and coughing. Coughing may not completely expel the foreign body but may instead cause its impaction in the subglottic region. Therefore, coughing after suspected aspiration should prompt a search for a foreign body, even if symptoms improve [4].

In pediatric patients with suspected foreign body aspiration, the sudden onset of choking or intractable cough associated with wheezing and respiratory distress occurs in more than 63% of cases [5,6]. In addition to coughing and choking, stridor is a frequent symptom. The absence of early coughing and choking is associated with delayed diagnosis and chronic presentations, such as recurrent pneumonia [4]. The sudden onset of dyspnea and odynophagia may indicate an impacted subglottic object. If the object is sharp and thin, the emergency clinician should suspect embedding between the vocal cords or in the subglottic region, resulting in partial obstruction [7].

Other components of the history can assist in diagnosing and characterizing foreign bodies in patients with aspiration of nonfood objects. Many types of items may be aspirated by children exploring their environment. Another at-risk population includes individuals who habitually store small items in their mouths for quick access; examples include construction workers (nails) and seamstresses (pins). The presentation of patients with a retained airway foreign object may involve only infectious complications. A foreign object can lead to a retropharyngeal abscess. Patients with atypical or recurrent pneumonia may have pulmonary infections caused by the persistence of a foreign object serving as a focus of infection [6].

Physical Examination

Physical findings depend on the degree of airway obstruction and the duration of the object’s presence. Depending on the size and location of the foreign body, the examination may reveal a normal patient, one with cyanosis and respiratory arrest, or any condition between these two extremes. Patients may exhibit stridor or hoarseness with upper airway foreign objects, and intercostal or sternal retractions may be observed in patients with high-grade obstruction caused by tracheal foreign bodies [8]. Hypoxemia may be present; however, normoxia does not rule out the presence of a foreign body. Patients with secondary infections may present with fever.

Oropharyngeal examination may reveal a foreign body posteriorly or donor sites of fractured teeth. The examination should also include a search for fractured or missing dental prostheses. Oropharyngeal examination can often be supplemented by indirect or direct laryngoscopy or nasopharyngoscopy, but these procedures should be performed only if the procedural stress does not pose an undue risk of airway compromise.

Coughing may result from local irritation caused by bronchial foreign bodies. Localized or apparently generalized wheezing is frequently auscultated in patients with lower respiratory tract foreign bodies [9]. Complete obstruction of a mainstem bronchus may result in absent ipsilateral breath sounds; however, breath sounds can sometimes be transmitted across the thorax, and the only physical abnormality may be asymmetric chest rise. Occasionally, a foreign body acts as a one-way valve, allowing air into the lung during inspiration but preventing its exit during expiration. The affected lung becomes hyperexpanded, which may be detected as hyper-resonance on percussion [6].

Alternative Diagnoses

The selected differential diagnoses for airway foreign bodies include anaphylactic reactions, acute pharyngitis, acute epiglottitis, retropharyngeal abscess, neck tumors, pulmonary carcinomas, pneumonia, bronchitis, bronchiolitis, and tuberculosis.

Acing Diagnostic Testing

Imaging should not delay intervention in cases of suspected acute asphyxiation but is indicated for stable patients [10].

Findings on imaging depend on the type and location of the material aspirated and the time elapsed. In practice, plain films of the neck and chest are often performed simultaneously and can be followed by site-specific CT if suspicion remains. The majority of foreign bodies are radiolucent and not easily identified on plain film. If obstruction of the upper airway (oropharynx and upper trachea) is suspected, initial imaging should include anterior-posterior and lateral soft tissue views of the neck [11]. If these tests are negative and suspicion for FBA persists, further imaging with CT may be indicated. When FBA of the lower airways (below the vocal cords) is suspected, a chest radiograph should be the initial radiographic test to look for an obvious radiopaque airway lesion. Negative scans may prompt further evaluation with CT. The reported sensitivity of chest radiography is approximately 60 to 80 percent in children, and clinical experience suggests similarly poor sensitivity in adults [12].

Given its widespread availability, flexible bronchoscopy is often the diagnostic procedure of choice for non-life-threatening FBA in adults, particularly in cases involving smaller foreign bodies in the lower airway. Flexible bronchoscopy allows precise identification and localization of foreign bodies and facilitates the selection of instruments necessary for retrieval [13]. Additionally, flexible bronchoscopy enables removal of the foreign body during the diagnostic procedure if the operator is skilled in these techniques. Standard diagnostic or therapeutic flexible bronchoscopes are usually adequate for the management of FBA in adults [6].

Risk Stratification

Risk factors in adults include loss of consciousness due to trauma, drug or alcohol intoxication, or anesthesia. Additional risk factors in older adults include age-related slowing of the swallowing mechanism, medication use (impairing cough and swallowing), stroke-related dysphagia, and various degenerative neurologic diseases such as Alzheimer’s or Parkinson’s disease [2].

Management

In a conscious adult, data support the efficacy of chest thrusts, back blows or slaps, blind finger sweeps, and abdominal thrusts in relieving complete foreign body airway obstruction [6, 14]. In cases of life-threatening asphyxiation, initial support should focus on treating airway obstruction and respiratory failure. Once the airway is secured, a laryngoscopic evaluation of the oropharynx should be performed immediately to diagnose and retrieve a supraglottic or glottic foreign body. If a foreign body is not seen, rigid bronchoscopy is generally the procedure of choice for suspected asphyxiating foreign bodies located in the trachea or major bronchi. In patients with non-life-threatening FBA, flexible bronchoscopy is typically performed [15].

When large foreign bodies completely or almost completely obstruct major upper airways (glottis, supraglottis, trachea), it is critical to ensure the patient is oxygenated and the airway is secured [16]. Support measures may include bag-valve-mask ventilation and endotracheal intubation. If ventilation is unsuccessful, an emergent cricothyrotomy or tracheotomy may be required if the foreign body is suspected to be above the vocal cords. Once the airway is secured, immediate inspection of the oropharynx (glottis, supraglottis) is indicated, as one-third of FBA cases presenting as acute asphyxiation are located in the supraglottis. Retrieval of the foreign body with Magill forceps can be safely performed using direct laryngoscopy (glottis, supraglottis) or with smooth or alligator forceps during rigid or flexible bronchoscopy (large central foreign body in the trachea or major bronchus) [17].

The choice of procedure for foreign body removal depends on the type of presentation, characteristics of the inhaled foreign body, its location, the duration it has been in the airway (if known), and local expertise. Anti-inflammatories and antibiotics are not routinely administered to patients with suspected or documented FBA. Antibiotics are indicated only in cases of clinically, radiologically, or microbiologically documented respiratory tract infections. However, their use should not delay foreign body extraction, even if pneumonia or sepsis is suspected [2].

Figure 1 - Approach to Upper Airway Foreign Body. Original Image can be found here: White JJ. Upper Airway Foreign Bodies: Emergency department presentation, Evaluation and Management. emDOCs.net - Emergency Medicine Education. April 12, 2021. Accessed May 9, 2023. http://www.emdocs.net/upper-airway-foreign-bodies-emergency-department-presentation-evaluation-and-management/.

Special Patient Groups

In the pediatric age group, moderate or high suspicion of FBA is suggested by any of the following:

  • Witnessed FBA, regardless of symptoms.
  • History of choking, with any subsequent symptoms or suspicious characteristics on imaging.
  • A young child with suggestive symptoms without another explanation, especially if there are suspicious characteristics on imaging. Suspicious symptoms include cyanotic spells, dyspnea, stridor, sudden onset of cough or wheezing (often focal and monophonic), and/or unilaterally diminished breath sounds.

The tracheobronchial tree should be examined in all cases with moderate or high suspicion of FBA, using rigid bronchoscopy (or, in some cases, computed tomography [CT]). On occasion, the adjunctive use of a flexible bronchoscope may be helpful. Normal chest radiographs are not sufficient to rule out FBA [19], primarily because most foreign bodies are radiolucent. Morbidity and mortality may increase if bronchoscopic evaluation is delayed.

When To Admit This Patient

Most patients improve clinically following FBA removal. Those with imaging abnormalities should undergo follow-up imaging six weeks to three months after extraction to confirm resolution. Patients presenting with a delayed presentation and belonging to high-risk groups should be admitted for management of complications and FBA retrieval and removal.

Revisiting Your Patients

The elderly patient, given his history of a recent stroke and being in a rehabilitation center, is at risk of FBA. His presentation with chronic cough and fever raises suspicion of pneumonia; however, the emergency medicine clinician should maintain a broad differential diagnosis based on further history, including foul-smelling sputum and nursing staff observations of difficulty swallowing and previous admissions for pneumonia. Such delayed presentations of FBA can occur in this age group. The patient’s management began with initial stabilization using oxygen support, along with workup for infection. Imaging modalities started with a chest plain film, which showed right lower lobe opacities but no clear foreign body. With suspicion for FBA still high, a chest CT scan was performed and revealed evidence consistent with FBA. The patient was started on broad-spectrum antibiotics, and bronchoscopy was scheduled as the definitive management for FBA. Follow-up bronchoscopy identified distal fragments of nuts impacted in the right lower lobe bronchus.

In the pediatric patient, the presentation is more acute and requires securing the airway. After placing the patient on a non-rebreather mask with 15L of oxygen, his saturation improved to 100%. Given the history of playing with a wallet, suspicion of coin aspiration was considered. A chest radiograph with posteroanterior and lateral views was performed, showing a rounded radiopaque structure in the right main bronchus. Airway support and supplemental oxygen should be provided until bronchoscopy is performed and the coin is retrieved.

Authors

Picture of Elhaitham Ahmed

Elhaitham Ahmed

Zayed Military Hospital, AbuDhabi

Picture of Khalifa Alqaydi

Khalifa Alqaydi

Zayed Military Hospital, AbuDhabi

Listen to the chapter

References

  1. Ruiz, F.E. (2022) Airway foreign bodies in children, UpToDate. Available at: https://www.uptodate.com/contents/airway-foreign-bodies-in-children?search=airway+foreign+bodies+in+children&source=search_result&selectedTitle=1~83&usage_type=default&display_rank=1 (Accessed: 08 May 2023).
  2. Shepherd, W. (2023) Airway foreign bodies in adults, UpToDate. Available at: https://www.uptodate.com/contents/airway-foreign-bodies-in-adults?search=adult+forign+body+&source=search_result&selectedTitle=3~150&usage_type=default&display_rank=3 (Accessed: 08 May 2023).
  3. Bajaj D, Sachdeva A, Deepak D. Foreign body aspiration. J Thorac Dis. 2021;13(8):5159-5175. doi:10.21037/jtd.2020.03.94
  4. Dabu J, Lindner M, Azzam M, et al. A Case of Chronic Cough and Pneumonia Secondary to a Foreign Body. Case Rep Med. 2017;2017:3092623. doi:10.1155/2017/3092623
  5. Mîndru DE, Păduraru G, Rusu CD, et al. Foreign Body Aspiration in Children-Retrospective Study and Management Novelties. Medicina (Kaunas). 2023;59(6):1113. Published 2023 Jun 9. doi:10.3390/medicina59061113
  6. Goodloe JM, Soulek J. Foreign Bodies . In: Rosen’s Emergency Medicine Concepts and Clinical Practice. 10th ed. Elsevier; 2023:666-681.
  7. Hazra TK, Ghosh AK, Roy P, Roy S, Sur S. An impacted meat bone in the larynx with an unusual presentation. Indian J Otolaryngol Head Neck Surg. 2005;57(2):145-146. doi:10.1007/BF02907672
  8. Swanson KL, Edell ES. Tracheobronchial foreign bodies. Chest Surg Clin N Am. 2001;11(4):861-872.
  9. Kazmerski T, Dedhia K, Maguire R, Aujla S. Chronic Esophageal Foreign Body Presenting as Wheezing and Cough in a Toddler. Pediatr Allergy Immunol Pulmonol. 2014;27(3):151-153. doi:10.1089/ped.2014.0370
  10. White JJ, Cambron JD, Gottlieb M, Long B. Evaluation and Management of Airway Foreign Bodies in the Emergency Department Setting. J Emerg Med. 2023;64(2):145-155. doi:10.1016/j.jemermed.2022.12.008
  11. António P, Raffaella C, Luigia R. Plain Film and MDCT Assessment of Neck Foreign Bodies. 2014;1007/978-88-470-5406-6_1.
  12. Svedström E, Puhakka H, Kero P. How accurate is chest radiography in the diagnosis of tracheobronchial foreign bodies in children?. Pediatr Radiol. 1989;19(8):520-522. doi:10.1007/BF02389562
  13. Turk D, Moslehi MA, Hosseinpour H. Role of Flexible Fiberoptic Bronchoscopy in the Diagnosis and Treatment of Pediatric Airway Foreign Bodies: A 5-Year Experience at a Tertiary Care Hospital in Iran. Tanaffos. 2022;21(3):354-361.
  14. Pavitt MJ, Swanton LL, Hind M, et al. Choking on a foreign body: a physiological study of the effectiveness of abdominal thrust manoeuvres to increase thoracic pressure. Thorax. 2017;72(6):576-578. doi:10.1136/thoraxjnl-2016-209540
  15. Bodart E, Gilbert A, Thimmesch M. Removal of an unusual bronchial foreign body: rigid or flexible bronchoscopy?. Acta Clin Belg. 2014;69(2):125-126. doi:10.1179/2295333714Y.0000000006
  16. Davis RJ, Stewart CM. Complete Glottic Obstruction by an Unusual Foreign Body. Otolaryngol Head Neck Surg. 2019;160(5):935-936. doi:10.1177/0194599818824298
  17. Singh GB, Aggarwal D, Mathur BD, Lahiri TK, Aggarwal MK, Jain RK. Role of magill forcep in retrieval of foreign body coin. Indian J Otolaryngol Head Neck Surg. 2009;61(1):36-38. doi:10.1007/s12070-009-0031-7
  18. White Upper Airway Foreign Bodies: Emergency department presentation, Evaluation and Management. emDOCs.net – Emergency Medicine Education. April 12, 2021. Accessed May 9, 2023. http://www.emdocs.net/upper-airway-foreign-bodies-emergency-department-presentation-evaluation-and-management/.
  19. Pinto A, Scaglione M, Pinto F, et al. Tracheobronchial aspiration of foreign bodies: current indications for emergency plain chest radiography. Radiol Med. 2006;111(4):497-506. doi:10.1007/s11547-006-0045-0

Reviewed and Edited By

Picture of Arif Alper Cevik, MD, FEMAT, FIFEM

Arif Alper Cevik, MD, FEMAT, FIFEM

Prof Cevik is an Emergency Medicine academician at United Arab Emirates University, interested in international emergency medicine, emergency medicine education, medical education, point of care ultrasound and trauma. He is the founder and director of the International Emergency Medicine Education Project – iem-student.org, chair of the International Federation for Emergency Medicine (IFEM) core curriculum and education committee and board member of the Asian Society for Emergency Medicine and Emirati Board of Emergency Medicine.

Fundamentals of Pediatric Advanced Life Support (2024)

by Burak Çakar & Ayça Koca

Introduction

Pediatric cardiac arrest (CA) is a rare but critical event associated with high mortality and significant risk of severe sequelae [1,2]. Unlike in adults, respiratory causes are the primary contributors to CA in children. Hypoxia and bradycardia can lead to cardiopulmonary failure, which may ultimately progress to CA.

Common causes of pediatric CA include infections (e.g., pneumonia, sepsis), trauma, asphyxia, seizures, asthma, suffocation, and sudden infant death syndrome [2]. Clinical signs of cardiopulmonary arrest include respiratory arrest, absence of a palpable pulse, muscle flaccidity, unresponsiveness, cyanosis or other discoloration, and dilated pupils. Recognizing and promptly addressing these signs is crucial for improving outcomes.

Recognition of a Critically Ill Child

Early recognition of a critically ill child is essential to implementing timely interventions that may prevent progression to CA [3]. Abnormal vital signs, relative to age-specific norms, are often the most reliable indicators of impending arrest.

Pediatric Early Warning Scores (PEWS) are recommended as a systematic tool to identify children at risk of clinical deterioration [4]. PEWS evaluates three domains: behavior, cardiovascular function, and respiratory status [5]. It incorporates vital findings such as respiratory rate, heart rate, blood pressure, oxygen saturation, temperature, level of consciousness, and capillary refill time [6]. 

Monitoring Vital Signs in Children

It is essential to recognize abnormal vital signs for early recognition of pediatric deterioration.

Blood Pressure

Systolic Hypotension is defined as a systolic blood pressure below the 5th percentile for age. The threshold for concern is when the systolic blood pressure is <70 mmHg + (2x the child’s age in years).

Respiratory Rate

Tachypnea: A respiratory rate exceeding 60 breaths per minute indicates tachypnea.
Decreased Respiratory Rate: A reduction in respiratory rate in a previously tachypneic patient could signal either improvement or fatigue. Fatigue in this context could precede respiratory failure, particularly if it occurs in conjunction with other signs of decompensation.

Temperature

Fever significantly affects physiology. For every 1°C increase in body temperature:

  • The heart rate increases by approximately 10 beats per minute.
  • The respiratory rate increases by 2 to 5 breaths per minute.

End-Tidal Carbon Dioxide (EtCO2)

Changes in EtCO2 levels are critical indicators of respiratory status. A progressive increase or decrease in EtCO2 levels can signal impending desaturation and respiratory failure.

Assessment

Given the poor outcomes associated with pediatric CA, the emphasis must be on early recognition of pre-arrest states. Identifying signs of impending respiratory failure and shock, regardless of their underlying cause, should be a primary focus [4].

Findings Preceding Cardiopulmonary Arrest

Key findings preceding cardiopulmonary arrest are categorized as follows:

  1. Airway: Signs include stridor, drooling, and retractions, which indicate significant airway obstruction or distress.

  2. Breathing: Irregular respiration, bradypnea, gasping respirations, and cyanosis are warning signs of severe respiratory compromise.

  3. Circulation: Indicators such as a capillary refill time greater than 5 seconds, bradycardia, hypotension, cool extremities, weak central pulses, and the absence of peripheral pulses suggest circulatory failure.

  4. Disability: An altered level of consciousness and decreased responsiveness point toward significant neurological impairment, often accompanying or preceding arrest.

Initial Assessment

The initial assessment begins with a first impression of the child’s general appearance, breathing pattern, and circulatory status. During the primary assessment, the ABCDE approach is followed, with immediate interventions performed at each step when abnormalities are identified. Initial management focuses on supporting airway, breathing, and circulation [7].

The clinician should rapidly assess the following:

Airway

  • Assess for patency (open, requiring maneuvers/adjuncts, partially or completely obstructed)
  • Perform cervical spine stabilization for injured children.
  • Provide 100% inspired oxygen, clear the airway (e.g., suction), apply airway maneuvers, and insert airway adjuncts if the child is unconscious.
  • Initiate chest compressions immediately if the child is unresponsive and shows no signs of life.
  •  

 Breathing

  • Evaluate respiratory rate, effort, tidal volume, lung sounds, and pulse oximetry.
  • Assist ventilation manually for patients unresponsive to basic airway maneuvers or exhibiting inadequate respiratory effort.
  • Monitor oxygenation and ventilation using pulse oximetry and ETCO₂.
  • Administer appropriate medications based on the cause of respiratory distress (e.g., albuterol for status asthmaticus, inhaled racemic epinephrine for croup).
  • Consider intubation when necessary, ensuring 100% oxygen delivery via a non-rebreather mask. Apply positive pressure ventilation with a bag-valve-mask (BVM) in cases of respiratory failure.

Circulation

  • Assess skin color and temperature, heart rate and rhythm, blood pressure, peripheral and central pulses, and capillary refill time.
  • Control hemorrhage in injured children.
  • For circulation deficiencies, monitor heart rate and rhythm, and establish vascular access for volume resuscitation or medication administration.

Disability

  • Evaluate neurological status and level of consciousness using the AVPU scale (Alert, Voice, Pain, Unresponsive) and the Glasgow Coma Scale (GCS) for trauma patients.
  • Assess pupil size and reactivity to light.
  • Check for hypoglycemia using rapid bedside glucose testing or by observing the response to empiric dextrose administration.

Exposure

  • Examine for skin findings, fever or hypothermia, and evidence of trauma.

Secondary and Tertiary Assessments

  • The secondary assessment involves a detailed head-to-toe physical examination, supplemented with a medical history.
  • The tertiary assessment focuses on identifying the underlying causes of trauma, illness, or infection through ancillary studies.

Respiratory Distress and Failure

Respiratory distress and failure are common precursors to CA in children. Early recognition of breathing difficulties is essential to improving clinical outcomes.

Respiratory distress is characterized by tachypnea, nasal flaring, retractions, and the use of accessory muscles. Additional signs include agitation, hypoxia, and abnormal breath sounds, such as stridor or wheezing. If not promptly addressed, these findings can progress to a decreased respiratory rate, respiratory fatigue, and eventual respiratory arrest.

Bradycardia

In children, bradycardia is often secondary to hypoxia. A heart rate slower than the age-appropriate normal range is indicative of bradyarrhythmia. Management focuses on optimizing oxygenation and ventilation through basic airway maneuvers.

If the heart rate remains below 60 beats per minute despite adequate oxygenation and ventilation, chest compressions should be initiated. Epinephrine (0.01 mg/kg) should be administered every 3–5 minutes. For bradycardia caused by increased vagal tone or primary atrioventricular block, atropine (0.02 mg/kg; maximum single dose 0.5 mg) is recommended [7].

Tachycardia

Tachycardia refers to a heart rate that exceeds the normal range for a child’s age, considering other factors such as physical activity or fever. The management of tachycardias depends on the child’s hemodynamic condition and rhythm [7].

Pulseless Arrest

Pediatric CAs are typically the result of cardiopulmonary distress, failure, or shock. When a child has no palpable pulse and is unresponsive, cardiopulmonary resuscitation (CPR) should be initiated.

A child with pulseless arrest will present as apneic and may exhibit gasping respirations. The rhythms associated with pulseless arrest include [2]:

Shockable rhythms: Ventricular fibrillation (VF), pulseless ventricular tachycardia (pVT).

Ventricular Fibrillation

Ventricular Tachycardia

Unshockable rhythms: Asystole, pulseless electrical activity (PEA).

Asystole

Pulseless Electrical Activity (PEA)

During CPR, reversible causes of PEA should be actively identified and addressed. The mnemonic 6H5T is useful for recalling these potential causes [2]:

  • 6 H’s:
    • Hydrogen ion (acidosis)
    • Hypoxia
    • Hypovolemia
    • Hypo- or hyper -kalemia, -calcemia, -magnesemia
    • Hypoglycemia
    • Hypo- or hyperthermia
  • 5 T’s:
    • Tension pneumothorax
    • Tamponade
    • Thrombosis (cardiac)
    • Thrombosis (pulmonary)
    • Toxic agents

By addressing these potential causes, advanced life support providers can significantly improve the likelihood of successful resuscitation.

Resuscitation

An effective resuscitation team is critical to the successful management of pediatric advanced life support (PALS). The team must perform multiple tasks simultaneously, including airway management, ventilation, vascular access, medication preparation and administration, chest compressions, monitor/defibrillator operation, recording/timing, and overall leadership.

The team leader plays a pivotal role by assigning tasks, directing team members, and modeling exemplary teamwork. In addition to medical expertise and resuscitation skills, the team must demonstrate effective communication. Key elements of effective team dynamics include:

  • Closed-loop communication
  • Clear messages
  • Defined roles and responsibilities
  • Knowing and communicating one’s limitations
  • Knowledge sharing
  • Constructive interventions
  • Reevaluation and summarization
  • Mutual respect [8]

Initiation of CPR

Timely recognition of CA, prompt initiation of high-quality chest compressions, and ensuring adequate ventilation are crucial for improving outcomes [2].

Healthcare providers should begin chest compressions promptly in any child who is unresponsive, not breathing normally, and has no signs of circulation [7, 9]. Pulse checks may be performed but should not delay the initiation of CPR for more than 10 seconds [10, 11]. Pulse palpation alone is unreliable in determining the need for compressions or confirming CA.

Since respiratory-related CA is more common in infants and children than primary cardiac causes, ensuring adequate ventilation during resuscitation is essential [2]. The recommended sequence for CPR is compressions-airway-breathing (CAB) [12].

High-quality CPR enhances blood flow to vital organs and increases the likelihood of return of spontaneous circulation (ROSC). The five key components of high-quality CPR are [2, 13]:

  • Optimal chest compression rate
  • Sufficient chest compression depth
  • Minimal interruptions in compressions
  • Complete chest recoil between compressions [7, 14]
  • Avoidance of excessive ventilation

Components of High-Quality CPR

  • Compression Rate: 100–120 compressions per minute [15–18].
  • Compression Depth:
    • At least one-third of the anterior-posterior diameter of the chest:
      • 4 cm for infants
      • 5 cm for children
      • 5–6 cm for adolescents who have reached puberty [7, 19].
    • Allow complete chest recoil after each compression.
    • Use 100% oxygen with a bag-valve-mask (BVM) during CPR.
    • Compression-Ventilation Ratios:
      • 30:2 for single rescuers.
      • 15:2 for two rescuers [7, 20].

To prevent fatigue and ensure adequate compressions, switch the person performing compressions at least every 2 minutes or sooner if necessary [7].

CPR Technique

For Infants

Single Rescuer: Use two fingers (Figure 1) or two thumbs below the nipple line (lower half of the sternum but one-finger width above the xiphisternum) [21–24].

Figure 1. Two-finger compressions

Two Rescuers: Use the two-thumb encircling hands technique (Figure 2) [25–29].

Figure 2. Thumb-encircling hands compression

If the recommended depth cannot be achieved, use the heel of one hand (Figure 3) [2, 18, 30, 31].

Figure 3. Compression with the heel of one hand

For children older than 1 year

Use either one-handed or two-handed CPR.

Perform chest compressions on a firm surface. Use a backboard or activate the bed’s “CPR mode” if available [32–35].

The Airway

Unless a cervical spine injury is suspected, the head tilt-chin lift maneuver is recommended to open the airway [36]. In trauma patients with suspected cervical spinal injury, the jaw thrust maneuver should be used. If the jaw thrust is ineffective, the head tilt-chin lift may be performed, even in cases of suspected cervical spine injury [2].

Use 100% oxygen delivered via bag-valve-mask (BVM) during CPR.

Advanced Airway Interventions During CPR

Bag-mask ventilation (BMV) is effective for most patients but requires pauses in chest compressions and carries risks of aspiration and barotrauma [2]. Advanced airway interventions, such as supraglottic airway (SGA) placement or endotracheal intubation (ETI), improve ventilation, reduce aspiration risks, and enable uninterrupted chest compressions. However, these procedures require specialized equipment and trained providers, and may be challenging for those inexperienced in pediatric intubation [2]. BMV is more reliable than advanced airway interventions during out-of-hospital pediatric CA [37–39].

For patients with advanced airway, ventilations should be asynchronous. Exceeding recommended ventilation rates can compromise hemodynamics and lower systolic blood pressure [40].

Ventilations should be tailored to age:

  • 25 breaths/min (infants)
  • 20 breaths/min (>1 year)
  • 15 breaths/min (>8 years)
  • 10 breaths/min (>12 years) [7].

Capnography should be used to confirm endotracheal tube placement and monitor for ROSC. However, ETCO₂ should not be used as a definitive quality indicator or target during PALS [7].

Drug Administration During CPR

Establish IV access as early as possible during PALS. If IV access is challenging, promptly consider intraosseous (IO) access as an alternative [7]. Drug dosing for children is typically based on weight, which can be challenging to determine in emergencies. When the actual weight cannot be obtained, various estimation methods are available [41]

The administration of vasoactive agents during CA aims to improve coronary and cerebral perfusion and increase the likelihood of ROSC. However, optimal timing and overall impact on long-term outcomes are still under investigation [42]

  • Administer epinephrine (10 mcg/kg; max 1 mg; IV or IO ) as soon as possible for non-shockable rhythms. For shockable rhythms, administer epinephrine immediately after the third shock, along with antiarrhythmic drugs. Once given, adrenaline should be repeated every 3–5 minutes until ROSC.

Antiarrhythmic drugs can reduce the risk of recurrent VF or pVT and improve the likelihood of successful defibrillation [43, 44]. Only in shockable rhythms, administer antiarrhythmic drugs immediately after the third shock, along with epinephrine.

  • Amiodarone: 5 mg/kg (max 300 mg); a second dose (max 150 mg) may follow after the fifth shock if the rhythm remains shockable.
  • Lidocaine: 1 mg/kg, as an alternative to amiodarone.

Magnesium sulfate (25–50 mg/kg) should be considered for torsades de pointes. Routine administration of sodium bicarbonate and calcium is not recommended unless specific conditions (e.g., electrolyte imbalances, drug toxicities) are present [45–48].

Flush all IV or IO resuscitation drugs with 5–10 mL of normal saline to ensure delivery to the central circulation.

Defibrillation During PALS

Shockable rhythms in children include pulseless ventricular tachycardia (pVT) and ventricular fibrillation (VF). When identified, defibrillation should be performed immediately, regardless of the ECG amplitude. If there is uncertainty about the rhythm, it should be treated as shockable to avoid delays in care. [7].

The preferred method for defibrillation during pediatric ALS is manual defibrillation, but an automated external defibrillator (AED) can be used if manual defibrillation is unavailable. Currently, self-adhesive defibrillation pads are the standard. When using these pads:

  • Chest compressionsshould continue while the defibrillator charges.
  • The pads should be placed in either the antero-lateral (AL)or antero-posterior (AP) positions:
    • AL position: One pad below the right clavicle and the other in the left axilla.
    • AP position: The front pad in the mid-chest just left of the sternum, and the back pad between the scapulae.

Avoid contact between the pads to prevent electrical arcing.

If self-adhesive pads are unavailable, paddles with gel or pre-shaped gel pads can be used as an alternative. In this case, charging should occur directly on the chest, pausing compressions during the process [7]. Pre-planning each step is critical to minimizing delays during the intervention.

Charge the defibrillator for an initial shock of 4J/kg. Avoid exceeding the maximum doses recommended for adults (typically 120–200J, depending on the defibrillator). Pause chest compressions briefly to deliver the shock, ensuring all rescuers are clear of the patient. Resume CPR immediately after the shock, minimizing the pause to under 5 seconds. Reassess the rhythm every 2 minutes and, if it remains shockable, deliver subsequent shocks at 4J/kg. For refractory VF/pVT (requiring more than 5 shocks), incrementally increase the dose up to 8J/kg (maximum 360J) [7].

CPR should continue until an organized, potentially perfusing rhythm is recognized during a rhythm check and is accompanied by signs of ROSC, identified either clinically (e.g., eye-opening, movement, normal breathing) or through monitoring (e.g., etCO2, SpO2, blood pressure, ultrasound) [7].

A summary of the fundamentals of pediatric ALS can be found in Figure 5.

Figure 5. Pediatric cardiac arrest algorithm [7] CPR: cardiopulmonary resuscitation, EMS: emergency medical services, ALS: advanced life support, VF: ventricular fibrillation, pVT: pulseless ventricular tachycardia, PEA: pulseless electrical activity, IV: intravenous, IO: intraosseous.

Post-cardiac Arrest Management

Achieving ROSC is only the first step in resuscitation. Comprehensive post-CA care is crucial to optimizing outcomes, particularly in pediatric patients. This phase focuses on treating the underlying cause of the event and preventing secondary injuries.

Key Components of Post-Cardiac Arrest Care

Targeted Temperature Management (TTM):

  • For unconscious children after ROSC, TTM helps prevent further brain injury.

Ventilation and Oxygenation:

  • Inspired oxygen should be titrated to maintain oxygen saturation (SpO₂) between 94% and 99%.
  • For intubated patients, confirm endotracheal tube (ETT) placement and monitor ventilation to avoid hyperoxia or hypoxia, as well as hypercapnia or hypocapnia.

Hemodynamic Support:

  • Prevent and treat hypotension using parenteral fluids and vasoactive medications, guided by physiologic endpoints and cardiac function.
  • Monitor for signs of recurrent shock and intervene promptly.

Glucose Management:

  • Maintain blood glucose levels below 180 mg/dL to avoid complications associated with hyperglycemia.

Seizure Management:

  • For unconscious children after ROSC, continuous electroencephalography monitoring is also recommended to detect subclinical seizures. Seizures should be monitored and treated aggressively, as they can exacerbate neurological injury.

Temperature Regulation:

  • Avoid hyperthermia (core temperature >37.5°C) using cooling measures as necessary to reduce metabolic demands and limit neuronal damage.

Summary

Pediatric CA remains a critical event with high mortality and significant morbidity. Unlike adult CA, pediatric cases are often precipitated by respiratory failure and hypoxia, highlighting the need for timely recognition and intervention. Early identification of abnormal vital signs, particularly through tools like PEWS, and a structured approach to initial assessment using the ABCDE framework are paramount in preventing CA. Furthermore, rapid and effective resuscitation, incorporating high-quality CPR, advanced airway management, and appropriate medication use, significantly improves the likelihood of survival and favorable outcomes.

Managing pediatric CA extends beyond achieving ROSC. Comprehensive post-cardiac arrest care, including targeted temperature management, optimized ventilation and oxygenation, hemodynamic support, glucose management, and seizure control, is critical to minimize secondary injuries and improve neurological recovery. Pediatric Advanced Life Support (PALS) algorithms and effective resuscitation team dynamics play essential roles in guiding care.

Ultimately, improving outcomes in pediatric CA requires a systematic approach to prevention, timely recognition, prompt intervention, and evidence-based post-resuscitation care. Continuous education, training, and adherence to updated guidelines are essential for healthcare providers to ensure the best possible outcomes for critically ill or arrested children.

Authors

Picture of Burak Çakar

Burak Çakar

Gaziantep Islahiye State Hospital, Department of Emergency Medicine, Gaziantep, Turkey

Picture of Ayça Koca

Ayça Koca

Ayça Koca is an emergency physician at Ankara University School of Medicine, Department of Emergency Medicine. She completed both her medical degree and residency at Ankara University, where she developed a deep connection to patient care and teaching. With a special interest in medical education and simulation, she is passionate about creating engaging learning experiences to support the growth and confidence of future healthcare providers.

Listen to the chapter

References

  1. López-Herce J, Del Castillo J, Matamoros M, et al. Factors associated with mortality in pediatric in-hospital cardiac arrest: a prospective multicenter multinational observational study. Intensive Care Med. 2013;39(2):309-318.
  2. Topjian AA, Raymond TT, Atkins D, et al. Part 4: Pediatric Basic and Advanced Life Support 2020 American Heart Association Guidelines for Cardiopulmonary Resuscitation and Emergency Cardiovascular Care. Pediatrics. 2021;147(Suppl 1):e2020038505D.
  3. Fink EL, Prince DK, Kaltman JR, et al. Unchanged pediatric out-of-hospital cardiac arrest incidence and survival rates with regional variation in North America. Resuscitation. 2016;107:121-128.
  4. Wyckoff MH, Greif R, Morley PT, et al. 2022 International Consensus on Cardiopulmonary Resuscitation and Emergency Cardiovascular Care Science With Treatment Recommendations: Summary From the Basic Life Support; Advanced Life Support; Pediatric Life Support; Neonatal Life Support; Education, Implementation, and Teams; and First Aid Task Forces [published correction appears in Resuscitation. 2024;201:110267.]. Resuscitation. 2022;181:208-288.
  5. Monaghan A. Detecting and managing deterioration in children. Paediatr Nurs. 2005;17(1):32-35.
  6. Halpern NA: Early Warning Systems for Hospitalized Pediatric Patients. Jama 2018, 319(10):981-982.
  7. Van de Voorde P, Turner NM, Djakow J, et al. European Resuscitation Council Guidelines 2021: Paediatric Life Support. Resuscitation. 2021;161:327-387.
  8. Sen IM, Kumar R, Grewal A, Singh M. A simple mnemonic to remember team tasks during cardiopulmonary resuscitation. J Anaesthesiol Clin Pharmacol. 2021;37(3):486-487.
  9. Uzun DD, Lang K, Saur P, Weigand MA, Schmitt FCF. Pediatric cardiopulmonary resuscitation in infant and children with chronic diseases: A simple approach?. Front Pediatr. 2022;10:1065585.
  10. O’Connell KJ, Keane RR, Cochrane NH, et al. Pauses in compressions during pediatric CPR: Opportunities for improving CPR quality. Resuscitation. 2019;145:158-165.
  11. Tibballs J, Weeranatna C. The influence of time on the accuracy of healthcare personnel to diagnose paediatric cardiac arrest by pulse palpation. Resuscitation. 2010;81(6):671-675.
  12. Lubrano R, Cecchetti C, Bellelli E, et al. Comparison of times of intervention during pediatric CPR maneuvers using ABC and CAB sequences: a randomized trial. Resuscitation. 2012;83(12):1473-1477.
  13. Niles DE, Duval-Arnould J, Skellett S, et al. Characterization of Pediatric In-Hospital Cardiopulmonary Resuscitation Quality Metrics Across an International Resuscitation Collaborative. Pediatr Crit Care Med. 2018;19(5):421-432.
  14. Sutton RM, Niles D, Nysaether J, et al. Quantitative analysis of CPR quality during in-hospital resuscitation of older children and adolescents. Pediatrics. 2009;124(2):494-499.
  15. Sutton RM, French B, Nishisaki A, et al. American Heart Association cardiopulmonary resuscitation quality targets are associated with improved arterial blood pressure during pediatric cardiac arrest. Resuscitation. 2013;84(2):168-172.
  16. Sutton RM, Reeder RW, Landis W, et al. Chest compression rates and pediatric in-hospital cardiac arrest survival outcomes. Resuscitation. 2018;130:159-166.
  17. Dezfulian C, Fink EL. How Bad Is It to Fail at Pushing Hard and Fast in Pediatric Cardiopulmonary Resuscitation?. Pediatr Crit Care Med. 2018;19(5):495-496.
  18. Kim MJ, Lee HS, Kim S, Park YS. Optimal chest compression technique for paediatric cardiac arrest victims. Scand J Trauma Resusc Emerg Med. 2015;23:36.
  19. Sutton RM, French B, Niles DE, et al. 2010 American Heart Association recommended compression depths during pediatric in-hospital resuscitations are associated with survival. Resuscitation. 2014;85(9):1179-1184.
  20. Wu ET, Li MJ, Huang SC, et al. Survey of outcome of CPR in pediatric in-hospital cardiac arrest in a medical center in Taiwan. Resuscitation. 2009;80(4):443-448.
  21. Clements F, McGowan J. Finger position for chest compressions in cardiac arrest in infants. Resuscitation. 2000;44(1):43-46.
  22. Finholt DA, Kettrick RG, Wagner HR, Swedlow DB. The heart is under the lower third of the sternum. Implications for external cardiac massage. Am J Dis Child. 1986;140(7):646-649.
  23. Orlowski JP. Optimum position for external cardiac compression in infants and young children. Ann Emerg Med. 1986;15(6):667-673.
  24. Phillips GW, Zideman DA. Relation of infant heart to sternum: its significance in cardiopulmonary resuscitation. Lancet. 1986;1(8488):1024-1025.
  25. Douvanas A, Koulouglioti C, Kalafati M. A comparison between the two methods of chest compression in infant and neonatal resuscitation. A review according to 2010 CPR guidelines. J Matern Fetal Neonatal Med. 2018;31(6):805-816.
  26. Lee JE, Lee J, Oh J, et al. Comparison of two-thumb encircling and two-finger technique during infant cardiopulmonary resuscitation with single rescuer in simulation studies: A systematic review and meta-analysis. Medicine (Baltimore). 2019;98(45):e17853.
  27. Lee SY, Hong JY, Oh JH, Son SH. The superiority of the two-thumb over the two-finger technique for single-rescuer infant cardiopulmonary resuscitation. Eur J Emerg Med. 2018;25(5):372-376.
  28. Pellegrino JL, Bogumil D, Epstein JL, Burke RV. Two-thumb-encircling advantageous for lay responder infant CPR: a randomised manikin study. Arch Dis Child. 2019;104(6):530-534.
  29. Tsou JY, Kao CL, Chang CJ, Tu YF, Su FC, Chi CH. Biomechanics of two-thumb versus two-finger chest compression for cardiopulmonary resuscitation in an infant manikin model. Eur J Emerg Med. 2020;27(2):132-136.
  30. Peska E, Kelly AM, Kerr D, Green D. One-handed versus two-handed chest compressions in paediatric cardio-pulmonary resuscitation. Resuscitation. 2006;71(1):65-69.
  31. Stevenson AG, McGowan J, Evans AL, Graham CA. CPR for children: one hand or two?. Resuscitation. 2005;64(2):205-208.
  32. Beesems SG, Koster RW. Accurate feedback of chest compression depth on a manikin on a soft surface with correction for total body displacement. Resuscitation. 2014;85(11):1439-1443.
  33. Fischer EJ, Mayrand K, Ten Eyck RP. Effect of a backboard on compression depth during cardiac arrest in the ED: a simulation study. Am J Emerg Med. 2016;34(2):274-277.
  34. Ruiz de Gauna S, González-Otero DM, Ruiz J, Gutiérrez JJ, Russell JK. A Feasibility Study for Measuring Accurate Chest Compression Depth and Rate on Soft Surfaces Using Two Accelerometers and Spectral Analysis. Biomed Res Int. 2016;2016:6596040.
  35. Sanri E, Karacabey S. The Impact of Backboard Placement on Chest Compression Quality: A Mannequin Study. Prehosp Disaster Med. 2019;34(2):182-187.
  36. Bhalala US, Hemani M, Shah M, et al. Defining Optimal Head-Tilt Position of Resuscitation in Neonates and Young Infants Using Magnetic Resonance Imaging Data. PLoS One. 2016;11(3):e0151789.
  37. Andersen LW, Raymond TT, Berg RA, et al. Association Between Tracheal Intubation During Pediatric In-Hospital Cardiac Arrest and Survival. JAMA. 2016;316(17):1786-1797.
  38. Hansen ML, Lin A, Eriksson C, et al. A comparison of pediatric airway management techniques during out-of-hospital cardiac arrest using the CARES database. Resuscitation. 2017;120:51-56.
  39. Ohashi-Fukuda N, Fukuda T, Doi K, Morimura N. Effect of prehospital advanced airway management for pediatric out-of-hospital cardiac arrest. Resuscitation. 2017;114:66-72.
  40. Sutton RM, Reeder RW, Landis WP, et al. Ventilation Rates and Pediatric In-Hospital Cardiac Arrest Survival Outcomes. Crit Care Med. 2019;47(11):1627-1636.
  41. Young KD, Korotzer NC. Weight Estimation Methods in Children: A Systematic Review. Ann Emerg Med. 2016;68(4):441-451.e10.
  42. Campbell ME, Byrne PJ. Cardiopulmonary resuscitation and epinephrine infusion in extremely low birth weight infants in the neonatal intensive care unit. J Perinatol. 2004;24(11):691-695.
  43. Holmberg MJ, Ross CE, Atkins DL, et al. Lidocaine versus amiodarone for pediatric in-hospital cardiac arrest: An observational study. Resuscitation. 2020;149:191-201.
  44. Valdes SO, Donoghue AJ, Hoyme DB, et al. Outcomes associated with amiodarone and lidocaine in the treatment of in-hospital pediatric cardiac arrest with pulseless ventricular tachycardia or ventricular fibrillation [published correction appears in Resuscitation. 2019;142:117-118.]. Resuscitation. 2014;85(3):381-386.
  45. Del Castillo J, López-Herce J, Cañadas S, et al. Cardiac arrest and resuscitation in the pediatric intensive care unit: a prospective multicenter multinational study. Resuscitation. 2014;85(10):1380-1386.
  46. Matamoros M, Rodriguez R, Callejas A, et al. In-hospital pediatric cardiac arrest in Honduras. Pediatr Emerg Care. 2015;31(1):31-35.
  47. Wolfe HA, Sutton RM, Reeder RW, et al. Functional outcomes among survivors of pediatric in-hospital cardiac arrest are associated with baseline neurologic and functional status, but not with diastolic blood pressure during CPR. Resuscitation. 2019;143:57-65.
  48. Lasa JJ, Alali A, Minard CG, et al. Cardiopulmonary Resuscitation in the Pediatric Cardiac Catheterization Laboratory: A Report From the American Heart Association’s Get With the Guidelines-Resuscitation Registry. Pediatr Crit Care Med. 2019;20(11):1040-1047.

Reviewed and Edited By

Picture of Elif Dilek Cakal, MD, MMed

Elif Dilek Cakal, MD, MMed

Picture of Arif Alper Cevik, MD, FEMAT, FIFEM

Arif Alper Cevik, MD, FEMAT, FIFEM

Prof Cevik is an Emergency Medicine academician at United Arab Emirates University, interested in international emergency medicine, emergency medicine education, medical education, point of care ultrasound and trauma. He is the founder and director of the International Emergency Medicine Education Project – iem-student.org, chair of the International Federation for Emergency Medicine (IFEM) core curriculum and education committee and board member of the Asian Society for Emergency Medicine and Emirati Board of Emergency Medicine.

The ABCDE Approach to Undifferentiated Critically Ill and Injured Patient (2024)

by Roxanne R. Maria, Hamid A. Chatha

You have a new patient!

A 40-year-old male, a truck driver, is involved in a head-on collision with another vehicle. He has been brought in by ambulance. According to the paramedics, the vehicles were traveling at approximately 85 km/hr, and the patient was restrained by a seatbelt. On arrival at the Emergency Department (ED), the patient is agitated and mildly disoriented. He is tachypneic with a respiratory rate of 30/min, maintaining an O2 saturation of 95% on 12 L/min oxygen via a non-rebreather mask, heart rate of 128 beats/min, blood pressure of 90/52 mmHg, and temperature of 36.1°C. The patient also received 1 L of 0.9% normal saline and 1 unit of O-negative packed red cells in the ambulance. Despite this, his respiratory rate, heart rate, and level of disorientation have worsened.

Emergency Department

In the ED, patients present with a variety of clinical presentations, including both life-threatening and non-life-threatening. Some may have been seen and referred by a clinician before arrival or brought to the department after pre-hospital assessment and care by the emergency medical services (EMS) [1]. Health emergencies affect all age groups and include conditions like acute coronary syndrome, strokes, acute complications of pregnancy, or any chronic illness. Emergency health care providers should respond to these clinically ‘undifferentiated’ patients with symptoms for which the diagnosis may not be known [2].  The root cause of most life-threatening conditions in the ED may be medical or surgical, infection or trauma [2].

In the Emergency Department (ED), there are several potentially life-threatening presentations that demand immediate stabilization. These include trauma, which can result from various forms of accidents or injuries, and shortness of breath, which might indicate critical respiratory distress. An altered mental state also requires prompt attention, as it may signal underlying neurological or systemic issues. Shock, often evidenced by dangerously low blood pressure. Chest pain or discomfort, which could be indicative of a cardiac event, are other urgent concerns. Additionally, cases of poisoning, ingestion of harmful substances, or exposure to toxic materials also necessitate rapid intervention to prevent further harm. Each of these presentations is a medical priority, highlighting the importance of timely and effective response in the ED to ensure patient safety and stability.

These symptoms maybe the only picture that the patients present with, and may constitute the early stage of a critical illness requiring rapid, appropriate intervention and resuscitation, even when the patient seems to appear relatively well [2].

Emergency conditions often require immediate intervention long before a definitive diagnosis is made to stabilize the critically ill patient [3]. Thus, this chapter intends to briefly introduce a basic systematic approach to identifying and managing acute, potentially life-threatening conditions in these patients. This approach will enable all frontline providers, including students, nurses, pre-hospital technicians, and physicians, to manage these patients even in the setting of limited resources [2].

A complete assessment and management of each of the presentations mentioned above is beyond the scope of this chapter. However, the initial approach remains the same, regardless of the patient population or setting [4].

History of the ABCDE approach

The ABC mnemonic’s origins may be traced back to the 1950s. The first two letters of the mnemonic, A and B, resulted from Dr Safar’s description of airway protection techniques and administration of rescue breaths. Kouwenhoven and colleagues later added the letter C to their description of closed-chest cardiac massage [3].

Styner is credited with further developing the Airway, Breathing, Circulation, Disability, and Exposure (ABCDE) approach. After a local aircraft disaster in 1976, Styner and his family were taken to a local healthcare facility, where he saw an insufficiency in the emergency treatment offered. He then founded the Advanced Trauma Life Support course, emphasizing a methodical approach to treating severely injured patients.

The ABCDE approach is universally accepted and utilized by emergency medicine clinicians, technicians, critical care specialists, and traumatologists [3]. Thus, this approach is recommended by international guidelines for suspected serious illness or underlying injury, irrespective of the diagnosis [5]. It is also the first step in post-resuscitation care after the patient achieves return of spontaneous circulation (ROSC) from a cardiac arrest [3]. This systematic approach also aims to improve coordination among the team members and saves time to make critical decisions [3].

The ABCDE approach

Since time is of the essence, the ABCDE method is a systematic approach that can be easily and quickly practiced in the ED. This is incorporated into what is known as ‘Initial patient assessment,’ one of the most crucial steps in evaluation [6]. At each step of this approach, life-threatening problems must be addressed before proceeding to the next assessment step. After the initial assessment, patients must be reassessed regularly to evaluate the treatment response. Anticipate and call for extra help early [7]. Appropriate role allocation and good communication are important for effective team working [7]. Once the patient is stabilized, a secondary survey should be conducted, which includes a thorough history, physical examination, and diagnostic testing [8]. Finally, the tertiary survey is done within 24 hours of presentation to identify any other missed injuries in trauma. Once it is recognized that the patient’s needs exceed the facility’s capabilities, the transfer process must be initiated to an appropriately specialized care center accordingly [8].

Ensure Safe Environment

Before initiating the ABCDE approach, it is essential to ensure both personal safety and a secure environment. This preparation includes addressing any potential risks, such as unexpected or violent behavior, environmental hazards, and the risk of exposure to communicable diseases. Health professionals should consider using appropriate personal protective equipment (PPE) suited to the situation, which may include gloves, gowns, masks, goggles, and thorough hand washing. These precautions are vital to protect both the healthcare provider and the patient, ensuring a safe environment for medical intervention [4].

Initiate First Response

The Resuscitation Council UK (RCUK) (2015) recommends performing a range of initial activities before proceeding with the ABCDE approach [4].

Examine the patient in general (skin color, posture, sensorium, etc.) to determine whether they seem critically ill [4].

After introducing yourself, an initial assessment can be completed in the first 10-15 seconds by asking patients their names and about their active complaints. If they respond normally, it means the airway is patent and brain perfusion is expected [9]. Check for breathing and pulse if the patient appears unconscious or has collapsed. If there is no pulse, call for help and immediately start cardiopulmonary resuscitation (CPR), adhering to local guidelines [9].

Detailed ABCDE Evaluation

Primary Survey

Patients are assessed and prioritized according to their presentations and vital signs. In primary survey, critically ill patients are managed efficiently along with resuscitation. The approach represents the sequence of steps as described below [10]:

A – Airway (with C spine control in Trauma patients)

B – Breathing and Ventilation

C – Circulation (With Hemorrhage control in active bleeding)

D – Disability

E – Exposure / Environment control

A – Airway

Airway obstruction is critical! Gain expert help immediately. If not treated, it can lead to hypoxia, causing damage to the brain, kidneys, and heart, resulting in cardiac arrest and death [4].

Airway management remains the cornerstone of resuscitation and is a specialized skill for the emergency clinician [9].

Assessment of airway patency is the first step. Can the patient talk? If yes, then the airway is patent and not in immediate danger. If not, look for the signs of airway compromise: Noisy breathing, inability to speak, presence of added sounds, stridor or wheezing, choking or gagging, cyanosis, and use of accessory muscles.

The next step is to open the mouth and look for anything obstructing the airway, such as secretions, blood, a foreign body, or mandibular/tracheal/laryngeal fractures [10].

While examining and managing the airway, great care must be taken to restrict excessive movement of the cervical spine and assume the existence of a spinal injury in cases of trauma [11].

Several critical factors can compromise a patient’s airway and must be addressed promptly in emergency settings. A depressed level of consciousness, which may result from conditions such as opioid overdose, head injury, or stroke, can impair airway protection and lead to significant risk [10]. Additionally, an inhaled foreign body, or the presence of blood, vomit, or other secretions, can obstruct the airway and necessitate immediate intervention. Fractures of the facial bones or mandible further complicate airway management due to potential structural damage. Soft tissue swelling, whether caused by anaphylaxis (angioedema) or severe infections like quinsy or necrotizing fasciitis, also seriously threatens the airway. These conditions highlight the importance of vigilant monitoring and rapid response to maintain airway patency and prevent complications.

angioedema - DermNet New Zeeland, CC BY NC ND 3.0
uvula edema - WikiMedia Commons - CC-BY-SA-3.0

Intervention: Several basic maneuvers can help maintain a clear airway. Suctioning should be performed if there are any secretions or blood present. Additionally, using the head-tilt, chin-lift, and jaw-thrust maneuvers can aid in keeping the airway open. For patients with a low Glasgow Coma Scale (GCS) score, placing an oropharyngeal or nasopharyngeal airway can be beneficial in maintaining airway patency. It’s also important to inspect the airway for any obvious obstructions; if a visible object is within reach, it may be removed carefully using a finger sweep or suction. It is crucial to remember that assistance from an anesthetist may be required in some cases. 

Head-Tilt, Chin-Lift maneuver

In trauma patients, to protect the C-spine, perform a jaw-thrust rather than a head-tilt chin-lift maneuver and immobilize the C-spine with a cervical collar [9].

A definitive airway, such as endotracheal intubation, may be necessary in patients with airway obstruction, GCS ≤ 8, severe shock or cardiac arrest, and at risk of inhalation injuries [8].

If intubation has failed or is contraindicated, a definitive airway must be established surgically [11].

B – Breathing and Ventilation

Effective ventilation relies on the proper functioning of the lungs, chest wall, and diaphragm, along with a patent airway and sufficient gas exchange to optimize oxygenation [10]. To assess breathing and ventilation, clinicians should evaluate oxygen saturation, monitor the respiratory rate for any signs of abnormality—such as rapid breathing (tachypnea), slow breathing (bradypnea), or shallow breathing (Kussmaul breathing)—and observe for increased work of breathing, such as accessory muscle use, chest retractions, or nasal flaring. Other critical assessments include checking for neck vein distention, examining the position of the trachea, chest expansion, and any injuries or tenderness, as well as auscultating for bilateral air entry and any additional sounds. Chest percussion should be performed to identify dullness, which may indicate hemothorax or effusion, or hyperresonance, suggestive of pneumothorax. Certain pathologies, like tension pneumothorax, massive hemothorax, open pneumothorax, and tracheal or bronchial injuries, can rapidly disrupt ventilation. Other conditions, including simple pneumothorax, pleural effusion, simple hemothorax, rib fractures, flail chest, and pulmonary contusion, may compromise ventilation to a lesser degree [10].

Interventions:

  • Oxygen – Ensure all patients are adequately oxygenated, with supplemental oxygen delivered to all severely injured trauma patients [11]. Place them on well-fitted oxygen reservoir masks with a flow rate > 10 L/min, which can then be titrated as needed to maintain adequate saturations. Other means of oxygen delivery (nasal catheter, nasal cannula, non-rebreather) can also be used.
  • Bag mask valve ventilation with oxygen – should be given to unconscious patients with abnormal breathing patterns (slow or shallow respiration).
  • Other interventions include salbutamol nebulizers, epinephrine, steroids, needle decompression, chest tube insertion, and the use of noninvasive ventilation and pressure support in different clinical scenarios.

C – Circulation (With Hemorrhage control in active bleeding)

Major circulatory compromise in critically ill patients can result from either blood volume loss or reduced cardiac output. In trauma cases, hypotension is assumed to be due to blood loss until proven otherwise. To assess the hemodynamic status, several key evaluations should be performed. These include checking the level of consciousness, as an altered state may indicate impaired cerebral perfusion, and assessing skin perfusion for signs like pallor, cyanosis, mottling, or flushing. Vital signs such as heart rate and blood pressure should be monitored for abnormalities like tachycardia, bradycardia, hypotension, or hypertension. Auscultation can reveal muffled heart sounds, which may suggest cardiac tamponade or pneumothorax, as well as murmurs or a pericardial friction rub that could indicate pericarditis. Checking the extremities for capillary refill and skin temperature is also essential. Additionally, palpation of the abdomen for tenderness or a pulsatile mass may reveal an abdominal aortic aneurysm, while peripheral edema, such as pedal edema, might indicate heart failure.

Interventions:

  • Two large-bore IV cannulations must be placed. If this attempt fails, intraosseous access is necessary. Hemorrhagic shock—A definitive control of bleeding along with replacement of intravascular volume is essential. Initial resuscitation should start with warm crystalloids, and blood products should be used. Massive Transfusion Protocol (MTP) should be activated according to local guidelines. In hemorrhagic shock, vasopressors and reversal of anticoagulation (if required) can be considered.
  • Hemorrhage control: External hemorrhage can be controlled by direct manual pressure over the site of the wound or tourniquet application.
  • In the case of pelvic or femur fractures, placement of pelvic binders or extremity splints may help to stabilize, although definitive management may be surgical or interventional radiological procedures.
  • Obstructive shock – Immediate pericardiocentesis for cardiac tamponade, chest tube insertion for tension pneumothorax, and thrombolysis for massive pulmonary embolism.
  • Distributive shock – intramuscular epinephrine for anaphylactic shock, empiric antibiotics for sepsis, and hydrocortisone for adrenal crisis.
  • Appropriate antihypertensives in hypertensive emergency.

D – Disability

Evaluate neurological status either with AVPU (Alert, Verbal, Pain and Unresponsive) [5] or GCS (Glasgow Coma Scale).

Evaluate for agitation, head and neck trauma, focal neurological signs (seizure, hemiplegia, etc), lateralizing signs, meningeal signs, signs of raised intracranial pressure, and pupillary examination (size and symmetry). Identify any classic toxidromes (sympathomimetic, cholinergic, anticholinergic, opioid, serotonergic, and sedative-hypnotic toxidromes). 

Choose the best response of patient
EYE OPENING
4: Spontaneously
3: To verbal command
2: To pain
1: No response
BEST VERBAL RESPONSE
5: Oriented and converses
4: Disoriented and converses
3: Inappropriate words; cries
2: Incomprehensible sounds
1: No response
BEST MOTOR RESPONSE
6: Obeys command
5: Localizes pain
4: Flexion withdrawal
3: Flexion abnormal (decorticate)
2: Extension (decerebrate)
1: No response
Glasgow Coma Score (GCS) (Modified from Teasdale, G., & Jennett, B. (1974). Assessment of coma and impaired consciousness: a practical scale. The Lancet, 304(7872), 81-84.) - Please read this article to get more insight regarding GCS.

The Glasgow Coma Scale (GCS) is a critical tool for assessing the level of consciousness in critically ill patients, providing a score based on eye, verbal, and motor responses. A GCS score ranges from 3 to 15, with lower scores indicating more severe impairment. Scores of 13-15 generally indicate mild impairment, 9-12 suggest moderate impairment, and scores of 8 or below (comatose patient) represent severe impairment and a high risk of poor outcomes. In critically ill patients, a declining GCS score can signal worsening neurological status, potentially due to factors like traumatic brain injury, hypoxia, or systemic deterioration, and often warrants immediate intervention to address underlying causes.

E – Exposure and Environmental control

It is necessary to expose the patient appropriately whilst maintaining dignity and body temperature.

Look at the skin for any signs of trauma (burns, stab wounds, gunshot wounds, etc.), rashes, infected wounds, ulcers, needle track marks, medication patches, implantable devices, tubes, catheters, and stomas; measure core body temperature, and perform logroll (trauma).

Do not forget to check frequently concealed and overlooked areas such as the genital, inguinal, perineal, axilla, back and under dressings [8].

Interventions:

  • Use specialized personal protective equipment (PPE), remove all possible triggers such as wet or contaminated clothing, and maintain core body temperature.
  • Minimize hypothermia (external rewarming, warm IV fluids) and hyperthermia (surface cooling, cold IV fluids, antipyretics for fever).

Adjuncts to primary survey

1. Electrocardiography (ECG)
2. Pulse oximetry
3. Carbon dioxide (CO2) monitoring
4. Arterial blood gas (ABG) analysis
5. Urinary catheterization (to assess for hematuria and urine output)
6. Gastric catheterization (for decompression)
7. Blood lactate level measurement
8. Chest and pelvis X-rays
9. Extended focused assessment with sonography for trauma (eFAST)

These adjuncts help provide a comprehensive evaluation of the patient’s condition [10].

Secondary Survey

After the initial primary survey and stabilization, proceed to the secondary survey. This includes a detailed history (SAMPLE)and a head-to-toe examination, including reassessment of vital signs, as there is a potential for missing an injury or other findings in an unresponsive patient [10].

The SAMPLE mnemonic is a structured approach for gathering essential patient history in emergency settings. It stands for Signs and Symptoms, Allergies, Medications, Past Medical History, Last Oral Intake, and Events leading to the illness or injury [5].

  • “Signs and Symptoms” involves asking the patient, family, or other witnesses about any observable signs or reported symptoms.
    “Allergies” are crucial to identify to prevent harm and may help recognize conditions like anaphylaxis.
  • Medications” requires a comprehensive list of all current and recent medications, including any changes in dosage.
  • Past Medical History” provides insights into underlying health conditions that may influence the current illness.
  • Last Oral Intake” is important for assessing risks of aspiration or complications if the patient requires sedation or surgery.
  • Finally, understanding the “Events” surrounding the illness or injury aids in determining its cause and severity.

Together, these components guide healthcare providers in developing a more accurate and effective treatment plan.

In the secondary survey, a thorough approach is taken to ensure comprehensive care for the patient. This includes performing relevant and appropriate diagnostic tests based on the clinical assessment to confirm diagnoses and guide further treatment. Critical, targeted treatments should be initiated promptly, along with adequate supportive care to stabilize the patient’s condition. If necessary, specialized consults are obtained to address specific medical needs. Additionally, the healthcare team must assess the need for escalation of care or consider an interfacility transfer if the patient requires more specialized resources or advanced care options [8]. This structured approach ensures that all aspects of the patient’s condition are managed effectively. 

Adjuncts to secondary survey

Additional x-rays for the spine and extremities, CT scans of the head, chest, abdomen, and spine, urography and angiography with contrast, transesophageal ultrasound, bronchoscopy, and other diagnostics [10].

If the patient starts to deteriorate, immediately go back to the ABCDE approach and reassess!

Special Patient Groups

In recent ATLS updates, the ABCDE approach has been modified to the xABCDEF approach, where “x” stands for eXsanguinating eXternal hemorrhage control and “F” stands for further factors such as special groups (pediatric, Geriatric, and Pregnancy).  While the xABCDEF approach is universal and applies to all patient groups, specific anatomic and physiological differences in different populations should be considered while evaluating and treating life-threatening conditions. Some special population groups are discussed here:

Pediatrics [10]

Children have smaller body mass but higher body surface area than their body mass and proportionately larger heads than adults. These characteristics cause children to have increased energy transfer, hypothermia, and blunt brain trauma.

A useful adjunct is the Broselow® Pediatric Emergency Tape, which helps to rapidly identify weight-based medication doses, fluid volumes, and equipment sizes.

The ABCDE approach in children should proceed in the same manner as in adults, bearing in mind the anatomical differences.

Airway – Various anatomical features in children, such as large tissues of the oropharynx (tongue, tonsils), funnel-shaped larynx, more cephalad and anteriorly placed larynx and vocal cords, and shorter length of the trachea, make assessment and management of the airway difficult. Additionally, in smaller children, there is disproportionality in size between the cranium and the midface, making the large occiput in passive flexion of the cervical spine, resulting in the posterior pharynx being displaced anteriorly. The neutral alignment of the spine can be achieved by placing a 1-inch pad below the entire torso of the infant or toddler.

The most preferred technique for orotracheal intubation is under direct vision, along with restriction of the cervical spine, to achieve a definitive airway.

Infants are more prone to bradycardia due to laryngeal stimulation during intubation than older children and adults. Hence, when drug-assisted intubation is required, the administration of atropine sulfate pretreatment must be considered. Atropine also helps to dry out oral secretions, further enhancing the view of landmarks for intubation.

When the airway cannot be maintained by bag-mask ventilation or orotracheal intubation, a rescue airway with either a laryngeal mask airway (LMA), an intubating LMA, or a needle cricothyroidotomy is required.

Red flag signs in children include stridor, excessive drooling, airway swelling, and the child’s unwillingness to move the neck. Examine the airway carefully for any foreign bodies, burns, or obstruction.

Breathing and ventilation – Children’s respiratory rates decrease with age. The normal tidal volumes in infants and children vary from 4-6 ml/kg to 6-8 ml/kg while assisting in ventilation. Care must be taken to limit pressure-related barotrauma during ventilation. It is recommended that children weighing less than 30 kg use a pediatric bag valve mask.

Injuries such as pneumothorax, hemothorax, and hemopneumothorax should be treated by pleural decompression, for tension pneumothorax, and needle decompression in the 2nd intercostal space (over the top of the third rib) at the midclavicular line. The site for chest tube insertion remains the same as in adults.

The most common cause of pediatric cardiac arrest is hypoxia, and the most common acid-base abnormality encountered is respiratory acidosis due to hypoventilation.

Circulation – Important factors in assessing and managing circulation and shock are looking for signs of circulatory compromise, ascertaining the patient’s weight and circulatory volume, gaining timely peripheral venous access, delivering an appropriate volume of fluids with or without blood replacement, evaluating the adequacy of resuscitation, and aiming for thermoregulation.

Children have increased physiological reserves. A 30% decrease in the circulating blood volume may be required for a fall in the systolic blood pressure. Hence, it is important to look for other subtle signs of blood loss, such as progressive weakening of peripheral pulses, narrow pulse pressure to less than 20 mm Hg, skin mottling (in infants and young children), cool extremities, and decreased level of consciousness.

The preferred route is peripheral venous access, but if this is unsuccessful after two attempts, intraosseous access should be obtained.

Fluid resuscitation must be commenced at 20 ml/kg boluses of isotonic crystalloids. If the patient has ongoing bleeding, packed red blood cells may be initiated at 10 ml/kg as soon as possible. Given that children have increased metabolic rates, thinner skin, and lack of substantial subcutaneous tissue, they are prone to develop hypothermia quickly, which may impede a child’s response to treatment, increase coagulation times, and affect the central nervous system (CNS) function. Therefore, overhead lamps, thermal blankets, as well as administration of warm IV fluids, blood products, and inhaled gases may be required during the initial phase of evaluation and resuscitation.

Disability – Hypoglycemia is a very common cause of altered mental state in children, and children can present with altered mental state or seizures. Check for blood glucose in children; if low, administer glucose (IV D10 or D25).

Geriatric [10]

In cases of trauma in geriatric patients, physiological events that may have led to it (e.g., cardiac dysrhythmias) must be considered. A detailed review of long-term medical conditions and medications, along with their effect on vital signs, is necessary. Risk factors for falls include physical impairments, long-term medication use, dementia, and visual, cognitive, or neurological impairments.

Elderly patients are more prone to sustaining burn injuries due to decreased reaction times, hearing and visual impairment, and inability to escape the burning structure. Burn injury remains the cause of significant mortality.

AirwayDue to loss of protective airway reflexes, airway management in the elderly can be challenging and requires a timely decision to establish a life-saving definitive airway. Opening of the mouth and cervical spine maneuvering may be challenging with arthritic changes. Loose dentures should be removed, while well-fitted dentures should be better left inside. Some patients may be edentulous, making intubation easier, but bag-mask ventilation is difficult.

While performing rapid sequence intubation, it is recommended to lower the doses of barbiturates, benzodiazepine, and other sedatives to 20% to 40% to avoid the risk of cardiovascular depression.

Breathing – Elderly patients have decreased compliance of the lungs and the chest wall, which leads to increased breathing work, placing them at a higher risk for respiratory failure. Aging also results in suppressed heart rate during hypoxia, and respiratory failure may present alongside.

Circulation – These patients may have increasing systemic vascular resistance in response to hypovolemia, given that they may have a fixed heart rate and cardiac output. Also, an acceptable blood pressure reading may truly indicate a hypotensive state, as most elderly patients have preexisting hypertension.

A systolic blood pressure of 110 mm Hg is used as a threshold for identifying hypotension in adults over 65.

Several variables, namely base deficit, serum lactate, shock index, and tissue-specific lab markers, can be used to assess for hypoperfusion. Consider early use of advanced monitoring of fluid status, such as central venous pressure (CVP), echocardiography, and bedside ultrasonography, to guide resuscitation.

Disability – Traumatic brain injury is one of the significant complications among the elderly. The dura becomes more adherent to the skull with age, which increases the risk of epidural hematoma. Moreover, these patients are commonly prescribed anticoagulant and antiplatelet medications, which puts these individuals at a higher risk of developing intracranial hemorrhage. Therefore, a very low threshold is indicated for further CT scan imaging in ruling out acute intracranial and spinal pathologies.

Exposure – Increased risk of hypothermia due to loss of subcutaneous fat, nutritional deficiencies, chronic medical illnesses, and therapies. Complications of immobility, such as pressure injuries and delirium, may develop.

Rapid evaluation and relieving from spine boards and cervical collars will help to reduce these injuries.

Pregnant [10]

Evaluation and management of pregnant individuals can be challenging due to the physiological and anatomical changes that affect nearly every organ system in the body. Therefore, knowledge of the physiological and anatomical changes during pregnancy regarding the mother and the fetus is important to provide the best and most appropriate resuscitation and care for both.  

The best initial treatment for the fetus is by providing optimal resuscitation of the mother.

Female patients in the reproductive age who present to the ED must be considered pregnant until proven by a definitive pregnancy test or ultrasound exam.

A specialized obstetrician and surgeon should be consulted early in the assessment of pregnant trauma patients; if not available, early transfer to an appropriate facility should be sought.

The uterus is an intrapelvic organ until the 12th week of gestation, around 34 to 36 weeks when it rises to the level of the costal margin. This makes the uterus and its contents more susceptible to blunt abdominal trauma, whereas the bowel remains somewhat preserved. Nevertheless, penetrating upper abdominal trauma in the late gestational period can cause complex intestinal injury due to displacement.

Amniotic fluid embolism and disseminated intravascular coagulation are significant complications of trauma in pregnancy. In the vertex presentation, the fetal head lies in the pelvis, and any fracture of the pelvis can result in fetal skull fracture or intracranial injury.

A sudden decrease in maternal intravascular volume can lead to a profound increase in uterine vascular resistance, thus reducing fetal oxygenation regardless of normal maternal vital signs.

The volume of plasma increases throughout pregnancy and peaks by 34 weeks of gestation. Physiological anemia of pregnancy occurs when there is an increase in red blood cell (RBC) volume, leading to decreased hematocrit levels. In normal, healthy pregnant individuals, blood loss of 1200 to 1500 ml can occur without showing any signs or symptoms of hypovolemia. Nonetheless, this compromise may be seen as fetal distress, indicated by an abnormal fetal heart rate on monitoring.

Leukocytosis is expected during pregnancy, peaking up to 25,000/mm3 during labor. Serum fibrinogen and other clotting factors may be mildly increased, with shorter prothrombin and partial thromboplastin times. However, bleeding and clotting times remain the same.

During late pregnancy, in a supine position, vena cava compression can cause a decrease in cardiac output by 30 % due to lesser venous return from the lower extremities.

In the third trimester of pregnancy, heart rate increases up to 10-15 beats/min than the baseline while assessing for tachycardia in response to hypovolemia. Hypertension, along with proteinuria, indicates the need to manage preeclampsia. Be mindful of eclampsia as a complication during late pregnancy, as its presentation can be similar to a head injury (seizures with hypertension, hyperreflexia, proteinuria, and peripheral edema)

An increase in the tidal volume causes increases in the minute ventilation and hypocapnia (PaCO2 of 30 mm Hg), which is common in the later gestational period. Therefore,

Maintaining adequate arterial oxygenation during resuscitation as oxygen consumption increases during pregnancy is also important.

By the seventh month of gestation, the symphysis pubis widens to about 4 to 8 mm, and sacroiliac joint spaces increase. These alterations must be kept in mind while evaluating pelvic X-ray films during trauma. Additionally, the pelvic vessels that surround the gravid uterus can become engorged, leading to large retroperitoneal hemorrhage after blunt trauma with pelvic fractures.

Every pregnant patient who has sustained major trauma must be admitted with appropriate obstetric and trauma facilities.

Pregnant individuals may present to the ED with non-obstetric causes such as intentional (intimate partner violence, suicide attempt) and unintentional trauma (MVC, fall), and obstetric causes such as ectopic pregnancy, vaginal bleed, contractions, abdominal pain, decreased fetal movement, etc.

“To optimize outcomes for the mother and fetus, assessment and resuscitation of the mother is performed first and then the fetus, before proceeding for secondary survey of the mother.”

Primary Survey - Mother

Airway – Ensure the patient has a patent and maintainable airway with adequate ventilation. In cases where intubation is necessary, maintain appropriate PaCO2 levels according to the patient’s gestational age.  Due to the superior displacement of abdominal organs and delayed gastric emptying, there is an increased risk of aspiration during intubation.

BreathingThese patients may have an increased rate of respiration due to pressure effects or hormonal changes. Pulse oximetry and arterial gas must be monitored as adjuncts. It must be remembered that normal maternal bicarbonate levels will be low to compensate for the respiratory alkalosis.

Circulation – Attempt to manually reposition the uterus towards the left side to relieve the pressure on the inferior vena cava and improve the venous return.

Since pregnant individuals have increased intravascular volumes, they can lose a large amount of blood before the onset of tachycardia, hypotension, or other signs of hypovolemia. Therefore, it is essential to remember that the fetus and the placenta are deprived of perfusion, leading to fetal distress while the maternal conditions appear stable.

Administer crystalloid IV fluids and type-specific blood. Vasopressors must be used only as a last resort to raise maternal blood pressure, as these agents can further cause a reduction of the uterine blood flow, leading to fetal hypoxia.

Primary Survey - Fetus

Leading causes of fetal demise include maternal shock and death, followed by placental abruption.

Assess for signs of abruptio placentae (vaginal bleeding, uterine tenderness, frequent uterine contractions, uterine tetany, and irritability). Another rare injury is the uterine rupture (abdominal tenderness, rigidity, guarding or rebound tenderness, abnormal fetal lie, etc.) accompanying hypovolemia and shock.

By 10 weeks of gestation, fetal heart tones can be assessed by Doppler ultrasound, and beyond 20-24 weeks of gestation, continuous fetal monitoring with a tocodynamometer must be performed. At least 6 hours of continuous monitoring in patients with no risk factors for fetal death is recommended, and 24 hours of monitoring in patients with a high risk of fetal death.

Secondary Survey

Perform the secondary survey for non-pregnant individuals, as mentioned.

An obstetrician should ideally examine the perineum, including the pelvis. The presence of amniotic fluid in the vagina, PH greater than 4.5, indicates chorioamniotic membrane rupture.

All pregnant patients with vaginal bleeding, uterine irritability, abdominal tenderness and pain, signs and symptoms of shock, fetal distress, and leakage of amniotic fluid should be admitted for further care.

All pregnant trauma patients with Rh-negative blood group must receive Rh immunoglobulin therapy unless the injury is remote from the uterus within 72 hours of injury.

Obese Patients [10]

In the setting of trauma, procedures such as intubation can be challenging and dangerous due to their anatomy. Diagnostic investigations such as E-FAST, DPL, and CT scans may also be challenging. Moreover, most of these patients have underlying cardiopulmonary diseases, which hinders their ability to compensate for the stress and injury.

Athletes [10]

Owing to their prime conditioning, they may not exhibit early signs such as tachycardia or tachypnea in shock cases. Additionally, they usually have low systolic and diastolic blood pressure.

Revisiting Your Patient

Let’s get back to the patient we discussed earlier and start assessing him:

Airway – The patient maintains his airway but finds breathing hard. Intervention: Apply 15L Oxygen via a nonrebreather mask.

Breathing—A strap mark contusion is seen with multiple bruises. His chest expansion is asymmetrical, with reduced breath sounds on the right side of his chest. There is a dull percussion note on the right lower half of his chest. He maintains oxygen saturation. Intervention: Prepare for chest tube insertion on the right side.

Circulation – Heart sounds are muffled with marked engorgement of the external jugular veins in the neck, a good pulse still palpable in his left radial, but cold clammy extremities. His pulse is 128/min, and his blood pressure is 92/50 mm Hg. Bedside ultrasound FAST (Focused Assessment Sonography in Trauma) shows a pericardial tamponade. Intervention: IV access was gained with two large-bore IV cannulas, blood was drawn for labs, the massive transfusion protocol for blood products was activated, a Foley catheter was inserted to monitor urinary output, and the surgery team was on board to plan for emergent pericardiocentesis.

Disability – Patient’s GCS remains 15, unremarkable pupillary examination and POC glucose is 7 mmol/dl.

Exposure – you notice the strap mark on his chest secondary to his seatbelt restraint, and the multiple bruises. The remaining evaluation is unremarkable, with no head, spine, abdomen, or limb injury.

Adjunct investigations – A portable chest x-ray shows increased cardiac shadow and multiple bilateral rib fractures. There is opacification in the right lung [12]. 

Discussion

This patient sustained a blunt trauma leading to pericardial tamponade and right-sided hemothorax, leading to hypovolemic shock. The most common cause of shock in a trauma patient is hypovolemic shock due to hemorrhage. However, other types of shock like cardiogenic shock (due to myocardial dysfunction), neurogenic shock (due to sympathetic dysfunction), or obstructive shock (due to tension pneumothorax, obstruction of great vessels) can occur.

Early signs of shock include tachycardia, which is the body’s attempt to preserve cardiac output and cool peripheries, and reduced capillary refill time caused by peripheral vasoconstriction. This is caused by the release of catecholamine and vasoactive hormone, which leads to increased diastolic blood pressure and reduced pulse pressure. For this reason, measuring pulse pressure rather than systolic blood pressure allows earlier detection of hypovolaemic shock, as the body can lose up to 30% of its blood volume before a drop in systolic blood pressure is appreciated.

Initiate fluid resuscitation in these patients and do not wait for them to develop hypotension.
The main aim is to maintain organ perfusion and tissue oxygenation. In children, start with crystalloid fluid boluses of 20 ml/kg, and in adults, an initial 1 L can be given. In patients who have sustained a major blood loss, consider initiating the Massive Transfusion Protocol (MTP) for blood products as soon as possible.

A few current trauma guidelines have recommended ‘permissive hypotension’ or ‘balanced resuscitation,’ where the principle is to stabilize any blood clots that may have been formed, and aggressive blood pressure resuscitation may disrupt this ‘first formed clot’ and may contribute to further hemorrhage.

To evaluate response to fluid resuscitation, assess the level of consciousness, improvement in tachycardia, skin temperature, capillary refill, and urine output (>0.5 ml/kg/hour in adults).
Besides administering packed red blood cells, do not forget to transfuse platelets, fresh frozen plasma, or cryoprecipitate, as large blood loss can develop coagulopathy in 30% of these injured patients. Tranexamic acid (TXA), an antifibrinolytic, can be utilized in addition as a 1 g bolus over 10 minutes followed by 1 g over 8 hours within 3 hours of trauma without an increased risk of thromboembolic events [11].

This systematic approach focuses on identifying and treating this hemorrhagic shock case. Bedside adjuncts such as FAST examination and portable chest X-ray can provide valuable clues to the cause of shock. A trauma CT scan is only performed once the patient is stable enough to go to the scan room.

This patient’s vital signs improve slightly but remain unstable, and blood is kept draining into the chest drain. The patient is taken to the operation theatre for an emergency thoracotomy [12].

Authors

Picture of Roxanne R. Maria

Roxanne R. Maria

Picture of Hamid A. Chatha

Hamid A. Chatha

Listen to the chapter

References

  1. Initial Assessment of Emergency Department patients, The Royal College of Emergency Medicine, Feb 2017
  2. World Health Organization. BASIC EMERGENCY CARE : Approach to the Acutely Ill and Injured.World Health Organization; 2018.
  3. Thim T. Initial assessment and treatment with the airway, breathing, circulation, disability, exposure (ABCDE) approach. International Journal of General Medicine. 2012;5(5):117-121. doi:https://doi.org/10.2147/IJGM.S28478
  4. Peate I, Brent D. Using the ABCDE Approach for All Critically Unwell Patients. British Journal of Healthcare Assistants. 2021;15(2):84-89. doi:https://doi.org/10.12968/bjha.2021.15.2.84
  5. Schoeber NHC, Linders M, Binkhorst M, et al. Healthcare professionals’ knowledge of the systematic ABCDE approach: a cross-sectional study. BMC Emergency Medicine. 2022;22(1). doi:https://doi.org/10.1186/s12873-022-00753-y
  6. Learning Objectives. https://www.moh.gov.bt/wp-content/uploads/moh-files/2017/10/Chapter-2-Emergency-Patient-Assessment.pdf
  7. Resuscitation Council UK. The ABCDE Approach. Resuscitation Council UK. Published 2021. https://www.resus.org.uk/library/abcde-approach#:~:text=Use%20the%20Airway%2C%20Breathing%2C%20Circulation
  8. Management of trauma patients – Knowledge @ AMBOSS. http://www.amboss.com. https://www.amboss.com/us/knowledge/Management_of_trauma_patients/
  9. Oxford Medical Education. ABCDE assessment. Oxford Medical Education. Published 2016. https://oxfordmedicaleducation.com/emergency-medicine/abcde-assessment/
  10. HENRY SM. ATLS Advanced Trauma Life Support 10th Edition Student Course Manual, 10e. 10th ed. AMERICAN COLLEGE OFSURGEO; 2018.
  11. Walls RM, Hockberger RS, Gausche-Hill M, Erickson TB, Wilcox SR. Rosen’s Emergency Medicine : Concepts and Clinical Practice. Elsevier; 2.
  12. Eamon Shamil, Ravi P, Mistry D. 100 Cases in Emergency Medicine and Critical Care. CRC Press; 2018.

Reviewed By

Picture of Arif Alper Cevik, MD, FEMAT, FIFEM

Arif Alper Cevik, MD, FEMAT, FIFEM

Prof Cevik is an Emergency Medicine academician at United Arab Emirates University, interested in international emergency medicine, emergency medicine education, medical education, point of care ultrasound and trauma. He is the founder and director of the International Emergency Medicine Education Project – iem-student.org, chair of the International Federation for Emergency Medicine (IFEM) core curriculum and education committee and board member of the Asian Society for Emergency Medicine and Emirati Board of Emergency Medicine.

Maxillofacial Trauma (2024)

by Maitha Ahmad Kazim & David O. Alao

You have a new patient!

A 48-year-old man was brought to the ED by ambulance shortly after sustaining blunt trauma to the face. The patient was off-loading his quad bike from a truck when it accidentally flipped over and fell directly on his face. He could not recall the incident.

Upon arrival, his vitals were BP: 144/85 mmHg, HR: 104 bpm, T: 36.8°C, RR: 23 bpm, and SPO2: 99% on room air. He was awake on the AVPU score. On examination, the patient was bleeding profusely from his nostrils, breathing from his mouth, and having diffuse facial swelling. You are concerned about the extent of the injuries sustained, and you assemble a team to manage the patient.

Importance

The significance of proficiently managing maxillofacial trauma in the fast-paced emergency medicine setting cannot be overstated. Not only do these traumas cause direct physical harm, but they also impact the patient’s appearance and their ability to perform vital functions like breathing, speaking, and chewing. Given the complex and sensitive nature of the maxillofacial region, emergency physicians must comprehensively understand how to manage such injuries effectively. Proficiency in diagnosing and managing maxillofacial trauma ensures timely and appropriate treatment and prevents potential complications and long-term sequelae. 

Epidemiology

Maxillofacial injuries are a prevalent global health concern. There were an estimated 7.5 million new facial fractures globally in 2017, with 1.8 million individuals living with a disability from a facial fracture [1]. Undoubtedly, the incidence and prevalence vary significantly from one country to another. Singaram et al. reported that the prevalence varied between countries from 17% to 69% [2]. In many regions, inadequate infrastructure, limited access to healthcare, and poor safety regulations contribute to a higher incidence of maxillofacial injuries.

Pathophysiology

Road traffic accidents, interpersonal violence, industrial accidents, and sports-related incidents are the most common etiologies of maxillofacial injuries globally. However, the predominant causes differ in developed and developing countries. Assault is the most common mechanism of injury in developed countries, while motor vehicle accident (MVA) is the most common mechanism in developing countries [3].

Low or high-impact forces can cause maxillofacial injuries. The force needed to cause damage differs from one bone to another. For instance, the zygoma and nasal bones can be damaged by low-impact forces. In contrast, the frontal bone, supraorbital rim, maxilla, and mandible are damaged by high-impact forces [4].

Furthermore, the etiology of maxillofacial trauma can predict the type of facial injuries and fractures sustained. For example, MVAs have been associated with higher instances of mandibular fractures. That is mainly due to its position compared to the rest of the facial bones and its relatively thin structure [5].

Medical History

Maxillofacial injuries often occur in association with other injuries and, thus, can be missed initially. Obtaining a systemic and thorough history can aid the diagnosis. At the initial presentation, the mnemonic “AMPLE” (Allergies, Medications currently used, Past illness/Pregnancy, Last meal, Events/Environment related to the injury) can be used to assess the patient’s pre-injury health status. Then, the following should be probed:

  • What was the mechanism of injury?
    Understanding the cause of the injury (e.g., fall, vehicle collision, assault) provides insights into potential injuries and the extent of trauma. Different mechanisms (blunt vs. penetrating, low vs. high-impact) influence the pattern and severity of injuries and aid in anticipating associated injuries.

    • Environment related to the injury
      Environmental context (e.g., construction site, sports field) can highlight additional risk factors or clues about the nature and potential complications of the injury. It may also help assess the likelihood of secondary injuries or infections.

    • Blunt vs. penetrative
      The type of trauma affects the damage pattern. Blunt trauma may result in fractures or soft tissue injuries, while penetrating trauma may involve more focal injury with a higher risk of infection and internal damage.

    • Low vs. high-impact force
      High-impact injuries are more likely to cause fractures and significant soft tissue damage. Knowing the force helps anticipate the severity and depth of injuries.

    • Direction of force
      The direction can indicate which structures might be compromised (e.g., anterior force could affect the nose, mandible, and dental structures, while lateral force may impact the zygomatic arch or TMJ).

  • Was there a loss of consciousness or an altered level of consciousness?
    Altered consciousness or loss of consciousness may indicate a head injury or neurological involvement, which necessitates further investigation and monitoring for brain injury.

  • Are there any visual disturbances?
    Vision changes can signal orbital fractures or injuries to the optic nerve, potentially affecting ocular function or indicating damage to the orbit and nearby structures.

  • Is there any change in hearing? Is the patient experiencing tinnitus or vertigo? Did they notice any discharge from the ears (clear or bloody)?
    Hearing changes, tinnitus, vertigo, or ear discharge suggest possible basilar skull fractures or damage to the auditory system, which are essential to identify to avoid long-term complications.

  • Any trouble breathing through the nose? Did they notice any discharge (clear or bloody)?
    Difficulty breathing through the nose or nasal discharge may indicate nasal fractures, airway obstruction, or cerebrospinal fluid (CSF) leakage if clear, which is critical to address in traumatic injuries.

  • Any pain while talking? Do the teeth come together normally?
    Pain when speaking or abnormal occlusion may signal fractures in the mandible, maxilla, or TMJ dislocation, impacting facial symmetry, function, and long-term outcomes.

  • Is there difficulty opening or closing the mouth? Is there any pain when biting down the teeth?
    Difficulty or pain in mouth movement often suggests mandibular fractures or TMJ injury. Restricted movement can help identify specific injury locations and aid in planning management.

  • Numbness or tingling sensation in any area of the face?
    Sensory changes suggest possible nerve damage, often related to fractures affecting the infraorbital, mental, or other facial nerves. This information helps predict potential complications and guides treatment planning.

Consider the following symptoms when obtaining a history from maxillofacial trauma patients:

  • Orbital floor fractures commonly present with symptoms such as tingling or numbness around the nose, upper lip, and maxillary gums due to infraorbital nerve damage, along with difficulty looking upward or laterally, double vision (diplopia), and pain during eye movement.
  • Nasal fractures are characterized by swelling, pain, and nosebleeds (epistaxis).
  • Nasoethmoidal fractures can cause cerebrospinal fluid (CSF) rhinorrhea, epistaxis, and tearing (epiphora) due to nasolacrimal duct obstruction.
  • Zygomaticomaxillary complex (ZMC) fractures may lead to numbness around the nose and upper lip, issues with eye movement, double vision, and difficulty opening the mouth (trismus).
  • Maxillary fractures often result in CSF rhinorrhea or epistaxis and may cause mobility in the upper teeth and gingiva.
  • Alveolar fractures are typically associated with gingival bleeding.
  • Mandibular fractures can present as painful jaw movements and tingling or numbness affecting half of the lower lip, chin, teeth, and gingiva.

Red Flags in History

Due to the complex nature of the maxillofacial region, one should be vigilant for red flags when taking history from the patients. Its proximity to the brain and central nervous system makes injuries to these very likely. Thus, identifying them early on can prevent irreversible sequelae and medicolegal implications. Red flags include memory loss, fluctuations in the level of consciousness, nausea/vomiting, and headache that does not improve with analgesia [6].

Neurological involvement can further be assessed by asking about the presence of diplopia or a change in visual acuity. Vision loss usually occurs immediately, but in 10%, symptoms are delayed [7]. Another red flag that is associated with high morbidity and mortality is cervical cord syndrome. Maxillofacial injuries associated with falls are often associated with cervical spinal injury. The patient may complain initially about neck pain or a loss of motor/sensory function in the arms [8].

Physical Examination

Maxillofacial trauma is commonly associated with polytrauma [9]. Thus, it often gets missed in examinations. Physical examination should be done systematically to ensure that all injuries are noted. Like all trauma cases, life-threatening injuries should be addressed first, and the ATLS protocol should be applied accordingly. After that, a physical examination of maxillofacial trauma would involve several key steps. Hard and soft tissue injuries (hematoma, laceration, foreign body, swelling, missing tissue, bleeding, or clear discharge) should be noted upon general inspection of the head and face. Symmetry and alignment of the face should also be noted, bearing in mind that asymmetry may be hidden by edema [10]. Facial elongation and flattening can be seen in midface fractures. Increased intercanthal distance, also known as telecanthus, indicates a nasoethmoidal injury.

Palpation of the whole face should follow, going from top to bottom to avoid missing any injury. Identify step-offs, crepitus, instability or excessive mobility, and malocclusion. Le Fort fractures, complex midface fractures, can be identified during physical examination. 

Next, a complete ocular examination should be done. Assess visual acuity, visual field, pupillary reflex, anterior chamber, and extraocular movements. An ophthalmologic consultation is recommended if any abnormalities are present [10]. The nose and septum should be inspected for any hematoma, bulging mass, or CSF leakage and palpated for any signs of fracture. The oral cavity should be inspected for palatal ecchymoses, lacerations, malocclusion, or missing teeth. Manipulate each tooth individually for movement or pain. Palpate the entire mandible for step-offs or injury. Motor and sensory functions of the face should be evaluated. A thorough cranial nerve examination will help identify sensorimotor injuries. 

Le Fort Classification

Le Fort I Fracture: A Le Fort I fracture, often referred to as a “floating palate,” is a horizontal maxillary fracture that separates the teeth from the upper face. The fracture line passes through the alveolar ridge, lateral nose, and the inferior wall of the maxillary sinus. Patients with this fracture often present with a swollen upper lip, open bite malocclusion, and ecchymosis of the hard palate. When the forehead is stabilized, the maxilla may also have noticeable mobility (including the hard palate and teeth).

Le Fort II Fracture: Known as a “floating maxilla,” the Le Fort II fracture builds upon the characteristics of Le Fort I but extends to involve the bony nasal skeleton, giving it a pyramidal shape. This fracture often leads to a widening of the intercanthal space, bilateral raccoon eyes, epistaxis, and open bite malocclusion. Physical examination may reveal mobility of the maxilla and nose, ecchymosis of the hard palate, and cerebrospinal fluid (CSF) rhinorrhea. Patients may also experience sensory deficits in the infraorbital region extending to the upper lip.

Le Fort III Fracture: Referred to as a “floating face” or “craniofacial disjunction,” the Le Fort III fracture involves a separation of the midfacial skeleton from the base of the skull. The fracture line extends from the frontozygomatic suture across the orbit and through the base of the nose and ethmoid region, running parallel with the skull base. Physical signs include bilateral raccoon eyes, ecchymosis of the hard palate, and a dish-face deformity characterized by elongation and flattening of the face. Additional signs may include enophthalmos (sunken eyes), Battle’s sign (ecchymosis over the mastoid bone), CSF rhinorrhea or otorrhea, and hemotympanum.

Red Flags in Examination

Look for “red flags” during physical examination. These red flags include cervical spine injuries, loss of teeth, Battle’s sign/Raccoon eyes with CSF rhinorrhea, and Le Fort fractures. Facial bones should not be manipulated until cervical spine injuries, which are present in 2.2% of cases, have been ruled out [11]. The oral cavity should be carefully examined for loss of teeth, as it may be aspirated during the injury. For missing teeth, a chest X-ray should be done to rule out or confirm aspiration.

Moreover, facial fractures can extend to the cranium [4]. Depending on the mechanism of injury, the patient may suffer from a concomitant base of the skull fracture, which may present with Battle’s sign and Raccoon eyes as well as CSF rhinorrhea in some cases [11]. LeFort fractures are complex fractures of the midface and are further classified into LeFort I, II, and III. These fractures are considered a red flag as they may cause airway obstruction and life-threatening bleeding [12].

Alternative Diagnoses

Given that the cause is usually known, doctors must identify the injuries sustained and the extent of injuries sustained. While blunt trauma to the face is an apparent cause of maxillofacial injuries, concomitant and alternative diagnoses should not be missed. Patients with maxillofacial trauma can present with a wide range of symptoms that are similar to those from intracranial and cervical spinal injuries.

Acing Diagnostic Testing

The diagnosis of maxillofacial injuries is not based on a single diagnostic test. It is a correlation between history, physical examination, and imaging studies. Given that the etiologies of the injury vary, the differentials are vast, and the clinical presentation differs from one patient to another. Thus, bedside testing and laboratory studies should be tailored to each patient’s clinical presentation and existing symptoms.

Bedside Testing

ECG monitoring is essential for all trauma patients. Dysrhythmias, atrial fibrillation, and ST segment changes can be seen in blunt cardiac injury. Point-of-care (POCT) glucose testing quickly assesses the patient’s glucose level. Hypoglycemia can cause confusion and an altered mental status, which are common findings in patients with maxillofacial trauma. Point-of-care blood gas testing may be beneficial in case of excessive bleeding or airway compromise. In case of tissue hypoperfusion and shock, metabolic acidosis and elevated lactate levels may be noted. Oxygen saturation and carbon dioxide should be monitored in case of midface fractures and suspected airway compromise. A POCT pregnancy test should be done in women of childbearing age, as almost all maxillofacial trauma patients require imaging for diagnosis.

Laboratory Testing

A complete blood count (CBC), particularly hemoglobin and hematocrit, is indicated when the patient is bleeding profusely. LeFort II and III have been associated with an increased risk of life-threatening hemorrhage compared to other facial fractures [12]. Therefore, blood typing and crossmatching are crucial if the patient needs a blood transfusion. A coagulation panel is done to rule out trauma-induced coagulopathy, a preventable factor for progressive brain injury and massive bleeding [13].

A CSF analysis is warranted when there are secretions from the nose or ear. Beta-2 transferrin testing is the current preferred test to confirm the presence of a CSF leak [14]. Other less used methods include beta-trace protein, double-ring sign, and glucose oxidase test. A blood ethanol test and urine toxicology screen can be considered in agitated patients or those with altered levels of consciousness.

Imaging Studies

CT scans are the “gold standard” diagnostic modality for evaluating maxillofacial trauma [15]. Using narrow-cut CT scans without contrast provides detailed cross-sectional images of the facial structures, thus allowing for a comprehensive evaluation of complex fractures. In addition to identifying facial fractures, it can detect head and cervical spinal injuries, air and fluid in the intracranial space and sinuses, periorbital injury, soft tissue injury, and embedded foreign bodies. A non-contrast head CT helps identify intracranial bleeding and distinguish between the types of bleeds if present. This is recommended, especially when the patient experiences loss of consciousness for several minutes. Because maxillofacial trauma is highly associated with cervical spine injury, the physician must have a high index of suspicion for cervical spine fractures. The NEXUS criteria is used to guide imaging in these situations. 

Plain radiographs of the head are used when CT scans are not available. They may be used to screen for fractures and provide some insight into displaced fragments, but they have low sensitivity for detecting and establishing the extent of the injuries. A chest x-ray should be done when a missing tooth is noted on physical examination, as the patient may have aspirated it.

Ultrasound is a helpful bedside diagnostic tool in any trauma patient, and it has been shown to be an accurate diagnostic method when evaluating orbital trauma [16]. It is used when an isolated orbital injury is suspected or a CT scan is not readily available. It can pick up muscle entrapment, soft-tissue herniation, and orbital emphysema.

Risk Stratification

Several risk stratification tools have been developed for maxillofacial trauma. However, these are commonly used in clinical research to assess injury severity and determine the appropriate course of action. Although no specific tool was developed for use in an emergency department, other nonspecific tools like the Glasgow Coma Scale (GCS) and NEXUS criteria come in handy. The GCS score is used to rapidly assess the patient’s level of consciousness, guiding immediate interventions. The NEXUS criteria is used to clear patients from cervical injury clinically without imaging. 

The diagnosis of maxillofacial trauma is based on a combination of clinical assessment and diagnostic imaging. A thorough evaluation of both helps predict the risk. Some common clinical factors that may contribute to poorer outcomes include severe and complex fractures, extensive soft tissue injury, high-energy trauma, open fractures, ocular injuries, and pediatric and geriatric age groups [17,18].

Management

Initial Stabilization

Treating patients with maxillofacial trauma aims to restore function and optimize appearance. However, the primary focus upon presentation is to stabilize the patient. Initial management begins with a primary survey, which constitutes the “ABCDE” approach to identify life-threatening conditions and treat them promptly. 

Airway

Airway patency is a serious concern in maxillofacial trauma, and the nature of the injury often complicates airway management. Airway compromise may be complete, partial, or progressive [9]. Early signs of airway compromise include tachypnea, inability to speak in complete sentences, and abnormal noisy breathing. Agitation and abnormal behavior may indicate hypercapnia.

If the patient has obstruction from soft tissue, perform a jaw thrust maneuver. Cervical spine injury should be presumed in all maxillofacial injury patients until proven otherwise. Therefore, avoid mobilizing the neck until it is cleared. Inspect the oral cavity for any bleeding or secretions and suction accordingly. Consider manual removal with a finger sweep or forceps if a foreign body or debris is identified. Control patients with nasopharynx or oropharynx bleeding with nasal packing or compression with gauze [19].

The need for airway protection increases with severe maxillofacial fractures, expanding neck hematoma, stridor, profuse bleeding or continuous vomiting, and unconsciousness [9]. A nasopharyngeal airway is indicated in a conscious patient without a midface trauma. If the patient was unconscious or had a midface injury, an oropharyngeal airway may help temporarily. However, a definitive airway must be secured in patients who cannot maintain airway integrity. Definitive airway control is done by an endotracheal intubation (nasal or oral). Nasal endotracheal intubation is contraindicated in a base of skull fracture. Given the area’s delicacy and complexity of the injuries sustained, fiberoptic intubation by a skilled physician may provide immediate confirmation of tracheal placement and avoid further complications [10]. If the previous methods cannot be accomplished, a surgical airway (cricothyroidotomy or tracheostomy) should be considered. 

Breathing

The patient’s breathing, ventilation, and oxygenation should be assessed. Check the alignment of the trachea and listen to the patient’s chest bilateral for air entry and added sounds. Deviated trachea and decreased air entry upon auscultation increase the likelihood of tension pneumothorax, and a needle decompression should be performed. Look for soft tissue abnormalities and subcutaneous emphysema.

The patient should be connected to a pulse oximeter to monitor adequate hemoglobin oxygen saturation. If the patient is hypoxic, they should receive oxygen supplementation. Non-invasive ventilation should precede invasive ventilation methods. However, in severe injuries, mask ventilation may be difficult due to the disrupted anatomy of the face [20].

Like all trauma patients, a “full stomach” should be presumed in patients with maxillofacial trauma as digestion stops during trauma. In addition, blood is often swallowed and accumulates in the stomach. Regurgitation and aspiration are a big risk in such patients, and evacuation of stomach content is recommended [20]. A nasogastric tube is contraindicated in a skull base fracture. An orogastric tube is recommended instead to prevent intracranial passage [21].

Circulation

Maxillofacial trauma can cause profuse bleeding that can lead to shock. Monitor blood pressure and heart rate, auscultate, and check capillary refill and hand warmth. Tachycardia precedes low blood pressure in shock. Establish bilateral IV access with two large bore cannulas and draw blood for type and crossmatch. Fluid therapy with crystalloids should be initiated. Identify the source of hemorrhage. If external or intraoral bleeding occurs, apply direct pressure, pack, and suture. Carefully examine the tongue, as persistent bleeding can obscure the airway. In the case of epistaxis, anteroposterior packing will control the bleeding in most cases [10]. Additionally, topical tranexamic acid can be used in anterior epistaxis. In cases of LeFort fractures, intermaxillary fixation might be required when packing fails to stop the bleeding [10]. If the previously mentioned measures fail, consult IR, ENT, or surgery for more advanced interventions like arterial embolization and fracture reduction [22].

Disability

The patient’s mental status and neurologic function should be assessed initially. Glucose is measured at this point if not done upon arrival. The Glasgow Coma Scale helps assess the patient’s level of consciousness. Note any change in the mental status. A brief neurological exam is recommended. 

Exposure

Expose the patient fully while keeping them warm. Look for bruises, bite marks, lacerations, and other injuries, as the etiology of maxillofacial trauma is broad and often presents as polytrauma. Decontamination might be required depending on the nature of the trauma.

Medications

Isotonic crystalloid fluids and blood products are common treatments in trauma patients. Adequate pain management should be provided with NSAIDs, opioids, or local anesthesia. There are no guidelines on the use of prophylactic antibiotics in maxillofacial trauma. Nonetheless, there are specific scenarios where prophylactic antibiotics administration is recommended. Depending on the type of injury sustained, additional medications might be required. Refer to Table to explore the additional medications used in the setting of maxillofacial trauma:

Drug name (Generic)

Potential Use

Dose

Frequency

Cautions / Comments

Acetaminophen

mild-moderate pain (can be given with NSAIDs, with or without Opioids)

325-1,000 mg PO

 

Max Dose: 4 g daily

q4-6h

  • Ask for allergies
  • Ask for if/when they took Acetaminophen at home

Ibuprofen

mild-moderate pain (can be given with Acetaminophen)

600 mg PO

 

Max Dose: 3,200 mg daily

q6h

  • Can cause GI upset and increase risk of GI bleed
  • Renal insufficiency

Hydromorphone

Moderate – severe pain

0.5-4 mg IV/IM/SC

 

Max Dose: n/a

q4-6h

  • Risk of respiratory depression
  • Risk of addiction and abuse

Morphine sulfate

Moderate – severe pain

2.5-10 mg IV/IM/SC

 

Max Dose: n/a

q2-6h

  • Risk of respiratory depression
  • Risk of addiction and abuse
  • Hypotension

Metoclopramide

Nausea and vomiting (to prevent risk of aspiration)

1 to 2 mg/kg/dose IV

 

Max Dose: n/a

Every 2 hours for the first two doses, then every 3 hours for the subsequent doses.

  • Extrapyramidal side effects
  • If acute dystonic reactions occur, 50 mg of diphenhydramine may be injected IM.

Ondansetron

Nausea and vomiting (to prevent risk of aspiration)

0.15 mg/kg IV (not to exceed 16 mg)

 

Max Dose: n/a

q8hr PRN

  • Increased risk of QT prolongation, which increases the risk of cardiac arrhythmia and cardiac arrest.

Amoxicillin-clavulanic acid

Nasal packing (ppx for epistaxis – TSS)

 

Facial fractures communicating with open wounds of the skin

 

Mandibular fractures that extend into the oral cavity

2g PO (extended-release tablets)

 

Max Dose: n/a

q12h (7 days)

  • Ask for allergies
  • Ask if they have taken any antibiotic recently.
  • Hives and skin rash

Procedures

Epistaxis: Epistaxis is a common issue in maxillofacial trauma due to damage to the nasal structures and blood vessels. Managing epistaxis is crucial to prevent blood loss and ensure the airway remains clear. For anterior epistaxis, anterior nasal packing can effectively apply pressure to stop the bleeding. If the bleeding source is posterior, posterior nasal packing using a balloon catheter or Foley’s catheter may be necessary. These techniques help control bleeding and stabilize the patient, especially in cases where blood loss might obstruct the airway or lead to hemodynamic instability.

Inability to Protect Airway: In cases of severe maxillofacial trauma, there may be a risk of airway compromise due to swelling, bleeding, or physical obstruction from broken facial structures. If a patient cannot protect their airway, endotracheal intubation is required to secure it and maintain ventilation. Intubation provides a definitive airway, bypassing obstructions and ensuring adequate oxygenation, which is critical in trauma patients to prevent hypoxia and support life-sustaining measures.

Failed Intubation: Occasionally, intubation may be unsuccessful, particularly in patients with extensive facial injuries or anatomical challenges. In such cases, a cricothyroidotomy is performed. This emergency surgical procedure creates an opening in the cricothyroid membrane, providing an alternative airway route directly into the trachea. Cricothyroidotomy is a life-saving measure when intubation fails, ensuring oxygen can still be delivered to the lungs when other methods are ineffective.

Tension Pneumothorax: Maxillofacial trauma can sometimes be associated with thoracic injuries, leading to complications like tension pneumothorax, where air is trapped in the pleural cavity and compresses the lungs and heart, causing a life-threatening situation. Needle decompression is the first step in relieving the pressure by inserting a needle into the pleural space to allow trapped air to escape. This is followed by a tube thoracostomy (chest tube placement) to maintain the release of air and prevent the recurrence of tension pneumothorax. This procedure is essential to restore normal lung function and stabilize the patient’s respiratory status.

Special Patient Groups

Pediatrics

Pediatric patients’ anatomical and developmental differences should be considered when evaluating them for maxillofacial trauma. An infant’s frontal bone dents, while a child’s frontal bone experiences a depressed fracture under a force that causes facial fractures in adults [4]. Smaller force loads are needed to damage the facial bones than adults [4]. Given pediatric patients’ underdeveloped facial skeletons and sinuses, growth dysplasia is a common outcome of suboptimal treatment. Standard facial radiographs often miss fractures; a CT scan is more reliable in this age group [23]. Assess for orbital fracture thoroughly, as children’s orbital floor is pliable, increasing the risk of entrapment and rectus muscle ischemia [6].

Geriatrics

The impaired physiologic response and frailty of geriatric patients make their treatment more challenging. Although they are subject to the same mechanism of maxillofacial trauma as the other age groups, their response to the injuries differ. They are at a high risk of intracranial hemorrhage, but their basal vital signs often do not reflect signs of hemorrhage or hypoperfusion, making diagnosing shock difficult. Comorbidities and polypharmacy in this age group further mask the normal shock response. In addition, the likelihood of associated injuries in this group is high [24]. Elderly patients were reported to have more frequent cerebral concussions and internal organ injuries [25]. Nonetheless, a GCS of <15 has also been associated with higher mortality rates, especially in those older than 70 years [25]. Putting all of this into perspective when assessing elderly patients, a lower threshold for extensive investigations and referral is necessary.

When to admit this patient

Definitive repair of facial fractures is not a surgical emergency, and patients can be discharged home with a close follow-up in the clinic in most cases. An awake patient with good home care and isolated stable injuries (i.e., mandibular or nasal fracture) may be discharged home. However, admission should be considered in a number of situations. These include severe complex facial fractures, open fractures, the presence of comorbidities, and cases of associated injuries that need close monitoring. Admission is made to the intensive care unit or a surgical ward with a high level of monitoring.

Revisiting Your Patient

A 48-year-old male was brought to the ED by ambulance shortly after sustaining blunt trauma to the face. The patient was loading his quad bike off a truck when it accidentally flipped over and fell directly on his face and upper body. He could not recall what happened thereafter.
Upon arrival, his vitals were BP: 144/85 mmHg, HR: 104 bpm, T: 36.8°C, RR: 23 bpm, and SPO2: 99% on room air. He was awake on the AVPU score. On examination, the patient was bleeding profusely from his nose, breathing from his mouth, and having diffuse facial swelling. You are concerned about the extent of injuries sustained and have assembled your team to manage the patient adequately.

History was taken from his brother, who witnessed the incident. The brother confirmed that the patient had no LOC, dizziness, or vomiting but reported that the patient kept complaining of neck pain. He is known to have L5-S1 disc prolapse, does not take any medication, and has no known allergies.

You worry that the patient might suffer from airway compromise and quickly begin your primary survey. You hear gurgling noises and check the patient’s mouth to find it filled with blood. You suction and look for sources of bleeding in the mouth but find none. The airway becomes patent. You notice that EMS has placed a C-spine collar on the patient already. His lungs are clear bilaterally, and you insert an orogastric tube to suction his stomach contents. He is bleeding profusely from his nostrils, so you pack his nose anteriorly. This does not stop the bleeding, and the patient is spitting out blood. You then apply topical tranexamic acid and more packs, and the bleeding stops. His pulses are present, extremities are warm, and capillary refill time is less than 2 seconds. His GCS is 15/15, and his pupils are reactive to light. Upon exposing him, you notice lacerations on his lips and ears but no other injuries on the rest of his body.

Two large bore IV lines are inserted peripherally, blood is drawn for laboratory investigations, and intravenous normal saline is administered immediately. A 12-lead ECG demonstrated sinus tachycardia. You perform a bedside E-FAST to rule out pneumothorax/hemothorax, pericardial fluid, and peritoneal fluid. You ask for urgent CT scans, including a CT Head and Neck without contrast and a Maxillofacial CT. The CT scan report confirms no C-spine fractures, skull fractures, or brain injury. However, it identifies a Le Fort 1 fracture and fracture involving the right orbital wall. You safely remove the c-spine collar. You consult the Oral and Maxillofacial surgeon and the Ophthalmologist, and both agree to see the patient. You give the patient morphine to alleviate his pain.

You performed a secondary survey to ensure the patient was not deteriorating and to identify any additional injuries. The patient remained stable, and he was admitted to the surgical floor.

Figure: Fracture of the lateral wall left maxilla (long arrow) and a tripod fracture of the right zygoma (short arrows).

Author

Picture of Maitha Ahmad Kazim

Maitha Ahmad Kazim

Dr. Maitha Ahmad Kazim is an Emergency Medicine Resident at Dubai Health, recognized for her dedication in patient care and medical research. She earned her Doctor of Medicine degree from the United Arab Emirates University, where she graduated with distinction. Dr. Kazim is known for her commitment to advancing emergency care, demonstrated by her active engagement in research, mentorship, and medical education.

Picture of David O. Alao

David O. Alao

David is a senior consultant in emergency medicine and associate professor of medicine College of Medicine and Health Sciences, United Arab Emirates University, Al Ain, UAE
He graduated from the University of Ibadan, Nigeria. After initial training in general surgery in Leeds and Newcastle Upon-Tyne, United Kingdom, he had higher specialist training in emergency medicine in the South West of England.
He was a consultant in emergency medicine for 15 years at the University Hospitals Plymouth, United Kingdom where he was a Clinical Tutor, Academic Tutor and, Assistant professor at Plymouth University Peninsular School of Medicine and Dentistry (PUPSMD) UK.
David is a fellow of the Royal College of Surgeons of Edinburgh and the Royal College of Emergency Medicine UK.
His interests are undergraduate and postgraduate medical education, skills training and transfer, trauma systems development and resuscitation science. He has published over 30 papers in peer-reviewed journal.

Listen to the chapter

References

  1. Lalloo R, Lucchesi LR, Bisignano C, et al. Epidemiology of facial fractures: incidence, prevalence and years lived with disability estimates from the Global Burden of Disease 2017 study. Inj Prev. 2020;26(Supp 1):i27-i35. doi:10.1136/injuryprev-2019-043297
  2. Singaram M, G SV, Udhayakumar RK. Prevalence, pattern, etiology, and management of maxillofacial trauma in a developing country: a retrospective study. J Korean Assoc Oral Maxillofac Surg. 2016;42(4):174. doi:10.5125/jkaoms.2016.42.4.174
  3. Nalliah RP, Allareddy V, Kim MK, Venugopalan SR, Gajendrareddy P, Allareddy V. Economics of facial fracture reductions in the United States over 12 months. Dent Traumatol Off Publ Int Assoc Dent Traumatol. 2013;29(2):115-120. doi:10.1111/j.1600-9657.2012.01137.x
  4. Pappachan B, Alexander M. Biomechanics of Cranio-Maxillofacial Trauma. J Maxillofac Oral Surg. 2012;11(2):224-230. doi:10.1007/s12663-011-0289-7
  5. Sharifi F, Department of Oral & Maxillofacial Surgery, Mashhad University of Medical Sciences, Mashhad, Iran., Samieirad S, et al. The Causes and Prevalence of Maxillofacial Fractures in Iran: A Systematic Review. WORLD J Plast Surg. 2023;12(1):3-11. doi:10.52547/wjps.12.1.3
  6. Van Gijn D. Tips for GP trainees working in oral and maxillofacial surgery. Br J Gen Pract. 2012;62(594):50-51. doi:10.3399/bjgp12X616490
  7. Lynham A, Tuckett J, Warnke P. Maxillofacial trauma. Aust Fam Physician. 2012;41(4):172-180.
  8. Philip MR, Soumithran CS. Prevalence of Neurologic Deficits in Combined Facial and Cervical Spine Injuries: A Retrospective Analysis. Craniomaxillofacial Trauma Reconstr. 2021;14(1):49-55. doi:10.1177/1943387520940182
  9. Saigal S, Khan MM. Primary Assessment and Care in Maxillofacial Trauma. Oral and Maxillofacial Surgery for the Clinician. 2021:983-995. doi:10.1007/978-981-15-1346-6_48
  10. Truong T. Initial Assessment and Evaluation of Traumatic Facial Injuries. Semin Plast Surg. 2017;31(02):069-072. doi:10.1055/s-0037-1601370
  11. Mukherjee S, Abhinav K, Revington P. A review of cervical spine injury associated with maxillofacial trauma at a UK tertiary referral centre. Ann R Coll Surg Engl. 2015;97(1):66-72. doi:10.1308/003588414X14055925059633
  12. Patel BC, Wright T, Waseem M. Le Fort Fractures. In: StatPearls. StatPearls Publishing; 2023. Accessed August 12, 2023. http://www.ncbi.nlm.nih.gov/books/NBK526060/
  13. Peng N, Su L. Progresses in understanding trauma-induced coagulopathy and the underlying mechanism. Chin J Traumatol. 2017;20(3):133-136. doi:10.1016/j.cjtee.2017.03.002
  14. Das D, Salazar L. Maxillofacial Trauma: Managing Potentially Dangerous And Disfiguring Complex Injuries. Emerg Med Pract. 2017;19(4):1-24.
  15. Meara DJ. Diagnostic Imaging of the Maxillofacial Trauma Patient. Atlas Oral Maxillofac Surg Clin North Am. 2019;27(2):119-126. doi:10.1016/j.cxom.2019.05.004
  16. Forrest CR, Lata AC, Marcuzzi DW, Bailey MH. The role of orbital ultrasound in the diagnosis of orbital fractures. Plast Reconstr Surg. 1993;92(1):28-34. doi:10.1097/00006534-199307000-00004
  17. Sharma R, Parashar A. Unfavourable outcomes in maxillofacial injuries: How to avoid and manage. Indian J Plast Surg. 2013;46(2):221. doi:10.4103/0970-0358.118597
  18. Krausz AA, Krausz MM, Picetti E. Maxillofacial and neck trauma: a damage control approach. World J Emerg Surg. 2015;10(1):31. doi:10.1186/s13017-015-0022-9
  19. Hutchison I, Lawlor M, Skinner D. ABC of major trauma. Major maxillofacial injuries. BMJ. 1990;301(6752):595-599. doi:10.1136/bmj.301.6752.595
  20. Barak M, Bahouth H, Leiser Y, Abu El-Naaj I. Airway Management of the Patient with Maxillofacial Trauma: Review of the Literature and Suggested Clinical Approach. BioMed Res Int. 2015;2015:724032. doi:10.1155/2015/724032
  21. Spurrier E, Johnston A. Use of Nasogastric Tubes in Trauma Patients – A Review. J R Army Med Corps. 2008;154(1):10-13. doi:10.1136/jramc-154-01-04
  22. Jose A, Nagori S, Agarwal B, Bhutia O, Roychoudhury A. Management of maxillofacial trauma in emergency: An update of challenges and controversies. J Emerg Trauma Shock. 2016;9(2):73. doi:10.4103/0974-2700.179456
  23. Stewart C, Fiechtl JF, Wolf SJ. Maxillofacial trauma: Challenges in ED diagnosis and management. Emerg Med Pract. 2008;10(2):1-18.
  24. Shumate R, Portnof J, Amundson M, Dierks E, Batdorf R, Hardigan P. Recommendations for Care of Geriatric Maxillofacial Trauma Patients Following a Retrospective 10-Year Multicenter Review. J Oral Maxillofac Surg. 2018;76(9):1931-1936. doi:10.1016/j.joms.2017.10.019
  25. Kokko LL, Puolakkainen T, Suominen A, Snäll J, Thorén H. Are the Elderly With Maxillofacial Injuries at Increased Risk of Associated Injuries?. J Oral Maxillofac Surg. 2022;80(8):1354-1360. doi:10.1016/j.joms.2022.04.018

Reviewed By

Picture of Arif Alper Cevik, MD, FEMAT, FIFEM

Arif Alper Cevik, MD, FEMAT, FIFEM

Prof Cevik is an Emergency Medicine academician at United Arab Emirates University, interested in international emergency medicine, emergency medicine education, medical education, point of care ultrasound and trauma. He is the founder and director of the International Emergency Medicine Education Project – iem-student.org, chair of the International Federation for Emergency Medicine (IFEM) core curriculum and education committee and board member of the Asian Society for Emergency Medicine and Emirati Board of Emergency Medicine.

Fundamentals of ACLS (2024)

by Mohammad Anzal Rehman

You have a new patient!

A 56-year-old man presents to the Emergency Department with complaints of chest pain and dizziness that began an hour ago. Upon assessment by the triage nurse, his vital signs are as follows: his heart rate is 107 beats per minute, and his respiratory rate is 22 breaths per minute. His blood pressure is  96/70 mmHg, and his oxygen saturation is at 90% on room air. His temperature is 36.8°C.

You are the student on shift when this patient arrives, and immediately, your mind begins to jump across differential diagnoses for this patient. As you rush toward the patient’s room to join your senior, you prepare to list out all the potential causes of chest pain proudly. This must be a Myocardial Infarction, or maybe even an Aortic Dissection. Perhaps it is that rare Boerhaave syndrome you read about last night!

You finally catch up to the Emergency Physician, but before you can open your mouth to wax lyrical about esophageal ruptures, the Doctor states “Let’s begin by evaluating the ABCs.”

Initial Assessment

Emergency Medicine is one of the few specialties in medicine where patient evaluation begins in the same way for every patient, regardless of the probable diagnosis. Most clinicians are wired to jump straight to the ‘mystery-solving’ component of clinical presentation, with many undergraduate curriculums based around disease recognition. Emergency Medicine, however, places an emphasis on systematic assessment of the patient, starting with ‘The Primary Survey’.

The Primary Survey – ABCDE Approach

The Primary Survey aims to identify life-threatening conditions rapidly and systematically in critically ill patients, with appropriate stabilizing interventions performed when an abnormality is recognized. Besides streamlining the process in a high-stakes and often chaotic environment, the alphabetical order is designed first to address the most severe causes of mortality [1].

The Primary Survey aims to identify life-threatening conditions rapidly and systematically in critically ill patients, with appropriate stabilizing interventions performed when an abnormality is recognized. Besides streamlining the process in a high-stakes and often chaotic environment, the alphabetical order is designed first to address the most severe causes of mortality [1].

Airway

A patient’s airway connects air, and therefore oxygen, from outside the body to the lungs. Airway management is a term used to evaluate and optimize the passage of oxygen in the upper airway, which may be impaired when there is a blockage or narrowing of this pathway. The most common cause of upper airway obstruction is the tongue, which may ‘close’ the oropharynx posteriorly in patients who are comatose or in cardiopulmonary arrest, for example.

Assessment of the airway typically starts by evaluating any external features that may impact the passage of air through the naso- and oro-pharynx, such as facial or neck trauma, fractures, deformities, and any masses or swelling that may disrupt the airway tract. Allergies, especially anaphylaxis, and significant burns may cause edema of the laryngeal airway and produce obstruction. Excessive secretions may also congest the oropharynx and produce airway obstruction.

A patent or ‘normal’ airway allows a responsive patient to speak in full sentences without difficulty, implying a non-obstructed air passage down the oropharynx and through the vocal cords.

Clinical signs of obstruction may include stridor, gurgling, drooling, choking, gagging, or apnea. A physician may also identify an impending airway obstruction where loss of gag reflex, intractable vomiting, or worsening laryngeal edema may inevitably compromise the passage of air to the lungs and produce a failure to oxygenate or ventilate, prompting a decision to secure the tract through intubation.

Management

In the responsive patient, allow for the patient to be seated or lying in their most comfortable position as you assess the patency of the airway.

‘Opening’ the airway involves positioning the patient’s head in the ‘sniffing position’. In this position, a slight extension of the head with flexion of the neck, keeping the external auditory meatus in line with or above the sternal notch, is used to optimally align the pharyngeal and laryngeal airway segments, preventing obstruction posteriorly by the tongue (Figure 1). This is useful in patients who are unresponsive and cannot consciously protect their airway.

Figure 1 – Use of ‘sniffing position’ to open the airway

Two maneuvers are helpful in opening an unresponsive or sedated patient’s airway, optimizing air entry to the lungs:

1. Head tilt chin lift (Figure 2A) – Using fingertips under the chin, lift the mandible anteriorly while simultaneously tilting the head back using the other hand. Do not use this if cervical spine injury is suspected!

Figure 2A – Head-tilt chin lift

2. Jaw thrust (Figure 2B) – With thenar eminences of both hands anchored over both maxillary regions of the patient’s face, use your fingers at both angles of the mandible to lift it anteriorly. This maneuver is preferable in cases of suspected cervical spine injury as it does not cause hyperextension of the neck.

In unresponsive patients with excessive secretions, use of a rigid suction device can clear fluid and particulate matter such as vomitus.

Intubation may be performed if airway assessment deems it necessary to protect or secure the airway tract in a definitive way. If intubation is required, it should be performed as early as possible to prevent the evolution of a difficult airway, which would lower the chances of a successful intubation. It may also be useful to establish the risk of an inherently difficult airway using the L-E-M-O-N airway assessment method as below:

Look externally – facial trauma, large incisors and/or tongue, hairy beard, or moustache

Evaluate the 3-3-2 rule – where optimal distance between incisors on mouth opening should be 3 finger breadths. Similarly, 3 finger breadths (patient’s fingers) should span the distance from chin to hyoid bone, while the distance from hyoid to thyroid should measure 2 finger breadths.

Mallampati score – grades the view of an open mouth, with class 3 or more predicting a difficult intubation

Obesity/obstruction – Epiglottitis or a tonsillar abscess can inhibit easy passage of an endotracheal tube.

Neck mobility – if limited, positioning is difficult and causes suboptimal views during intubation.

iEM-infographic-pearls-airway - Assessing Airway Difficulty
assessing airway difficulty

Cervical spine immobilization

When the patient arrives in the Emergency Department (ED) following a significant physical trauma, such as head injury or motor vehicle collision, it is crucial to consider the integrity of the cervical spine. If injury is present in this region, further manipulation or movement of the neck may lead to spinal cord damage. Therefore, evaluation and management of airway for these patients should go hand in hand with cervical spine immobilization.

If no specialized equipment is available, or until one is prepared for use, attempts to limit neck movement can be done using manual in-line stabilization, where the provider’s forearms or hands may be positioned at the sides of the patient’s head to prevent indirect movements that could exacerbate underlying injury (see Figure 3).

Cervical spine immobilization is then performed using a rigid cervical collar. It may be augmented with head blocks on lateral sides to limit movement further as the patient is evaluated for injury (see Figure 4). The thoracolumbar region of the spine is immobilized using a spinal backboard, which keeps the patient in a supine position with minimal external force on the spine. Frequently utilized in Emergency Medical Services (EMS) during extrication and transport, all efforts should be made to transition the patient off the spinal board in the ED as it is quite uncomfortable, with prolonged use associated with pressure ulcers and pain.

Breathing

The lungs perform the vital function of delivering oxygen from the airway to the alveoli through ventilation. Perfusion at the alveoli allows for gas exchange; therefore, effective ventilation and perfusion both play a key role in the availability and utilization of oxygen by the human body. Evaluation of the Breathing component assesses factors that would indicate a compromise in ventilation.

The chest inspection should look for respiratory rate, use of accessory muscles, position of trachea (midline versus deviated), symmetry of chest rise, and/or any visible trauma to the thorax. Auscultation evaluates breath sounds for any bilateral inequal air entry or presence of crackles, crepitus, or wheeze. Percussion, though sometimes useful, is often difficult to perform adequately in a resuscitation environment.

Let’s compare the findings in normal lungs, pleural effusion, and pneumothorax based on chest rise, trachea position, percussion, and auscultation.

Normal Lungs: Chest rise is symmetrical with the trachea in the midline position. Percussion reveals a resonant sound. Auscultation presents vesicular breath sounds peripherally and bronchovesicular sounds over the sternum, with no added sounds.

Pleural Effusion: Chest rise remains symmetrical, and the trachea is midline. Percussion is dull over the area of effusion, and auscultation shows decreased breath sounds in the region of the effusion.

Pneumothorax: Chest rise is unequal, and the trachea may be deviated in cases of tension pneumothorax. Percussion reveals a hyper-resonant sound in the area of the pneumothorax, and auscultation shows decreased breath sounds over the pneumothorax region.

Measuring oxygen saturation using pulse oximetry (spO2) provides a percentage of oxygen in circulating blood, with normal levels typically at 95% or above. However, in patients with chronic lung disease, baseline oxygen saturation levels may decrease and can be as low as 88% in many cases. For patients experiencing shortness of breath and showing signs of hypoxia, pulse oximetry readings below 94% suggest that supplemental oxygen may be necessary. This can be administered through various oxygen delivery systems, as outlined in Figure 5 and described below.

Figure 5 – Common equipment used in airway management 1- Nasal cannula, 2- Simple face mask, 3- Nebulizer,* 4- Non-rebreather mask, 5- Venturi mask valves, 6- Rigid suction tip, 7- Bag-valve mask device, 8- Oropharyngeal airway (OPA), 9- Nasopharyngeal airway (NPA), 10- Direct Laryngoscope, 11- Endotracheal tube with stylet, 12- Colorimetric end-tidal CO2 detector, 13- Bougie, 14- Laryngeal Mask Airway (LMA) *NOT an oxygen delivery device, used to administer inhaled medication such as bronchodilators and steroids CO2: Carbon dioxide

General concepts—We typically breathe in room air, which contains 21% oxygen. Each Liter per minute of supplemental oxygen provides an additional 4% inspired oxygen (FiO2) to the patient.

Nasal cannula – Administered through patient nostrils, can provide a maximum flow rate of 4-6 Liters per minute of oxygen, which equals roughly 37 – 45% FiO2

Simple face mask – Applied over the patient’s nose and mouth, can provide a maximum flow rate of 6-10 Liters per minute of oxygen, which equals roughly 40 – 60% FiO2

Venturi mask – Typically used in COPD, where over-oxygenation is avoided. Different colors deliver various flow rates to limit oxygen delivery to the required amount only; Blue (2-4L/min, FiO224%), White (4-6L/min, FiO2 28%), Yellow (8-10L/min, FiO235%), Red (10-12 L/min, FiO2 40%), Green (12-15 L/min, FiO260%)

Non-rebreather mask – Utilizes an attached bag with a reservoir of oxygenated air along with one-way valves on the mask to prevent rebreathing of expired air, optimizing oxygenation. It can provide a maximum flow rate of 15 Liters per minute of oxygen, which equals roughly 85 – 90% FiO2.

Non-invasive ventilation (CPAP/BiPAP) is a tight-fitting mask device that uses high positive pressure to keep the airway open and enhance oxygenation. It is particularly useful in conditions such as COPD exacerbation, acute pulmonary edema/heart failure, and sleep apnea.

Bag-valve mask device: A self–inflating bag attached to a reservoir delivers maximal, high-flow 100% oxygen. This method of manual ventilation is used in rescue breathing and oxygen delivery in nonresponsive or cardiopulmonary arrest patients.

Circulation

The circulation component of the Primary Survey evaluates the adequacy of perfusion by the cardiovascular system. The patient’s general appearance is assessed for signs of pallor, mottling, diaphoresis, or cyanosis, which indicate inadequate or deteriorating perfusion status. Pulses are checked centrally (e.g. carotid pulse, especially if patient with impaired breathing) and peripherally (e.g. radial) alongside hemodynamic assessment, including blood pressure and heart rate checks. Information from this segment also provides valuable insight into potential signs of shock. Extremities are palpated in order to determine any delays in capillary refill time (more than 2 seconds signifies inadequate perfusion, e.g. hypovolemia), peripheral edema in lower extremities (signs of heart failure), and skin temperature (cool or warm to touch).

In cases of trauma, systematic evaluation of circulation also seeks to ascertain areas of potential blood loss or collection, with interventions for any long-bone deformities and/or bleeding from open wounds performed as they are discovered.
Intravenous (IV) line insertion is also performed as part of the management of circulation, as any required fluid or blood products can then be administered through a large-bore IV line (16 gauge or higher). If IV insertion is difficult on multiple attempts, when volume resuscitation is urgently required, Intraosseous (IO) access should be sought to prevent delay in any needed treatment. Insertion of a peripheral venous line often occurs concomitant to blood extraction for any urgent laboratory investigations and/or point-of-care testing. Some common examples of tests performed on critically ill patients include venous blood gas, complete blood count, type and crossmatch, troponin, urea, electrolytes, and creatinine.

Finally, circulation assessment requires an evaluation of cardiac rhythm. Basic auscultation may reveal the rate and regularity of rhythm along with murmurs. However, a critically ill patient will also benefit from the immediate attachment of cardiac pads to the bare chest and connection to a cardiac monitoring device, which provides the physician with the patient’s current cardiac rhythm.

A normal sinus rhythm (Figure 6) consists of a P wave (atrial depolarization), followed by a QRS wave (ventricular depolarization – normally less than 120 ms), with a subsequent T wave (ventricular repolarization). P-R intervals typically have a duration of 120 – 200 ms. A regular rhythm, with a consistent P wave preceding QRS complexes, with a normal heart rate (between 60 – 100 beats per minute (bpm)) is required to consider a rhythm to be normal sinus on the cardiac monitor.

Figure 6 – Normal sinus rhythm

The American Heart Association’s (AHA) Advanced Cardiac Life Support (ACLS) course and guidelines outline a series of internationally recognized cardiac rhythms and their general management when encountered [2]. Some of the most important rhythms, along with the AHA bradycardia and tachycardia algorithms, are summarized below:

Figure 7.1 - Sinus bradycardia (HR < 50 bpm)

Several different conditions, including abnormal heart conduction, damage to the myocardium, metabolic disturbances, or hypoxia, can cause bradycardia. A lower heart rate can result in decreased perfusion to end-organs, such as the brain, with resultant signs and symptoms such as dizziness, confusion, shortness of breath or chest pains. Management (Figure 7.2) aims to treat the underlying cause and increase the heart rate (atropine, dopamine/epinephrine and/or cardiac pacing) if needed to restore the heart’s ability to perfuse organs adequately.

Figure 7.2 – American Heart Association’s Bradycardia Algorithm

Tachycardia (Figure 8.1) is a heart rate of more than 100 bpm that may present as several types of waveforms on the cardiac monitor. Supraventricular tachycardia (SVT) originates in the upper chambers of the heart. The rapid heart rate prevents adequate filling of the heart between contractions, causing signs and symptoms such as dizziness, palpitations, or chest pain.

Figure 8.1 - Supraventricular Tachycardia (SVT)

Management (Figure 8.2) typically involves Valsalva maneuvers, medication (e.g. adenosine), and/or synchronized cardioversion as needed to revert the rhythm back to baseline.

Figure 8.2 – American Heart Association’s Tachycardia Algorithm

SVT produces a narrow-complex tachycardia (QRS segments < 120 ms). In comparison, monomorphic Ventricular Tachycardia (Figure 8.3) originates in the lower chambers of the heart and produces a wide-complex (QRS segments > 120 ms) tachycardia on the cardiac monitor. Similarly, this rhythm may cause dizziness, shortness of breath, or chest pain and is managed with medication or synchronized cardioversion.

Figure 8.3 - Ventricular Tachycardia

ACLS algorithms often divide patients based on “stable” and “unstable” categories. This grouping aims to ascertain which individuals have a pathology severe enough to impair cardiac output to the point of causing serious inadequacies in end-organ perfusion. This ‘instability’ is manifested by altered mental status, ischemic chest pain, drastically low hemodynamic parameters (e.g. systolic BP < 90 mmHg), signs of shock, and signs of acute decompensated heart failure.

Disability

This segment evaluates the level of consciousness and responsiveness of the patient. Level of consciousness may be assessed generally using the AVPU scale (below);

Alert: fully alert patient
Verbal: some form of verbal response is present, though not necessarily coherent.
Pain: response to painful stimulus
Unresponsive: no evidence of motor, verbal or eye-opening response to pain

or more explicitly, using the Glasgow Coma Scale (GCS)

Choose the best response of patient
EYE OPENING
4: Spontaneously
3: To verbal command
2: To pain
1: No response
BEST VERBAL RESPONSE
5: Oriented and converses
4: Disoriented and converses
3: Inappropriate words; cries
2: Incomprehensible sounds
1: No response
BEST MOTOR RESPONSE
6: Obeys command
5: Localizes pain
4: Flexion withdrawal
3: Flexion abnormal (decorticate)
2: Extension (decerebrate)
1: No response
Glasgow Coma Score (GCS) (Modified from Teasdale, G., & Jennett, B. (1974). Assessment of coma and impaired consciousness: a practical scale. The Lancet, 304(7872), 81-84.) - Please read this article to get more insight regarding GCS.

Exposure

Complete exposure of the patient may be necessary to completely evaluate for any external signs of infection, injury, and rash. This is especially useful in trauma, where log-rolling of the patient is included to ensure the back and spine are also included in a complete assessment for any traumatic injuries. As you expose the patient, obtain consent, be mindful of their dignity, and uncover each segment of the body sequentially, covering it back to prevent any hypothermia for the patient. A core temperature reading also completes vital sign measurements for the patient.

Practical implementation of the Primary Survey

The “cursory” primary survey

It may seem surprising to consider that virtually every patient who enters the Emergency Department, despite the severity of the illness, undergoes some form of a Primary Survey by the treating physician. However, the practicality of this becomes quite obvious when you consider a simple question frequently asked at the beginning of a patient encounter:

“How are you?”

An adequate response of “I am all right” or “Well, I have had this pain in my stomach…” seems fairly standard, but it addresses most of the components detailed in the previous section. A patient who can form words without difficulty or added sounds generally has an intact or patent Airway. Their ability to form words depends on air that has been sufficiently ventilated and moving through the vocal cords, hence the Breathing is adequate. An appropriate response to the question allows us to assume that Circulation adequately perfuses the brain to allow comprehension and formulation of new words oriented to the circumstances of the encounter, hence providing insight into Circulation and, to a degree, Disability.

Synchrony in the Emergency Department

Although systematic assessment during the Primary Survey is laid out in order, it is also important to note that an Emergency Department consists of teams of healthcare professionals who often have the personnel and resources to simultaneously perform tasks to efficiently address all components of the Primary assessment, without delay between segments.
In practice, an example of how synchrony works would involve a patient who, on initial, immediate assessment, is deemed to be in significant distress and/or critically ill. The patient is immediately moved into the ED to a resuscitation area, where team members expose the chest, attach cardiac pads to connect the patient to a cardiac monitor, obtain a fresh set of vital signs, including spO2monitoring, with IV cannula insertion, blood extraction for testing as needed. At the same time, a primary survey is conducted simultaneously by another physician who moves through Airway, Breathing, Circulation, Disability and Exposure. In more advanced systems, a member may be dedicated to each component of the Primary Survey.

Adjuncts

A number of resources are accessible to the Emergency Physician that may aid in diagnosing and investigating the critically ill patient. Utilizing these alongside the initial Primary Survey provides valuable, relevant information that can further guide clinical decision-making and diagnosis during evaluation.

  1. Electrocardiogram – A 12-lead electrocardiogram provides a complete picture of the heart’s electrical activity in various vectors and segments, allowing for a more accurate evaluation for rhythm disturbances, such as in acute myocardial infarction, hyperkalemia, bundle branch blocks, and torsade de pointes. This often ties into the Circulation assessment and allows for a more comprehensive look into the heart’s electrophysiology.
  2. Portable X-rays – Particularly in trauma, urgent chest and pelvic X-ray films can often be obtained without having to transfer the patient to Radiology, hence providing more information on suspected lung pathologies (e.g. pneumothorax, effusion/hemothorax) and pelvic abnormalities (e.g. fracture, displacement).
  3. Urinary/ gastric catheters – Urinary catheters are useful to evaluate fluid status and monitor output for the patient undergoing volume resuscitation. When relevant, gastric tube insertion can assist in gastrointestinal decompression, if needed, as well as minimize the risk of aspiration in certain patients.
  4. Point-of-Care Ultrasonography (PoCUS) – A rapidly evolving and increasingly prevalent modality in the ED is the ultrasound.[3] Various probes, at different frequencies, utilize ultrasound waves to provide the physician with real-time visualization of the body’s internal structures. These images are fast and often very reliable in determining major findings that can guide decision-making in critically ill patients (e.g. presence of post-traumatic intra-abdominal free fluid, pneumothorax, cardiac tamponade). Figure outlines some examples of information that can be extracted using PoCUS.

 

HI-MAP in Shock

Reassessment

Each intervention performed in the Primary Survey should ideally be accompanied by a reassessment of vital signs and patient clinical status and a restarted Primary Survey beginning from Airway. Identifying any improvements, deteriorations, or non-responses that will be pivotal in guiding the initiation or discontinuation of further intervention as per the clinical case is crucial.

Focused History and Secondary Survey

If the patient is appropriately evaluated and stabilized following the Primary Survey, the treating physician may proceed with a focused history and secondary survey appropriate to the clinical circumstances. One example of a focused history incorporates the mnemonic SAMPLE to organize pertinent information as follows:

S – Signs/symptoms of presenting complaint

A – Allergies to any food or drugs

M – Medications (current, recent changes)

P – Pertinent past medical history

L – Last oral intake

E – Events leading to the illness or injury

A secondary survey in the Emergency Department is a more comprehensive physical examination performed systematically in a head-to-toe fashion to investigate any clinically relevant findings. In case of trauma, this also involves careful inspection for any missed injuries, deformities, or signs of underlying blood collection.

As the secondary survey is performed, relevant investigations and/or imaging may be ordered to augment the evaluation of the present clinical condition (e.g. Computerized Tomography (CT) of the brain after signs of basal skull fracture noted on inspection of the face and head). Information gathered from the survey and results of any ordered investigations, coupled with the clinical condition and/or response to therapy in the ED, if any, is used to determine patient disposition at the end of the ED encounter.

Revisiting Your Patient

You assist the Emergency Physician in performing a Primary Survey. The airway is patent, with the patient phonating in full sentences and breathing with mild tachypnea but no added sounds on auscultation. You initiate supplemental oxygen through a non-rebreather mask, with an increase in spO2 to 99%. You reassess and proceed through Airway, Breathing, and Circulation. As you discuss initiating IV fluids with your senior, the patient complains of worsening chest pain, palpitations, and dizziness.You attach the patient to the cardiac monitor and notice the rhythm below:

Cardiac pads have already been attached to the patient. Noting the presence of ischemic chest pain, you correctly identify the patient as having an unstable, narrow-complex tachycardia, most likely an SVT and prepare for synchronized cardioversion. Conscious sedation is conducted after explaining the procedure and obtaining consent from the patient. 50 joules of biphasic energy is then administered for synchronized electrical cardioversion. The rhythm changes on the monitor to the reading below:

You observe an organized rhythm but note that the patient is now unresponsive, with eyes closed and no palpable carotid pulse.

Basic Life Support

Cardiopulmonary arrest occurs when the heart suddenly stops functioning, resulting in lack of blood flow to vital organs in the body, such as the lungs and brain. Therefore, signs of arrest are manifested as a lack of breathing (apnea), lack of pulse and unresponsiveness. The most common cause of cardiac arrest is coronary artery disease.[4] Respiratory arrest refers to a cessation of lung activity, but with a present, palpable pulse and functioning heart.
The International Liaison Committee on Resuscitation (ILCOR) and the American Heart Association (AHA) are some of the key figures who have developed international guidelines on the recognition and management of cardiac arrest patients.[5] Basic Life Support (BLS) and Advanced Cardiac Life Support (ACLS) courses were established to optimize the workflow and, therefore, patient outcomes in Cardiopulmonary Resuscitation (CPR).

CPR forms the cornerstone of BLS to effectively maintain the victim’s circulatory and ventilatory function until circulation either spontaneously returns or is hopefully restored through intervention. The general concepts within BLS are outlined below:

1. A person who has a witnessed collapse, lack of response or who is suspected of being unresponsive due to cardiac arrest should be approached for further assessment and management. However, it is important for the rescuer to first determine whether the scene is safe around the patient before attempting any intervention. An example of this would be a victim drowned in water, who should be removed from the body of water onto a dry surface prior to attempting life-saving chest compressions or defibrillation.

Figure 9 - Witness
Figure 10 - Check for responsiveness

2. Check for responsiveness. Firmly tapping both shoulders with the palms of your hands and a clear, verbal prompt, such as “Hey, are you okay?” should be incorporated to ensure that the victim is, indeed, unresponsive to an otherwise arousable stimulus.

3. You have determined that the patient is unresponsive. If you are alone, shout loudly and clearly for help and assistance. If no help is nearby, call Emergency Medical Services using your mobile phone.

Figure 11 - Call for help
Figure 12 - Open airway, palpate carotid artery, observe the chest

4. Open the patient’s airway (tilt chin upward into sniffing position). Palpate the carotid pulse by placing two fingers (index and middle finger) just lateral to the trachea on the side closest to you while simultaneously observing the chest for any spontaneous chest rise (breathing). The pulse check should take a minimum of five (5) seconds but no more than 10 seconds to avoid delay in life-saving intervention.

5. When help is available, the chain of survival begins by activating the Emergency Response System. In addition to activating the Emergency Response, ask the person who has responded to your call for help getting an Automated External Defibrillator (AED) device. An example of instruction to a bystander (out of hospital) would be to ‘call an ambulance and get an AED!’. Inside a hospital, if another healthcare provider has come to aid, you may ask them to ‘activate the Emergency Response System/’Code Blue’ and get the crash cart/AED.’

6. Begin high-quality chest compressions. Hands are placed with fingers interlaced to exert pressure using the heel of one hand at the center of the chest, over the lower half of the breastbone (sternum), in line with the nipples (in men), with shoulders directly over your hands and arms straight at a perpendicular angle to the victim’s chest. High-quality chest compression is one of the few variables which have been evidenced to improve patient survival in cardiac arrest.

Figure 13 - Chest compression

Keep the following features in mind to maintain high-quality chest compressions:

  • More than 80% of the time in resuscitation or more should be spent on compressions (Chest compression fraction of > 80%)
  • The frequency of compressions should follow a rate of 100–120 compressions per minute.
  • Compression depth in adults is at least 2 inches. In infants and children, depth should be at least one-third of the anterior-posterior diameter of the chest.
  • After each compression, the hands should be withdrawn to allow adequate chest recoil and fill the heart between compressions.
  • Minimize interruptions in chest compression
  • Avoid hyperventilation (see next point).
Figure 14 - Bag-Valve-Mask Ventilation. Two-Hand technique

7. Compressions should follow the ratio 30:2, that is, 30 compressions followed by 2 rescue breaths delivered by a mouth barrier device (pocket mask) in the sniffing position or a Bag-valve mask (BVM) device if another rescuer is present to manage the airway in hospital. The BVM’s mask should be held with a tight seal using the E-C technique over the bridge of the nose and covering the mouth. 

Breaths should be over 1 second, with enough air pushed in to observe a chest rise and no hyperventilation or excessive bagging of the BVM to avoid gastric insufflation. Two attempts at rescue breaths are performed, minimizing time to under 10 seconds and resuming chest compressions immediately after. If a definitive airway (e.g. endotracheal tube) is in place, resume compressions without pause at a rate of 100-120 compressions per minute while breaths are delivered once every 6 seconds.

8. Once an AED or cardiac monitor/defibrillator is available, place the pads on the victim’s bare chest (dry the skin if wet) in either an anterior-lateral or anterior-posterior position.When in doubt, follow the machine’s prompts and the instructions on the pads themselves to guide placement.

Figure 15 - Correct placement of transcutaneous pacing pads.jpg

9. Follow the prompts on the AED. Stop compressions when the device analyzes rhythm and stay clear of the patient (not touching any part of the patient’s body). During an in-hospital resuscitation, as per ACLS workflow, stay clear, as the team leader should analyze the initial rhythm to ascertain the presence of a shockable or non-shockable rhythm. Either way, the device or team leader should prompt whether a shock is advised. Continue compressions as the device charges, but ensure that all rescuers are clear of the patient when the shock is delivered using the AED/defibrillator device.

Figure 16 - Shock delivery.

A victim who is unresponsive but has a palpable pulse has respiratory arrest, which is managed using rescue breathing only. Breaths are delivered once every 6 seconds without chest compressions while transport to a higher level of care and/or management of any underlying cause for the condition is initiated.

Advanced Cardiac Life Support

The Advanced Cardiac Life Support algorithms were designed to deliver a higher level of resuscitative care where providers with increased training and improved resources are available. This type of augmented management is customary to the Emergency Department, where a Rapid Response Team or Code Blue team would respond when activated and initiate a more team-based approach to cardiopulmonary resuscitation.

Instead of an AED, in-hospital settings have a cardiac monitor/defibrillator, usually mounted atop a crash cart consisting of a CPR back-board (to support chest compressions by providing a firm surface to use under the patient’s chest), drawers with medication used during cardiac arrest, and various equipment for airway management and IV/IO access. Once brought to the bedside, the cardiac pads are similarly placed on the patient’s chest while BLS maneuvers (chest compressions and rescue breaths) continue. Once placed, however, compressions should be paused to assess the cardiac monitor’s cardiac rhythm. The type of rhythm should be identified asshockableornon-shockable(Figure 17s).

Figure 17.1 - NON-SHOCKABLE - Asystol
Figure 17.2 - NON-SHOCKABLE - Pulseless electrical activity – organized rhythm in the absence of palpable pulse
Figure 17.3 - SHOCKABLE - Pulseless Ventricular Tachycardia
Figure 17.4 - SHOCKABLE - Ventricular fibrillation

“Shockable” rhythms (pulseless Ventricular Tachycardia and Ventricular Fibrillation) are a product of aberrant electrical conduction of the heart. Rapid, early correction of this rhythm is the most important step in returning the body to its normal circulatory function. Early defibrillation is one of the few variables that has been evidenced to improve patient survival in cardiac arrest, the other notable one being high-quality chest compressions.[6]

Defibrillation involves using an asynchronous 200J of biphasic (360J if monophasic) energy, delivering an electric current through the cardiac pads attached to the patient’s chest to revert the heart to a rhythm that can sustain spontaneous circulation. Chest compressions should be ongoing while charging, but all persons should stay clear of the patient when shock is being delivered, and this is frequently verified with verbal feedback (‘Clear!’) before pressing the defibrillator button to deliver the shock. Immediately after the shock, chest compressions should resume to minimize interruptions between compressions.

Two minutes of chest compressions and rescue breaths make up each cycle of CPR, at the end of which a rhythm check should be performed for any changes and/or presence of pulse. Figure 18 outlines the ACLS algorithm used to manage shockable and non-shockable rhythms in cardiac arrest. Early shock in shockable rhythms is followed by a cycle of CPR, a second shock if still with a shockable rhythm, after which 1mg of IV epinephrine is given, with subsequent doses every 3 to 5 minutes. During the third cycle of CPR, after 3 shocks have been delivered for a persistent shockable rhythm, a bolus of IV Amiodarone 300mg is typically administered, with a dose of 150mg in a subsequent CPR cycle if still with a shockable rhythm.

“Non-shockable” rhythms (pulseless electrical activity (PEA) and asystole) are not typically a product of disorganized electrical activity in the heart. Instead, an underlying cause has resulted in cardiac arrest for these patients. While the majority of cardiac arrest is caused by coronary artery disease, the consideration of reversible causes by use of the H’s (hypovolemia, hypoxia, hyper-/hypokalemia, hydrogen ions (acidosis), and hypothermia) and T’s (thrombosis/embolism, toxins, tension pneumothorax, and cardiac tamponade) may help recognize and manage other possible etiologies in patients.

The management of non-shockable rhythms focuses on consistent, high-quality CPR, with regular pulse checks every 2 minutes, addressing reversible causes, and administering IV epinephrine 1mg every 3 to 5 minutes.
A palpable pulse with measurable blood pressure signals the Return of Spontaneous Circulation (ROSC).

Figure 18 - ACLS Adult Cardiac Arrest Algorithm

Resuscitation Team Dynamics

The Emergency Department is equipped with the resources and personnel to provide care beyond basic life support. Resuscitation is optimized when multiple providers work together to effectively perform tasks toward management of the patient, thereby multiplying the chances of a successful outcome for the patient. A high-performance team typically consists of members allocated to the following roles and responsibilities:

  • Airway – Opens and maintains the airway. Manages suctioning, oxygenation, and ventilation (Bag-valve mask) and assesses the need for a definitive airway if needed.
  • Medication – Inserts and maintains IV/IO access. Manages medication administration and fluids.
  • Monitor/defibrillator – Ensures attached cardiac pads and AED/cardiac monitor/defibrillator device are working appropriately to display the patient’s cardiac rhythm in clear view of the team leader. Administers shocks using the devices as needed. May alternate with the compressor every 5 cycles or 2 minutes to prevent compression fatigue
  • Compressor – Performance of high-quality chest compressions as part of CPR for the cardiac arrest patient. Focuses on quality and consistency of compressions. You may switch to another standby compressor or monitor/defibrillator every 5 cycles or 2 minutes if compressions are affected by fatigue.
  • Recorder – Documents the timing of medication, intervention (shocks, compression), and communicates these to the Team Leader, with prompts to enable timely dosing of frequent medication (e.g., ensuring epinephrine every 3 to 5 minutes is administered as per the verbalized order)
  • Team leader – A defined leader who coordinates the team’s efforts and organizes them into roles and responsibilities that are clear, well-understood, and within their individual limitations. Provides explicit instructions and direction to the resuscitation effort, focused on patient care and optimized performance from all team members. Promotes understanding and motivates members, identifying any potential deficit or depreciation of quality during resuscitation and facilitating improvement in performance as needed.

All team members are encouraged to conduct themselves with mutual respect and practice closed-loop communication, where each message or order is received with verbal confirmation of understanding, then execution of the order, centralizing all information back to the team leader. Figure 19 provides an example of the possible placement of each member during resuscitation that may optimize their workflow through the resuscitation attempt. Ideally, the team leader remains at the foot of the bed, in clear view of all members, with involvement limited to coordination of the team’s efforts and minimal direct execution of tasks.

Figure 19 - An example of optimized team placement during resuscitation

Post Arrest Care

If the patient is found to have Return of Spontaneous Circulation (ROSC), post-cardiac arrest care should be initiated to enhance the preservation of brain tissue and heart function. This involves a sequential assessment and optimization of Airway, Breathing, and Circulation in the initial stabilization phase. A definitive airway may be placed so ventilation is more appropriately controlled, with parameters set to optimize oxygen administered with ventilatory function. Figure 20 outlines the ACLS algorithm and parameters often used to help guide post-cardiac arrest care. Circulation incorporates fluids, vasopressors, and/or blood products to achieve an adequate systolic blood pressure above 90 mmHg, with Mean Arterial Pressure of at least 65 mmHg typically indicating perfusion within stable parameters.

It is imperative to obtain a 12-lead ECG early to ascertain the presence of an ST-elevation myocardial infarction (STEMI), which will require expedited transfer of the patient to a Cath Lab for definitive reperfusion therapy. The patient’s responsiveness should be reassessed, and the determination for additional investigation should be performed in conjunction with other critical care management as needed.

Of note, unresponsive patients may benefit from Targeted Temperature Management (TTM), which involves the maintenance of core body temperature at a target of 32 – 36 ℃ for 24 hours, or preferably normothermia at 36 °C to 37.5 °C with an emphasis on prevention of hyperthermia, in order to protect and optimize brain recovery post-arrest.[7]

Almost all cardiac arrest survivors will require a period of intensive care observation and management. If no immediate intervention is needed (e.g., reperfusion therapy), patients inside a hospital will need to be transitioned to an Intensive Care Unit (ICU) for further care.

Figure 20 – Post-Cardiac Arrest Care

What do you need to know?

  • Emergency Medicine, especially in critical care, emphasizes a systematic approach to the unwell patient.
  • The Primary Survey is designed to recognize and address life-threatening conditions effectively and timely.
  • The Primary Survey components are Airway (& and C-spine in trauma), Breathing, Circulation, Disability, and Exposure.
  • If an intervention is performed at any level of the survey, you must reassess the patient by commencing the Primary Survey again, starting with Airway.
  • Reassess and review your patient for changes frequently.
  • Many of the actions performed in the initial assessment of the critically ill patient may occur simultaneously when more team members are present in an Emergency Department. Do not let the chaos of the scene distract you from completing each step of the assessment.
  • The AHA has well-established guidelines for assessing and managing patients through the Primary Survey. Use the algorithms and the patient’s status as ‘stable’ or ‘unstable’ to guide the management of recognized pathologies, especially in Circulation.
  • The ED is home to a variety of adjuncts, including portable X-rays, ECG, and point-of-care ultrasound, which can provide the physician with rapid, readily accessible information to guide management.
  • Remember the SAMPLE mnemonic for a focused history in the critically ill patient.
  • An unresponsive patient should be immediately recognized, and Emergency Response Systems should be activated.
  • Performance of Basic and Advanced cardiac life support focuses on preserving blood circulation transiently to maintain the perfusion of organs, such as the brain, until the cause of the condition is reversed or managed.
  • The majority of cardiac arrest is caused due to coronary artery disease.
  • The two most important predictors of patient survival in cardiac arrest are high-quality CPR and early defibrillation (for a shockable rhythm)
  • An effective resuscitation in the ED often relies on the concerted efforts of multiple team members, led by a team leader who coordinates tasks in an organized, effective way to improve patient survival and outcomes.

Author

Picture of Mohammad Anzal Rehman

Mohammad Anzal Rehman

EM Residency Graduate from Zayed Military Hospital in Abu Dhabi, UAE. Founder/President of the Emirates Collaboration of Residents in Emergency Medicine (ECREM). Editor-in-Chief for the Emirates Society of Emergency Medicine (ESEM) Monthly Newsletter. I have a vested interest in sharing updated knowledge and developing teaching tools. As a healthcare professional, I continually strive to incorporate the newest clinical research into practice and am an active advocate for the use of Point of Care Ultrasonography (POCUS) in the ED.

Listen to the chapter

References

  1. Reynolds T. Basic Emergency Care: Approach to the Acutely Ill and Injured. World Health Organization; 2018.
  2. 2020 Advanced Cardiac Life Support (ACLS) Provider Manual. American Heart Association; 2021.
  3. Hashim A, Tahir MJ, Ullah I, Asghar MS, Siddiqi H, Yousaf Z. The utility of point of care ultrasonography (POCUS). Ann Med Surg (Lond). 2021;71:102982. Published 2021 Nov 2. doi:10.1016/j.amsu.2021.102982
  4. Cardiac Arrest Registry to Enhance Survival (CARES) 2022 Annual Report; 2022, https://mycares.net/
  5. Wyckoff MH, Singletary EM, Soar J, et al. 2021 International Consensus on Cardiopulmonary Resuscitation and Emergency Cardiovascular Care Science With Treatment Recommendations: Summary From the Basic Life Support; Advanced Life Support; Neonatal Life Support; Education, Implementation, and Teams; First Aid Task Forces; and the COVID-19 Working Group. Resuscitation. 2021;169:229-311. doi:10.1016/j.resuscitation.2021.10.040
  6. Soar J, Böttiger BW, Carli P, et al. European Resuscitation Council Guidelines 2021: Adult advanced life support [published correction appears in Resuscitation. 2021 Oct;167:105-106]. Resuscitation. 2021;161:115-151. doi:10.1016/j.resuscitation.2021.02.010
  7. Lüsebrink E, Binzenhöfer L, Kellnar A, et al. Targeted Temperature Management in Postresuscitation Care After Incorporating Results of the TTM2 Trial. J Am Heart Assoc. 2022;11(21):e026539. doi:10.1161/JAHA.122.026539

Acknowledgements

  • Marina Margiotta – Illustrator
  • Paddy Kilian – Emergency Physician – Mediclinic City Hospital, Dubai, Director of Academic Affairs – Mohammed Bin Rashid University Of Medicine and Health Sciences
  • Rasha Buhumaid – Consultant Emergency Physician – Mediclinic Parkview Hospital, Dubai, Assistant Professor of Emergency Medicine – Mohammed Bin Rashid University Of Medicine and Health Sciences, President of the Emirates Society of Emergency Medicine (ESEM)
  • Amog Prakash – Medical Student – Mohammed Bin Rashid University Of Medicine and Health Sciences
  • Fatima Al Hammadi- Medical Student – Mohammed Bin Rashid University Of Medicine and Health Sciences

Reviewed By

Picture of Arif Alper Cevik, MD, FEMAT, FIFEM

Arif Alper Cevik, MD, FEMAT, FIFEM

Prof Cevik is an Emergency Medicine academician at United Arab Emirates University, interested in international emergency medicine, emergency medicine education, medical education, point of care ultrasound and trauma. He is the founder and director of the International Emergency Medicine Education Project – iem-student.org, chair of the International Federation for Emergency Medicine (IFEM) core curriculum and education committee and board member of the Asian Society for Emergency Medicine and Emirati Board of Emergency Medicine.

Physiologically Difficult Airway – Metabolic Acidosis

Physiologically Difficult Airway - Metabolic Acidosis

Case Presentation

A 32-year-old male with insulin-dependent diabetes mellitus came to your emergency department for shortness of breath. He was referred to the suspected COVID-19 area. His vitals were as follows: Blood pressure, 100/55 mmHg; pulse rate, 135 bpm; respiratory rate, 40/min; saturation on 10 liters of oxygen per minute, 91%; body temperature, 36.7 C. His finger-prick glucose was 350 mg/dl.

The patient reported that he had started to feel ill and had an episode of diarrhea 1 week ago. He developed a dry cough and fever in time. He started to feel shortness of breath for 2 days. He sought out the ER today because of the difficulty breathing and abdominal pain.

The patient seemed alert but mildly agitated. He was breathing effortfully and sweating excessively. On physical examination of the lungs, you noticed fine crackles on the right. Despite the patient reported abdominal pain, there were no signs of peritonitis on palpation.

An arterial blood gas analysis showed: pH 7.0, PCO2: 24, pO2: 56 HCO3: 8 Lactate: 3.

The point-of-care ultrasound of the lungs showed B lines and small foci of subpleural consolidations on the right.
At this point, what are your diagnostic hypotheses?

Two main diagnostic hypotheses here are:

  • Diabetic ketoacidosis (Hyperglycemia + metabolic acidosis)
  • SARS-CoV2 pneumonia

We avoid intubating patients with pure metabolic decompensation of DKA if possible, as they respond to hydration + insulin therapy + electrolyte replacement well and quickly. 

But in this scenario, the patient is extremely sick and has complicating medical issues, such as an acute lung disease decompensating the diabetic condition, probably COVID19. Considering these extra issues may complicate the recovery time and increase the risk of respiratory failure, you decide to intubate the patient in addition to the treatment of DKA.

You order lab tests and cultures. You start hydration and empirical antibiotics while starting preoxygenation and preparing for intubation.

Will this be a Difficult Airway?

Evaluating the patient for the predictors of a difficult airway is a part of the preparation for intubation. Based on your evaluation, you should create an intubation plan. 

This assessment is usually focused on anatomical changes that would make it difficult to manage the airway (visualization of the vocal cords, tube passage, ventilation, surgical airway), thereby placing the patient at risk.

“Does this patient have any changes that will hinder opening the mouth, mobilizing the cervical region, or cause any obstruction for laryngoscopy? Does this patient have any changes that hinder the use of Balloon-Valve-Mask properly, such as a large beard? What about the use of the supraglottic device? Does this patient have an anatomical alteration that would hinder emergency cricothyroidotomy or make it impossible, like a radiation scar? ”

So the anatomically difficult airway is when the patient is at risk if you are unable to intubate him due to anatomical problems.

The physiologically difficult airway, however, is when the patient has physiological changes that put him at risk of a bad outcome during or shortly after intubation. Despite intubation. Or because of intubation, because of its physiological changes due to positive pressure ventilation.

These changes need to be identified early and must be mitigated. You need to recognize the risks and stabilize the patient before proceeding to intubation or be prepared to deal with the potential complications immediately if they happen.

5 main physiological changes need attention before intubation are: hypoxemia, hypotension, severe metabolic acidosis, right ventricular failure, severe bronchospasm.

Back to our patient: Does he have physiologically difficult airway predictors?

  • SI (Shock Index): 1.35 (Normal <0.8) – signs of shock
  • P / F: 93 (Normal> 300) – Severe hypoxemia
  • pH: 7.0: Severe metabolic acidosis – expected pCO2: 20 (not compensating)
  • qSOFA: 2 + Lactate: 3 (severity predictor)

Physiologically Difficult Airway

"Severely critical patients with severe physiological changes who are at increased risk for cardiopulmonary collapse during or immediately after intubation."

Severe Metabolic Acidosis

In this post, we will focus only on the compensation of the metabolic part, but do not forget that this is a patient who needs attention on oxygenation and hemodynamics as well. That is, this is intubation with very difficult predictions.

What happens during the rapid sequence of intubation in severe metabolic acidosis?

To perform the procedure, the patient needs to be in apnea. During an apnea, pulmonary ventilation is decreased and the CO2 is not “washed” from the airway. These generate an accumulation of CO2, an acid, decreasing blood pH. In a patient with normal or slightly altered pH, this can be very well-tolerated, but in a patient with a pH of 7.0, an abrupt drop in this value can be ominous.

We know that the respiratory system is one of the most important compensation mechanisms for metabolic acidosis and it starts its action in seconds, increasing the pH by 50 to 75% in 2 to 3 minutes, guaranteeing the organism time to recover. So, even seconds without your proper actions can be risky for critical patients.

In addition, it must be remembered that increased RF is the very defense for the compensation of metabolic acidosis, and most of the time the organism does this very well. So if after the intubation the NORMAL FR and NORMAL minute volume are placed in the mechanical ventilator parameters, again there is an increase in CO2 and a further decrease in pH.

And what’s wrong? After all, a little bit of acidosis even facilitates the release of oxygen in the tissues because it deflects the oxyhemoglobin curve to the right, right?

Severe metabolic acidosis (pH <7.1) can have serious deleterious effects:

  • Arterial vasodilation (worsening shock)
  • Decreased myocardial contractility
  • Risks of arrhythmias
  • Resistance to the action of DVAs
  • Cellular dysfunction

What to do?

Always the primary initial treatment is: treating the underlying cause! In patients with severe metabolic acidosis, it is best to avoid intubation! Especially in metabolic ketoacidosis, which as hydration and insulin intake improves, there is a progressive improvement in blood pH.

Sodium bicarbonate

The use of sodium bicarbonate to treat metabolic acidosis is controversial, especially in non-critical acidosis values ​​(pH> 7.2). If you have acute renal failure associated, its use may be beneficial by postponing the need for renal replacement therapy (pH <7.2).

As for DKA, where sodium bicarbonate is used to the ketoacidosis formed by erratic metabolism due to the lack of insulin and no real deficiency is present, its use becomes limited to situations with pH <6.9.

The dose is empirical, and dilution requires a lot of attention (avoid performing HCO3 without diluting!)

NaHCO3 100mEq + AD 400ml

Run EV in 2h

If K <5.3: Associate KCl 10% 2amp

I would make this solution and leave it running while proceeding with the intubation preparations.

Attention: Remember, according to the formula below, that HCO3 is converted to CO2, and if done in excess, is associated with progressive improvement of the ketoacidosis and recovery of HCO3 from the buffering molecules. In a patient already with limited ventilation, its increase can cause deviation of the curve for the CO2 increase, which is also easily diffused to the cells and paradoxically decrease the intracellular pH, in addition to carrying K into the cell.

H + + HCO3 – = H2CO3 = CO2 + H2O

Mechanical ventilation

I think the most important part of the management of these patients is the respiratory part.

If you choose the Rapid Sequence Intubation: Prepare for the intubation to be performed as quickly as possible: Use your best material, choose the most experienced intubator, put the patient in ideal positioning, decide and apply medications skillfully, to ensure the shortest time possible apnea.

You will need personnel experienced in Mechanical Ventilation and you must remember to leave the ventilatory parameters adjusted to what the patient needs and not to what would be normal!

I found this practice very interesting: First, you calculate what the expected pCO2 should be for the patient, according to HCO3:

Winter’s Equation (Goal C02) = 1.5 X HCO3 + 8 (+/- 2)

And then, according to this table, you try to reach the VM Volume Minute value.
Goal CO2 Minute Ventilation
40 mmHg
6-8 L
30 mmHg
12-14 L
20 mmHg
18-20 L

These are just initial parameters. With each new blood gas analysis repeated in 30 minutes to an hour, you re-make fine adjustments using the formula below:

Minute volume = [PaCO2 x Minute volume (from VM)] / CO2 Desired

With the treatment of ketoacidosis, new parameters should be adjusted, hopefully for the better.

Another safer option for these patients would be to use the Awake Patient Intubation technique so that you would avoid the apnea period. However, Awake Patient Intubation Technique is contraindicated in suspected or confirmed COVID-19 cases due to the risk of contamination.

That’s it, folks, send your feedback, your experiences, and if you have other sources!

Further Reading

  1. Frank Lodeserto MD, “Simplifying Mechanical Ventilation – Part 3: Severe Metabolic Acidosis”, REBEL EM blog, June 18, 2018. Available at: https://rebelem.com/simplifying-mechanical-ventilation-part-3-severe-metabolic-acidosis/.
  2. Justin Morgenstern, “Emergency Airway Management Part 2: Is the patient ready for intubation?”, First10EM blog, November 6, 2017. Available at: https://first10em.com/airway-is-the-patient-ready/.
  3. Salim Rezaie, “How to Intubate the Critically Ill Like a Boss”, REBEL EM blog, May 3, 2019. Available at: https://rebelem.com/how-to-intubate-the-critically-ill-like-a-boss/.
  4. Salim Rezaie, “RSI, Predictors of Cardiac Arrest Post-Intubation, and Critically Ill Adults”, REBEL EM blog, May 10, 2018. Available at: https://rebelem.com/rsi-predictors-of-cardiac-arrest-post-intubation-and-critically-ill-adults/.
  5. Salim Rezaie, “Critical Care Updates: Resuscitation Sequence Intubation – pH Kills (Part 3 of 3)”, REBEL EM blog, October 3, 2016. Available at: https://rebelem.com/critical-care-updates-resuscitation-sequence-intubation-ph-kills-part-3-of-3/.
  6. Lauren Lacroix, “APPROACH TO THE PHYSIOLOGICALLY DIFFICULT AIRWAY”, https://emottawablog.com/2017/09/approach-to-the-physiologically-difficult-airway/
  7. Scott Weingart. The HOP Mnemonic and AirwayWorld.com Next Week. EMCrit Blog. Published on June 21, 2012. Accessed on July 15th 2020. Available at [https://emcrit.org/emcrit/hop-mnemonic/ ].
  8. IG: @pocusjedi: “Pocus e Coronavirus: o que sabemos até agora?”https://www.instagram.com/p/B-NxhrqFPI1/?igshid=14gs224a4pbff

References

  1. Sakles JC, Pacheco GS, Kovacs G, Mosier JM. The difficult airway refocused. Br J Anaesth. 2020;125(1):e18-e21. doi:10.1016/j.bja.2020.04.008
  2. Mosier JM, Joshi R, Hypes C, Pacheco G, Valenzuela T, Sakles JC. The Physiologically Difficult Airway. West J Emerg Med. 2015;16(7):1109-1117. doi:10.5811/westjem.2015.8.27467
  3. Irl B Hirsch, MDMichael Emmett, MD. Diabetic ketoacidosis and hyperosmolar hyperglycemic state in adults: Treatment. Post TW, ed. UpToDate. Waltham, MA: UpToDate Inc. https://www.uptodate.com (Accessed on July 15, 2020.)
  4. Cabrera JL, Auerbach JS, Merelman AH, Levitan RM. The High-Risk Airway. Emerg Med Clin North Am. 2020;38(2):401-417. doi:10.1016/j.emc.2020.01.008
  5. Guyton AC, HALL JE. Tratado de fisiologia medica. 13a ed. Rio de Janeiro(RJ): Elsevier, 2017. 1176 p.
  6. Kraut JA, Madias NE. Metabolic acidosis: pathophysiology, diagnosis and management. Nat Rev Nephrol. 2010;6(5):274-285. doi:10.1038/nrneph.2010.33
  7. Calvin A. Brown III, John C. Sakles, Nathan W. Mick. Manual de Walls para o Manejo da Via Aérea na Emergência. 5. ed. – Porto Alegre: Artmed, 2019.
  8. Smith MJ, Hayward SA, Innes SM, Miller ASC. Point-of-care lung ultrasound in patients with COVID-19 – a narrative review [published online ahead of print, 2020 Apr 10]. Anaesthesia. 2020;10.1111/anae.15082. doi:10.1111/anae.15082
[cite]

More Posts by Dr. Santos

Push Th(d)ose Vasopressors

Push Th(d)ose Vasopressors

Background

Since Scott Weingart first advocated for using push-dose pressors in the Emergency Department (ED) over a decade ago(1), push-dose vasopressors, also known as bolus-dose vasopressors have seemingly found their way into many EDs. However, recent studies have sought to ask more questions regarding its use and safety in the Emergency Department.

Vasopressors such as epinephrine and norepinephrine are commonly used for regulating and maintaining adequate blood pressure or mean arterial pressure (MAP). While these are usually administered as a continuous infusion via central access, administering them as a small bolus through peripheral access came to be known as push-dose vasopressor in practice.

Traditionally, this small bolus strategy was used in the operating room (OR) by anesthetists to treat transient hypotension due to sedating agents or spinal anesthesia. Multiple studies have supported the safety and efficacy of push-dose vasopressors in this clinical setting/patient population (2).

Swensen, et al. (3) studied the safety of bolus-dose phenylephrine for hypotension in the Emergency Department, however, data on the efficacy and safety of push-dose pressors remains sparse in ED and in-patient settings. Studies published in the past few years have questioned the lack of evidence regarding the safety and efficacy of push-dose pressor use in ED settings and highlighted some negative consequences of its use (4). To understand the concerns, it’s important we first understand the vasopressors, indications for use, and preparation in the ED.

Push-dose pressors in the Emergency Department

The two common vasopressors used as push-dose pressors in the Emergency Department are Epinephrine and Phenylephrine. Patients needing emergency airway, traumatic brain injury, and post-cardiac arrest with the return of spontaneous circulation may all experience hypotension which could lead to adverse outcomes. Push-does pressors have been proposed as a temporary measure to limit the hypotension while a vasopressor infusion/definitive treatment is being set up (5).

phenilephrine vs epinephrine
push dose epinephrine
push dose phenilephrine

Clinical settings in the ED where the use of push-dose pressor is proposed:

  1. Airway management: Hypotension prior, during, and post-intubation could be treated with bolus-dose vasopressors. Panchal et al. (6) did a retrospective chart review of intubated hypotensive patients in which phenylephrine was used. Bolus-dose phenylephrine demonstrated an increase in systolic blood pressure and the authors recommended further studies to understand the best use of phenylephrine for post-intubation hypotension.
  2. Return of spontaneous circulation (ROSC): In patients with ROSC, bolus-dose pressors may aid in the maintenance of end-organ perfusion, which is often impaired after ROSC (7).
  3. Traumatic brain injury: By rapidly increasing mean arterial pressure and thus cerebral perfusion pressure, bolus-dose vasopressors may help to prevent secondary brain injury.

What are the concerns regarding the use of push-dose pressors in the ED?

Acquisto and Bodkin (8) cited a few dosing errors while using push-dose pressors and highlighted that emergency physicians are less familiar with the practice of medication preparation/manipulation and hence dosing errors are expected, inadvertently causing patients more harm than benefit. They also emphasized on the lack of evidence in the literature regarding the efficacy and safety of push-dose pressors in a stressful environment like the ED.

Rotando and Picard et al. (9) in their prospective observational study of 146 patients receiving push-dose pressors in the ICU had thirteen (11.2%) patients have a dose-related medication error and seventeen (11.6%) adverse events. They concluded while push-dose pressors where efficacious, they were associated with adverse drug events and medication errors.

Cole et al (10). performed a retrospective analysis of 249 patients receiving push-dose pressors and found a higher incidence of adverse hemodynamic effects (39%) and human errors (19%). They emphasized the need for further studies to question whether push-dose pressors improve outcomes, and if so, how to safely implement them in practice.

Another concern raised is whether physicians may bypass standard resuscitation practices of fluid boluses in favor of using push-dose pressors. Schwartz et al. (11) found that only 34% of patients received an appropriate fluid challenge before using push-dose pressors in a retrospective chart review of 73 patients receiving push-dose pressors for acute hypotension in the ED. Furthermore, it appeared that patients who did not receive an appropriate fluid bolus needed more doses of bolus-dose pressors followed by the need for continuous vasopressor infusion within 30 minutes of bolus-dose pressor use.

Emergency physicians work in stressful environments which raises concerns on the ability of the physician to perform accurate dose calculations under duress (4). The prepared syringe also contains multiple individual doses, and using more concentrated solutions potentially increases the risk of overdose and extravasation injury (12).

Conclusion

While the practice of using push-dose pressors has found its way into the Emergency Department, it is crucial to acknowledge that evidence regarding its safety and benefits is limited. However, rather than disregarding the practice, high-quality research should be encouraged, which could potentially be practice-changing. Holden et al. (12) offer a framework of operational and safety considerations for the use of push-dose pressors in the ED and is a must-read for all using push-dose pressors in their current practice.

References

  1. Scott Weingart. EMCrit Podcast 6 – Push-Dose Pressors. EMCrit Blog. Published on July 10, 2009. Accessed on September 25th 2020. Available at [https://emcrit.org/emcrit/bolus-dose-pressors/ ].
  2. Lee A, Ngan Kee WD, Gin T. A quantitative, systematic review of randomized controlled trials of ephedrine versus phenylephrine for the management of hypotension during spinal anesthesia for cesarean delivery. Anesth Analg. 2002 Apr;94(4):920-6, table of contents. doi: 10.1097/00000539-200204000-00028. PMID: 11916798.
  3. Swenson K, Rankin S, Daconti L, Villarreal T, Langsjoen J, Braude D. Safety of bolus-dose phenylephrine for hypotensive emergency department patients. Am J Emerg Med. 2018 Oct;36(10):1802-1806. doi: 10.1016/j.ajem.2018.01.095. Epub 2018 Feb 19. PMID: 29472039.
  4. Cole JB. Bolus-Dose Vasopressors in the Emergency Department: First, Do No Harm; Second, More Evidence Is Needed. Ann Emerg Med. 2018 Jan;71(1):93-95. doi: 10.1016/j.annemergmed.2017.05.039. Epub 2017 Jul 26. PMID: 28754354.
  5. Weingart S. Push-dose pressors for immediate blood pressure control. Clin Exp Emerg Med. 2015;2(2):131-132. Published 2015 Jun 30. doi:10.15441/ceem.15.010
  6. Panchal AR, Satyanarayan A, Bahadir JD, Hays D, Mosier J. Efficacy of Bolus-dose Phenylephrine for Peri-intubation Hypotension. J Emerg Med. 2015 Oct;49(4):488-94. doi: 10.1016/j.jemermed.2015.04.033. Epub 2015 Jun 20. PMID: 26104846.
  7. Gottlieb M. Bolus dose of epinephrine for refractory post-arrest hypotension. CJEM. 2018 Oct;20(S2):S9-S13. doi: 10.1017/cem.2016.409. Epub 2017 Jan 10. PMID: 28069098.
  8. Acquisto NM, Bodkin RP, Johnstone C. Medication errors with push dose pressors in the emergency department and intensive care units. Am J Emerg Med. 2017 Dec;35(12):1964-1965. doi: 10.1016/j.ajem.2017.06.013. Epub 2017 Jun 7. PMID: 28625533.
  9. Rotando A, Picard L, Delibert S, Chase K, Jones CMC, Acquisto NM. Push dose pressors: Experience in critically ill patients outside of the operating room. Am J Emerg Med. 2019 Mar;37(3):494-498. doi: 10.1016/j.ajem.2018.12.001. Epub 2018 Dec 3. PMID: 30553634.
  10. Cole JB, Knack SK, Karl ER, Horton GB, Satpathy R, Driver BE. Human Errors and Adverse Hemodynamic Events Related to “Push Dose Pressors” in the Emergency Department. J Med Toxicol. 2019 Oct;15(4):276-286. doi: 10.1007/s13181-019-00716-z. Epub 2019 Jul 3. PMID: 31270748; PMCID: PMC6825064.
  11. Schwartz MB, Ferreira JA, Aaronson PM. The impact of push-dose phenylephrine use on subsequent preload expansion in the ED setting. Am J Emerg Med. 2016 Dec;34(12):2419-2422. doi: 10.1016/j.ajem.2016.09.041. Epub 2016 Sep 22. PMID: 27720568.
  12. Holden D, Ramich J, Timm E, Pauze D, Lesar T. Safety Considerations and Guideline-Based Safe Use Recommendations for “Bolus-Dose” Vasopressors in the Emergency Department. Ann Emerg Med. 2018 Jan;71(1):83-92. doi: 10.1016/j.annemergmed.2017.04.021. PMID: 28601272.
[cite]

More Posts From Dr. Hudlikar

Oxygenation and Oximetry

Oxygenation and Oximetry

Authors: Job Rodríguez Guillén, Chief of Emergency Department. Hospital H+ Querétaro, México. Regina Pineda Leyte Internal Medic, Anahuac Querétaro University, Mexico. 

Introduction

One of the main goals of mechanical ventilation is oxygenation. Both hypoxemia and hyperoxemia must be avoided and the objectives must be individualized according to the clinical situation and comorbidities of each patient. Oxygenation monitoring is possible at the bedside by physical examination (late clinical signs), pulse oximetry (non-invasive continuous monitoring), and arterial blood gas analysis (gold standard for arterial oxygenation analysis).

Determinants of oxygenation

The main determinant of oxygenation is the mean airway pressure (Paw) and the inspired fraction of oxygen (FiO2). Paw is the average pressure to which the lung is exposed during inspiration and expiration mechanical ventilation (Figure 1). Paw improves oxygenation by allowing the redistribution of oxygen from highly compliant alveoli to less compliant alveoli.(1,2)
Oxygenation and Oximetry - figure 1
Figure 1: Mean airway pressure (Paw) is the integral (area under the curve) of pressure and time. PIP: peak inspiratory pressure; PEEP: positive pressure at the end of expiration; Ti: inspiratory time; Te: expiratory time.
According to the determinants of Paw and the relationship between them, there are five different ways to increase it (Figure 2)
Oxygenation and Oximetry - figure 2
Figure 2: Maneuvers to increase the mean airway pressure (Paw). PEEP: positive pressure at the end of expiration. Only maneuvers 3 and 4 are used in clinical practice to increase Paw and improve oxygenation.

The second determinant of oxygenation is Inspired Oxygen Fraction (FiO2). The use of supplemental oxygen at the hospital level is a common practice and a critical element of intensive care in patients with mechanical ventilation for the management of hypoxemia. However, in recent years it has been shown that higher oxygenation is not the goal. (3) In the same way that hypoxemia should be avoided, hyperoxia should be prevented. (4)

Although the FiO2 can be adjusted in ranges of 21% and up to 100% the lowest value required must be set (preferably <60%) to reach the desired oxygen saturation (SO2) target.

Oxygenation monitoring

Pulse oximetry allows non-invasive monitoring of oxygenation (SpO2), it is simple and reliable in many areas of clinical practice. SpO2 has a confidence rate of 95% ± 4%, so readings ranging between 70% and 100% are considered reliable.(5) In patients with mechanical ventilation, the objective is to identify hypoxemia.

It is important to remember that oximeters do not measure arterial oxygen pressure (PaO2), for this reason, they cannot directly diagnose hypoxemia or hyperoxemia (PaO2 <60 mmHg and PaO2> 120 mmHg respectively).(6)  What they do is “estimate” hypoxemia when SpO2 falls <90%, which would correspond to a PaO2 <60 mmHg according to the oxyhemoglobin dissociation curve (Table 1). (7)  However, it must be taken into account that changes in temperature and pH cause changes in this relationship. As the pH increases (alkalosis) or the temperature decreases (hypothermia), the shift of the curve is to the left since hemoglobin binds more strongly with oxygen, delaying its release to the tissues. Acidosis and fever shift the curve to the right as the hemoglobin molecule decreases its affinity for oxygen, facilitating the release of oxygen to the tissues.

Oxygenation and Oximetry - Table 1
Table 1: Estimation of the oxygenation state according to SpO2. SpO2: oxygen saturation by pulse oximetry; PaO2: arterial oxygen pressure.

SpO2 values <70% are not reliable. If necessary, the oxygenation assessment should be supplemented by arterial gas analysis. The arterial oxygen saturation (SaO2) is the oxygen saturation obtained by this test.

Oxygenation Goals

According to the oxyhemoglobin dissociation curve, the goal of oxygen titration is to achieve a PaO2 in the range of 60-65 mmHg or an SpO2 of approximately 90-92%. However, the objectives must be individualized and the current recommendations for oxygen therapy in critically ill patients. (8) are as follows (Table 2).

Table 2: Recommendations for oxygenation by SpO2. O2: oxygen; SpO2: oxygen saturation by pulse oximetry; AMI: Acute myocardial infarction; EVC: Cerebral vascular event; VM: mechanical ventilation; SIRA: Acute respiratory distress syndrome. Some exceptions apply like carbon monoxide poisoning.

It has been suggested that critically ill patients can tolerate lower levels of PaO2 (“permissive hypoxemia”) (9-10), however, studies are limited to make a recommendation to routine clinical practice.

Conclusions

Oxygenation goals should be established once the requirement for mechanical ventilation is indicated according to the clinical condition of each patient and monitoring that these objectives are met. Pulse oximetry allows continuous, non-invasive monitoring at the bedside. It should be remembered that hyperoxemia, as well as hypoxemia, should be avoided.

References

  1. Marini JJ,Ravenscraft SA. Mean airway pressure: physiologic determinants and clinical importance–Part 1: Physiologic determinants and measurements. Crit Care Med. 1992 Oct;20(10):1461-72.
  2. Marini JJ,Ravenscraft SA. Mean airway pressure: physiologic determinants and clinical importance–Part 2: Clinical implications. Crit Care Med. 1992 Nov;20(11):1604-16.
  3. Girardis M, Busani S, Damiani E, et al. Effect of conservative vs conventional oxygen therapy on mortality among patients in an intensive care unit: the Oxygen-ICU Randomized Clinical Trial. JAMA. 2016;316(15):1583-1589.
  4. Bitterman, Haim. “Bench-to-bedside review: oxygen as a drug.” Critical Care1 (2009): 205.
  5. Chan MM,Chan MM, Chan ED. What is the effect of fingernail polish on pulse oximetry?. Chest. 2003 Jun;123(6):2163-4.
  6. Wandrup JH. Quantifying pulmonary oxygen transfer deficits in critically ill patients. Acta Anaesthesiol Scand Suppl 1995;107:37–44
  7. Allen J. Photoplethysmography and its application in clinical physiological measurement. Physiol Meas 2007;28:R1–39
  8. Siemieniuk Reed A C, Chu Derek K, Kim Lisa Ha-Yeon, Güell-Rous Maria-Rosa, Alhazzani Waleed, Soccal Paola M et al. Oxygen therapy for acutely ill medical patients: a clinical practice guideline BMJ 2018;  363 :k4169
  9. Gilbert-Kawai ET, Mitchell K, Martin D, Carlisle J, Grocott MP. Permissive hypoxaemia versus normoxaemia for mechanically ventilated critically ill patients. Cochrane Database Syst Rev 2014;5:CD009931.
  10. Capellier G, Panwar R. Is it time for permissive hypoxaemia in the intensive care unit? Crit Care Resusc 2011;13:139–141.
[cite]

3D Video Laryngoscopes

Laryngoscopy can be described as endoscopy of the larynx, which used to facilitate tracheal intubation during general anesthesia or cardiopulmonary resuscitation. For decades, direct laryngoscopy has been the standard technique for tracheal intubation. But today, there are two main types of laryngoscopy: direct and indirect. Indirect laryngoscopy means the provider visualizes the patient’s vocal cords without having a direct line of sight. Indirect Laryngoscopy includes video laryngoscopes, fiberoptic bronchoscopes, and optically-enhanced laryngoscopes. Video laryngoscopy introduced in recent years and it aims to overcome the limitations of direct laryngoscopy by using a camera attached to the laryngoscope. While it has clear advantages over direct laryngoscopy, video laryngoscopy still has a high cost of investment. It remains a rare commodity for Emergency Medicine clinics, especially in resource-limited settings.

While the COVID-19 pandemic was affecting the world, the people who were under the most significant risk were healthcare workers. We know that the risk of transmission of the disease is quite high, especially when performing high-risk medical procedures such as endotracheal intubation. It is a known fact that personal protective equipment such as masks or face shields are very important in protection. But it is even more important to stay physically away from the patient whenever possible. When intubating a patient, video laryngoscopy has a clear advantage in terms of eliminating the need to approach the patient’s head and trying to have a direct line of sight.

Video laryngoscopy devices are expensive. But, if you think about the essential components of it, you can easily realize that it doesn’t have to be this way. You need a blade, a camera system, a display, and a way to attach the blade and the camera system. While laryngoscopy blades are essential for Emergency clinics anyway, I can safely assume every Emergency clinic has them. A camera system and a display are also both fairly cheap and easy to obtain for most of the places on earth. Find those three and voila! You have a cheap video laryngoscope (In this post, I will not elaborate on the technique of combining a normal blade with a video camera).

For those who want to go to the next level, there are some ways of making your very own prettier video laryngoscopy devices. You just need a 3D printer, but luckily it is possible to find 3D printers in many cities these days.

So here we go.

Umay

The pandemic paved the way for innovation in many ways. Numerous doctors from all over the world rolled up their sleeves to develop new medical devices. Yasemin Özdamar, an Emergency Medicine specialist from Turkey, designed 3D-printable video laryngoscope blades named “Umay” (possibly an allusion to Orkhon inscriptions) in pediatric and adult forms based on normal laryngoscope blades.

The printing files of these blades can be downloaded for free in formats suitable for printing with PLA material, which is frequently used in 3D printers, and PA12, which is preferred for more professional printing. You can download the files here: Pediatric – Adult.

AirAngel

AirAngel is a not-for-profit tutorial center dedicated to making video laryngoscopes accessible in under-resourced nations. You can purchase the blade or video laryngoscopy devices from their website with a fairly low price of US$100-180. You can also get the file of the blade for free and 3D print it yourself. Its design is really similar to a D blade. You can head to AirAngel’s website and grab the printing file now.

Here is an example tutorial for AirAngel:

In our tests (in Turkey), the cost of printing one blade approximately 50 Turkish Liras (roughly equal to US$7 with today’s exchange rates). We also bought a “Borescope USB Camera” with a camera head outer diameter of 5.5mm from our local internet store for approximately US$13 (A similar product from Amazon). So, the cost was US$20 in total, which is cheaper than AirAngel’s offer, and a lot cheaper than a conventional video laryngoscope. We have attached the camera to the blade using special parts on them and connected the camera to a phone. And under a minute, a video laryngoscope was born.

Please note: The intended purpose of these designs is to be used as a training tool. They do not replace any medical-grade video laryngoscope systems. They are not in any way approved medical device designs, nor have they been reviewed by the FDA or any other organization. Be aware that many plastics vary in strength, heat resistance, and chemical resistance. The strength and durability of the blade will vary depending on what you print it with. Harmful and life-threatening complications may occur if pieces break in the airway.

[cite]